111
Nivelovonzodo Troslodondo el orgumenlo de Gódel del dominio mqtemótico ol reino de los seres humonos v sus creencios. El outor explico losbqses de losideos de Gódel en un lenguoie comprensivo porotodos. Unoobundqnte colección de ocerliios sobre mentirosos y veroces (bribones y cobolleros) ocompoño uno descripción infroductorio de lo lógico simbólico y uno explicoción de lo formo de resolver grupos enleros de esle tipode ocerlijos. En loscopílulos posteriores se encuentrq, porejemplo el temqde los<creenciqs qutosuficientesr. ¿Cómo pueoe unq mero creencto converlirse en unoconvicción de que es cierto o trqvés de delerminqdq proposición? ¿Acoso se trqlo oquíde un fenómeno religioso? Smullyon explico cómoesos sistemos de creencios se relocionqn con ¡mportontes sislemos molemóficos y,finomente con lq (semónlico de losmundos posibles>, iniciqdq por Leibniz y perfeccionodo porel lógico Soul Kripke. misteriosor ...-:i liililniLrili[iinrinrr 9"7 88t+7 4"323O8"" lisa editor¡al {l 1 i::, i., !: : : ,i:l i: ¡t' aa:t. -r,: ' -ri' u:ll ta ja ,: t. ,i llr :l e¡. .':.¡i. ,,,:l t, ii: 1t ,i ,rr' i:l *{: I ¡:,- f |.j t. . t.i ' ,,.if' :, ti ta t: :: i: l *1 it, :: i: il*.¡{: rir .,,,..n ,,.-{l ':l ' ,,,,d.' & ReKñeg& :rr*f.il, ':$ .'r{rl-_.

SMULLYAN, R. Juegos por siempre misteriosos. España. Gedisa 2008

Embed Size (px)

Citation preview

Page 1: SMULLYAN, R. Juegos por siempre misteriosos. España. Gedisa 2008

Nivelovonzodo

Troslodondo el orgumenlo deGódel del dominio

mqtemótico ol reino de losseres humonos v sus

creencios. El outorexplico los bqses de los ideos

de Gódel en un lenguoiecomprensivo poro todos.

Uno obundqnte colección deocerliios sobre mentirosos y

veroces (bribones ycobolleros) ocompoño uno

descripción infroductorio de lológico simbólico y uno

explicoción de lo formo deresolver grupos enleros de

esle tipo de ocerli jos.En los copílulos posteriores

se encuentrq, por ejemplo eltemq de los <creenciqs

qutosuficientesr. ¿Cómopueoe unq mero creencto

converlirse en uno convicciónde que es cierto o trqvés de

delerminqdq proposición?

¿Acoso se trqlo oquí de unfenómeno religioso? Smullyon

explico cómo esos sistemosde creencios se relocionqncon ¡mportontes sislemos

molemóficos y, finomente conlq (semónlico de los mundosposibles>, iniciqdq por Leibnizy perfeccionodo por el lógico

Soul Kripke.

misteriosor

...-:i

liililniLrili[iinrinrr9"7 88t+7 4"323O8""

lisaeditor¡al

{ l1

i : : , i . ,!: :

: ,i:l

i: ¡t'aa:t .

-r,: ' -ri'

u: l lta ja

,: t. ,il l r " ¡ : l

e¡. .':.¡i.

,,,:l

t, ii:1t,i ,rr'

i:l

*{: I¡:,- f

| . j

t .

. t . i' ,,.if'

:, ti

ta t:

: : i : l *1 i t ,: : i : i l * . ¡ { :

rir .,,,..n _¡ ,,.-{l':l ',,,,d.'

& ReKñeg&:rr*f.il,

':$.'r{rl-_.

Page 2: SMULLYAN, R. Juegos por siempre misteriosos. España. Gedisa 2008

,.\ I ' i

Page 3: SMULLYAN, R. Juegos por siempre misteriosos. España. Gedisa 2008

t-Juegos por siempre

misteriosos

Page 4: SMULLYAN, R. Juegos por siempre misteriosos. España. Gedisa 2008

Colecciónjuegos

Carlo Frabetti

Jean-Louis Fournier

Raymond Smullyan

Enric Camous

Eric Emmet

Gyles Brandeth

Raymond Smullyan

lan Stewart

Raymond Smullyan

Eric Emmet

J. Friant y Y. L'Hospitalier

Martin Gardner

Raymond Smullyan

Raymond Smullyan

Martin Gardner

Nicholas Falletta

James F. Fixx

James L. Adams

Jean-Pierre Alem

Jean-Pierre Alem

El Tablero mágico

Aritmética aplicada e impertinente

Satán, Cantor y el lnfinlto (en prensa)

Problemas y juegos con la matemátlca

Juegos pafa devanarse los sesos

Juegos con números

Juegos para im¡tar a un pájaro imitador

Ingen¡osos encuentros entre juego y matemát¡ca

Juegos por siempre misteriosos

Juegos de acerti¡os enigmáticos

Juegos lógicos en el mundode la inteligencia artificial

Juegos, Los mágicos números del Dr, Matrix

Juegos de ajedrez y los misterlososcaballos de Arabla

Juegos y problemas de ajedrezpara Sherlock Holmes

Juegos y en¡gmas de otros mundos

Parado¡as y juegos

Juegos de recreación mental pa¡a muy inteligentes

Gu¡a y juegos para superar bloqueos mentales

Nuevos juegos de lngenio y entrctenimiento matemático

Juegos de ingenio y entretenim¡ento matemático

Raymond Smullyan

Juegos por s¡empremisteriosos

gedJ$,?

Page 5: SMULLYAN, R. Juegos por siempre misteriosos. España. Gedisa 2008

Título del original en inglés:Forever UndecidedO by Raymond Smullyan. This translation published byarrengement with Alfred A. Knopf, Inc., 1987

Tr adu c c i ó n : Margarita MizrajiSupervisión Técnica: Dr. Humberto R. AlagiaCubierta: departamento de diseño Editorial Gedisa

Tercera reedición: octubre de 2008. Barcelona

Derechos reservados para todas las ediciones en castellano

O Editorial Gedisa, S.A.Avda. Tibidabo, 12 3'08022 Barcelona (España)Tel. 93 253 09 04Fax 93 253 09 [email protected]

ISBN: 978-84 -7 432-308-5Depósito legal: B. 41797-2008

Impreso por Sagrafic

Impreso en EspañaPrinted in Spain

Queda prohibida la reproducción total o parcial por cualquiermedio de impresión, en forma idéntica, extractada o modificada,en castellano o en cualquier otro idioma.

Dedicad,o a todos los razonadorq consistent* quenunca podrán saber que son consistentes.

Page 6: SMULLYAN, R. Juegos por siempre misteriosos. España. Gedisa 2008

II\IDI@

, - . , ,1- ' - ' i l

, PA¡ffEI:¡QUIZA PODRA SOaPRENDERTE!

Cap'rrulo I: Un acertiF diebóltcó ..........,.:¡...,....,.;.,..-....,.....,.....,............., 15Capftulo lI: ¿Sotprendidos? .,¡¡...,...i....-..............,.;,..;...,.,...........;..;,r;.....;.... 19

. , : : . .P^ARTE II:

I.A, IOGICA DE MENIIR Y DE DECIR I.A VERDAD

cap-rWlo 3: El empadrroncdoi j...............,.......,.,,....;,,,.,....,25C¿pltulo 4r En br¡sca de OonaCaffrulo 5: Un laberinto interplaneta¡io ..,,..;..............;...,.........,.....,........35

'

PART! III:CABAILEROS, BRIBONES Y IOGICÁ, PROPOSICIONAL

. i

Cepltulo 6: LJn poco de lóglca proposiciona|......................................:...€Caplh¡üó 7: Caballeros, bribonee yllfgiea proposicional ........................,51C.ePlh¡lo & Clausura y co.¡sistenci¡ lógicas.......................-.............,..,...:56

PARTE IV:sne¡r¡o5-dr¡lprbosos'

Capltulo 9: ¿Paredójico?................................ ' ............r...............................67C¡plh¡lo 10: EX problema se profundiza .......¡i..:..,..........................-.....75

PARII V:IA COMPUC'ADA PROBIE¡"U\:NCA,

DE T.ACONSISTENCTA

C.aplnrlo t1: Los lógicos que razomn sobre sl ¡nismoo...,.......,..:..-...,..85Caplnrlo 12: Ia sorprendente proHerntltha de la consister¡ch ......,,,..,..ytCaplurlo 13: Sistemac gttdeliar¡oo .............98Crplurlo l4z fulás problcmae con la co¡rs!*e¡rch..;........................."...... 1ül

Page 7: SMULLYAN, R. Juegos por siempre misteriosos. España. Gedisa 2008

PARTE M:Iá'S CREENCIAS AUTOSUFICIENTES

Y Et TEOREMA DE tÓB

Capítulo 15: Creencies que se autocumplen .........................................111Capítulo 16: El diamante del Rajá ......... 119Capín:lo 17: La isla de Lób........... .........124

PARTE VII:EN AGUAS MAS PROFUNDAS

Capítulo18:Razonadoresdet ipoG.. . . . . . . . . . . . . . . . . . . . . . . . . . . . .133Capltulo 19: Modestia, reflo<ividad y estabilidad .......... 140

PARTE MII:IMPOSIBLE DE DECIDIR

Capítulo 20: Por siempre indecidible ............................. 149Capítulo21:¡Aunmásindecis iones!. . . . . . . . . . . . . . . . . . . . . . . . . . . . . . .155

PARTE TX:MUNDOS POSIBLES

Capítulo 22: ¡No es necesariamente así! . . . . . . . . . . . . . . . . . . . . . . . . .763Capítulo 23: Mundos posibles .. . . . . . . . . . . . . . .169Capítulo 24: De la necesidad a la demostrabi l idad .. . . . . . . . . . . . . . . . . . . . . . . . . . . . . .174

PARTE X:EL NUCLEO DE LA CUESTION

Capítulo 25: Un univerSo gtrdel izado... , . . . . . . . . . . . . . . . . . . . . . . . . .179Capítulo 26: Algunas notables máquinas lógicas .. . . . . . . . . . . . . . . . . . . . . . . . . . . . . . . . .1g6Capítulo 27: Sistemas modales autoaplicados ...............1!!

PARTE )C:FINAL

Capítulo 28: Sistemas modales, máquinas y ra2onadores.....................203Capítulo 29: Razonadores extraños ........208Capítulo 30: Retrospectiva .. . . . . . . . . . . . . . . . . . . . . . . . . . . . . . . . . . . . . . . . . . . . .214

Prefacio

¿Puede un ser humano racional estar en una posición en la que no lees posible creer que esponsistente, sin perder su consistencia en el proceso?Este es uno de los temas principales de esta obra. Está basada en el modelodel famoso descubrimiento de Kurt Gódel (el llamado Segundo Teoremade Incompleütud) que postula que todo sistema matemático consistentecon suficiente poder para re lizar lo que se conoce como aritmética ele-mental debe padecer la sorprendente limitación de no poder nunca de-mostrar su propia consistencia.

Existen varias razones por las cuales he trasladado el argumento deGódel desde el dominio de los sistemas matemáticos y de las proposicionesen ellos demostrables hasta el reino de los seres humanos y de las proposi-ciones que ellos crcen Los seres humanos y sus creencias resultan muchomás familiares parael no esp€cialista que los sistemas matemáticos abstrac-tos, y por eso puedo así explicar las bases de las ideas de Gódel en unlenguaje que todos pueden comprender. Además, el hecho de poner estascuestiones en términos humanos tiene un enorme atractivo psicológico yresulta sumamente importante para el campo en constante avance de lainteligencia artificial.

Como en mis anteriores libros de acertiios, comienzo con un sinnúmerode acertijos sobre menürosos y veraces (bribones y caballeros). Además dela novedad de estos problemas (prácticamente todos se publican aquí porprimera vez), liray un importante rasgo adicional: intercalado en estosacertijos hay una descripción int¡oductoria ala lógícasimbólic,a (tema queafortunadamente se enseña hoy en día en muchos de nuestros colegiossecundarios) y una explicación sobre la forma en que esta valiosa técnicapuede resolver sistemáticamente grupos enteros de acertijos del üpo caba-llero-bribón. (Esto debería ser de particular interés para los profesores delógica -ya sea en el nivel secundario o universitario- que deseen vivificarsus cursos de lÓgica con acertijos creativos y movilizadores.) Por lo tanto,el conocimiento que el lector adquirirá resultará apropiado para esclarecerlos capítulos posteriores y más profundos sobre la obra de Gódel.

Otro de los temas de los últimos capíhllos -y de igual interés paralainteligencia artificial- es el de las creenclas autosufrclentes ¿En qué cir-q¡nstancias puede la simple creencia en una proposición dar como resul-tado la convicción de que es cierta? QPuede esto tener algo que ver con

1l

Page 8: SMULLYAN, R. Juegos por siempre misteriosos. España. Gedisa 2008

la rcligión?) Hay un importante teorema del lógico M. H. tób que resultamuy importante en este caso. Ese teorema se relaciona estrechamente conel teorema de Gódel, y cuando se enuncia al comienzo en términos derazonadores y de sus creencias, es fácilmente accesible al lector común.

. Aunque en principio pongo el acento en sistemas de creencias, deninguna manera me quedo ahí. A medida que el libro avanza, se lleva allector a una comprensión cada vez más profunda sobre la forma en queesos siste'mas de creencias se relacionan con importantes sistemas matemá-ticos. Esto a su vez nos lleva al tema filosóficaméne fascinante de lasemán-t^ica del mundo posible: campo concebido por Ieibniz y llevado a su per-fección en este siglo por el lógico saul Kripke. Ademái este tema deslm-p.la r1n papel imp^ortante hoy en {ta enla ciencia de la computación y enla inteligenci a artificial.

He dado gran canüdad de conferencias sobre todo este material paragrupos tan diversos como brillantes de estudiantes secundarios y docóresen matemática, filosoffa y computación. La respuesta de ambos grupos fueigualmente gratificante: sinüeron curiosidad. En realidad, cualqu-ier neófitoque sepa algo de matemática o de ciencia puede dominar completamentetodo este libro (aunque se necesitará un póco de esmero), aunque más deun experto encontrará aquí un caudal de material nuevo y renovado.

Es notable la forma en que la lógica, la filosofla, la psicóloga, la intelt_gencia artificial, Ia ciencia de la computación y la maiemáticise acercancada vez más en estos días. ¡Vivimos en una era apasionante!

Parte I

¡QUIZA PODRASORPREI\TDERLE!

RAYMoND SMULLYANElka Pat*,N.Y.

Julio de 1986

t2

Page 9: SMULLYAN, R. Juegos por siempre misteriosos. España. Gedisa 2008

Un acertijo diabólico

Plerso que el siguiente acertijo bien puede ser el enigrna más diabGlico que se haya inventado (y si es así, me atribuyo con orgullo el méritode la creación).

Dos personas-Ay B-hacen cadaunaunaofefta,que se da más aba-jo. El problema es determina¡ qué oferta es la mejor.

Oferta de A. Tienen que formular un enunciado. Si el enunciado esverdadero, ganan exactamente diez dólares. Si el enunciado es falso, enton-ces ganan menos o más de diez dólares, pero no diez dÓlares exact¿mente.

Oferta de B. Tienen que formular un enunciado. Sea el enunciado ver-dadero o falso, ganan más de üez dólares.

¿Cuál de las dos ofertas preferirfan? la meyoñade la gente decide quela oferta de B es la mejor, dzdo que garantlza más de diez dÓlares, mientrasque con la oferta de A, no llay cefteza de ganar más de diez. Y realmenteparece que la oferta de B es mejor, pero a veces las apariencias engañan.En realidad, haré mi propia oferta: Si alguno de ustedes está dispuesto aproponerme la oferta de A, les pagaé veinte dólares por adelantádo. ¿Algu-no fuega? (Antes de aceptar mi propuesta, les conviene leer el resto de estecapítulo.)

No voy a dzr lt solución del problema que acabo de plantear hastahaber considerado en primer lugar algunos otros acerüjos afines un pocomás simples.

En mi libro To Mock a Mocklrgblr{ presenté el siguiente acertijo:Supongamos que ofrezco dos premios: Premio I y Premio 2. Tienen queformular un enunciado. Si el enunciado es verdadero, entonces debo dadesuno de los dos premios (sin decir cuál de los dos). Si su enunciado es falso,entonces no ganan ningún ilremio. Obviamenüe pueden estar seguros deganar r¡no de los dos premios si dicen: 'Dos más dos es cuatto", perosupongamos que desean el premio 1; ¿qué enunciado podlan formular queles garanüce que ganarán el premio 1?

I¿ solución que di es que deben decir: "Usted no me dará el premio2'. Si el enuncrado es falso, entonces lo que dice no es cierto, lo cual significaque les daré el premio 2.Pero no puedo darles un premio por formular

15

Page 10: SMULLYAN, R. Juegos por siempre misteriosos. España. Gedisa 2008

un enunciado falso, y en conseoencia el enunciado no puede ser falso.Por lo tanto, debe ser verdadero. Dado que es verdadero, entonces lo quedice es cierto, lo cual significa que no ganarán el premio 2. Pero dado quesu enunciado era verdadero, debo darles uno de los dos premios, y dadoque no es el premio 2, debe ser el premio 1.

Como veremos enseguida, este pequeño acertijo se relaciona estre-chamente con el famoso Teorema de la Incompletitud de Gódel. Paradescubrir cómo, conqideremos un acertijo similar (en realidad, el mismoacertijo planteado de otra forma). En primer lugar, vamos a la Isla de losCaballeros ylos Bribones (que desempeña un papel destacadoeneste libro)en la que cada habitante es un caballero o un bribón. En esta isla loscaballeros sólo formulan enunciados verdaderos, y los bribones sólo enun-ciados falsos.

Ahora supongamos que hay dos clubes en esta isla: Club I y Club II.Sólo los caballeros pueden ser socios de estos clubes; los bribones estánexcluidos rigurosamente de ambos. Además,cadacaballero essocio de unoy sólo uno de los dos clubes. Un día visitamos la isla y nos enconftamoscon un naüvo desconocido que formula un enunciado a partir del cualpodemos deducir que debe ser un miembro del Club L ¿Qué enunciadopuede haber formulado a parür del cual podamos deducir eso?

En analogía con el úlümo problema, el hablante pudo haber dicho: "Nosoy socio del Club II". Si el hablante fuera un bribón, entonces verdadera-mente no podría ser socio del Club II y hubiera formulado un enunciadoverdadero, lo cual un bribón no puede hacer. Por Io tanto debe ser uncaballero, y su enunciado debe haber sido verdadero; en realidad no essocio del Club II. Pero dado que es un caballero, debe sersocio de un club;por lo tanto pertenece al Club I.

La analogia debe de resultar evidente: El Club I corresponde a aque-llos que formulan enunciados verdaderos y ganarln el premio 1; el ClubII corresponde a los que formulan enunciados verdaderos y ganarán elpremio 2.

Estos acertijos encarnan la idea esencial que subyace en el famosoenunciado de Gódel que afirma su carácter de no demostrable en unsistemamatemático dado. Supongamos que clasificamos todas las oraciones uer-daderas del sistema (como los caballeros en el acerüio anterior) en dosgrupos: El grupo I consiste en todas las oraciones del sistema que, aunqueson verdaderas, no son detnostrables en el sistema; y el grupo II consisteen todas las oraciones que no sólo son verdaderas sino que realmente sondemostrables en el sistema. Lo que hizo Gódel fue construir una oraciónque afirmaba que pertenecía al grupo II; la oración puede parafrasearsecomo: "Nosoy demostrable en el sistema". Si la oración fuera falsa, entonceslo que dice no sería cierto, lo que significaría que es demostrable en el sis-tema, lo cual no puede ser (dado que todas las oraciones demost¡ables enel sistema son verdaderas). Por lo tanto la oración debe ser verdadera, ytal como lo dice, no es demostrable en el sisüema. Por lo tanto la oraciónde Gódel es verdadera, pero no es demostrable en el sistema.

l6

Di¡emos muct¡o más sobre las oraciones de Gódel en el curso de estaoir¡a. Por ahora, voy a considerar algunas variantes más sobre el acertijodel premio.

1. Primqauariante

Nuevamente ofrezco dos premios: Premio 1 y premio 2. Si formulanun enunciado verdadero, les daré por lo menos uno de los dos premios yposiblemente ambos. Si formulan un enunciado falso, no ganán ningúnpremio. Supongamos que son ambiciosos y quieren ganar ambos premios.¿Qué enunciado formula¡lan? (I¿s soluciones aparecen al final de los capí-tulos a menos que se especifique lo contrario.)

2. Segunda uariante

En este caso las reglas cambian un poco: Si formulan un enunciadoverdadero, ganan el premio 2; si formulan un enunciado falso, no gananel premio 2 (pueden o no ganar el premio 1). Ahora bien, ¿qué enunciadoles hará ganar el premio 1.?

3. Una uariante peruelsa.

- Supongamos que en un estado de ánimo perverso, les digo ahora que

si formulan un enunciado falso, ganarán uno de los dos premios, peró siformulan un enunciado verdadero, no ganan ningún prerrúo. ¿eué enun-ciado ganari el premio 1?

4. Más sobre el acertijo diabólico

Ahora regresemos al acertijo que abrió este capítulo. Lo diabólico fueml ofer¡z de pagarles veinte dólares por hacerm e la oferta de A, porque siaceptaban hacerlo, podría haberles ganado todo el dinero que-quiiiera,digamos, un miüón de dólares. ¿pueden descubrir cómo?

SOLUCIONES

. 1. Un enunciado que sirve es: "O gano ambos premios o ninguno". Siel enunciado es falso, entonces lo que dice no es cierto, Io cual significaque ganarán exactamente un premio. Pero no pueden ganar un premio porun enunciado falso. Por lo tanto el enunciado debe de ser verdadero, y réal-mente ganarán ambos premios o ninguno. Dado que no formularon unenunciado falso, cuyo resultado sería no ganar ningún premio, deben ganarambos premios.

t7

Page 11: SMULLYAN, R. Juegos por siempre misteriosos. España. Gedisa 2008

2. Simplemente deben decir: 'No ganaté ningún premio". Si el enun-ciado es verdadero, entonces por un lado no ganan ningún premio (comodice el enunciado), pero por otro lado, ganan el premio 2 por haber for-mulado un enunciado verdadero. Esta es una evidente contradicción; enconsecuencia el enunciado debe de ser falso. Entonces, al contrario de loque dice el enunciado, deben ganarun premio. No pueden gañarel premio2 por un enunciado falso; por lo tanto ganan el premio 1.

J. En este caso deben decir: "Ganaré el premio 2". Dejo la demostra-ción en manos del lector.

4. Lo único que tengo que hacer es decir: "Usted no me pagará exac-tamente ni diez dólares ni exactamente un millón de dólares". Si mi enun-ciado es verdadero, entonces por un lado ustedes no me pagaráoexac-tamente diez dólares ni exactamente un millón de dólares, pero por otrolado deben pag rme exactamente diez dólares por haber formulado unenunciado verdadero. Esto es una conEadicciÓn; en consecuencia el enun-ciado no puede ser verdadero y debe ser falso. Dado que es falso, lo quedice no es cierto, lo cual significa que ustedes me pagarán exactamen-te diez dólares o exactamente un millón de dólares, pero no pueden pagar-me exactamente diez dólaies por un enunciado falso; ¡por lo tanto debenpagarme un millón de dólares!

¿Alguno sigue jugando?

¿Sorprendido9Ahora consideremos la páradoja de la prueba sorpresa, paradoja que

cs bastante perünente a la consideración de Gódel en este libro. Vamos aplantearla del siguiente modo: Un lunes a la maiana, un profesor le dijoa su clase: "Tendrán una prueba sorpresa esta semana. Puede ser hoy,mañana, el miércoles, el jueves o el viernes como máximo. La mañana deldía del examen, cuando vengan a clase, no sabrán que ése es el día del exa-men".

Bueno, un esh¡diante lógico razonó de esta manera: 'Obüamente, elexamen no puede ser el último día, viernes, porque si no he rendido elexamen al finalizar la dase del jueves, entonces sabré que el viernes a lamañana es el día, y el examen no será una sorpresa. Esto descarta el viernes,por lo tanto ahora sé que el jueves es el último día posible. Y, si no rindoel examen al fnalizat el miércoles, entonces sabré el jueves a la mañanaque ése debe ser el día (porque ya he descartado el viernes); en consecuen-cia no será una sorpresa. Por lo tanto también descarto el jueves'.

Luego el alumno descartó el miércoles con el mismo argumento,después el martes, y finalmente el lunes, el día en que est¿ba hablando elprofesor. Su conclusión fue: 'Por lo tanto no habrá ningún examen; elprofesor no üene posibilidad de cumplir con su enunciadoo. En ese mo-mento, el profesor dijo: 'Ahora les tomaré el examen". El alumno quedósumamente sorprendido.

¿Cuál era el error en el razonamiento del alumno?

Se han escrito muchos arúculos sobre este famoso problema, y apa-rentemente todavia no se ha logrado un acuerdo uniforme sobre el análi-sis corecto. Mi punto de vista es el siguiente, en pocas palabras:

Voy a ponerme en el lugar del alumno. Yo afirmo que podría tener unaprueba sorpresa cualquier üa, ilncluso eluietts ! Este es mi razonamien-to: Supongamos que llega el viemes a la mañana y todavía no rendí elexamen. ¿Qué pensada entonces? Suponiendo que le crel al profesor enprimer lugar (y este supuesto es necesario para el problema), ¿podría seguircreyéndole firmemente al profesor el viernes a la mañana si todavía no ren-dí el examen? No veo cómo podría hacerlo. Indudablemente podría creerque rendiría el examen hoy (viernes), pero no podría creer que tendda unaprueba sorpresa hoy. Por lo tanto, ¿cómo podría confiar en la exactitud del

2

r8 l9

Page 12: SMULLYAN, R. Juegos por siempre misteriosos. España. Gedisa 2008

profesor? Al teraer duüs sobre el profesor, no sabría qué pensar. En lo queá mí respecta podria pasar cualquier cosa, y po¡ lo tanto bien podía serque me pudiera sotpterrrler al tener el examen el viernes.'

En realida6l, el profesor diio dos cosas: 1) Tendrán un examen estasemana, 2) No sabrán la rnañatta del examen que ése es el día. Pienso quees importante separar estos dos enunciados. Podrla ser que el primerenuniiado del profeso¡ Fuera correcto y el segundo incorrecto. El viernesala mañana, yo ao podrla creer consistentemente que ambos enunciadosdel profesor eran correctos, pero podrla cree¡ firmemente en su primeren rñciado. 5i¡ embargo, si lo hago, entonces el segundo enunciado es in-correcto (dado que entonces pensaré que rertdiré el examen hoy). Por orolado, si dudo del P¡imer enunciado del profesor, entonces no sabré sirendi¡é el exañen hoy o no, 1o cual significa que el segundo enunciadodel profesor ss ha cu¡p¡do (suponiendo que c'umple con su palabra y metoml el examen). Por lo tanto losorprendentees que el segundo enunciadodel profesor e5 verdadero o falso y depende respectivamente del hecho dequeyo no crea o sí crea su primer enunciado. Por lo tanto la única formaen que el profesor puede tener razÓn es si yo tengo dudas sobre él; si dudode é1, eso-significa que úene razón, mientras que si confÍo absolutamenteen é1, entonces está eqLrivocado. (No sé si este curioso punto ha sido toma-do en consid eraciÓ¡ ant€riormente.)

La ue¡síón de un d,ía- La complicaciÓn de tener más de un día es real-mente aiena al núcleo de1 problema, por lo tantose ha propuesto la siguien-te versién de "un día" de la paradoja El profesor le dice al alumno: "Tendráuna prueba sorpresa hoy'. ¿Qué debe interpretar el alumno?

Ún problerna eQui¡¡¿lente es el siguiente; Supongamos que un alumnole orequnta a su profss.r de teología, "¿Existe Dios realmente?" El profesorr.rpoñd", "Dios existe, pero usted nunca creerá que Dios existe." ¿Qué de-be interpretar el alurnrro? Más adelante veremos que aceptando ciertos su-p.restoi-rry razonables sobre la capacidadde razonamiento del alumno,ésté no puede creerle al profesor sin volverse inconsistente.

Un oroblerna 9ue se relaciona todavía más coqel núcleo central de estelibro es el siguiente: Ntrevamente, el alumno le pregunta al profesor deteología si Dios existe- En este caso el profesor da la siguiente respuestacurioia, "Dios existe si y sólo siusted no creeen absoluto que Dios existe."De modo equivalen¡s,' el profesor dice: "Si Dios existe, entonces ustednunca creerá que Dios existe, pero si Dios noexiste, entonces usted creeráque Dios ¡s¿lmente e)dsfe." ¿Qué debe interpretar el pobre alumno a partttde esd ¿Puede el alumno cree¡le al profesorsin volverse inconsistente? Sí;el resultado es que puede hacerlo, pero (nuevamente según ciertos supues-tos razonables sobre la capacidad de razonamiento del alumno, que seránexplicados en el curso de este libro) el alumno, que le cree al profesor, pue-de seguir siendo consistente sÓlo si no sabeqrrc es consistente. En otraspalabras, si el alumns le cree al profesor y cree en su propia consistencia,Lntonces se volverá inconsistente.

20

Esta paradoja se relaciona est¡echamente con el notable Segundo' t'eorema de la Incompletiu.rd de Gbdel el korema de la no demostrabilidadt.le la consistencia. Gódel consideró algunos de los sistemas matemátrcosrnás amplios que se conocen hasta ahora. Esos sistemas son verdadera-mente consistentes (dado que todo lo demostrable en ellos es verdadero),pero lo sorprendente es que esos sistemas, a Pesar de su poder (o, vistode otra forma, como consecuencia de su poder), son incapaces de de-mosÚar su propia consistencia. Conocemos la consistencia de esos siste-rnas sólo por medio de métodos que no puede¡formalizarse en los propiossistemas.

Este libro investigará esas paradojas y la obra de GÓdel. Para que nos.sirva de ayuda en esta búsqueda (y para divertirnos un poco) consideremosprimero algunos acertijos lógicos y su relación con la lógica proposicional.

21

Page 13: SMULLYAN, R. Juegos por siempre misteriosos. España. Gedisa 2008

Parte II

LA LOCI¿E DE MENTIRY DE DECIR LA VERDAD

Page 14: SMULLYAN, R. Juegos por siempre misteriosos. España. Gedisa 2008

El empadronador

.- Gra¡ plae de la acción en este libro tendrá lugar en la Isla de los Caba_lleros y los Bribones donde, como ya vimos, los dbaileros siempre formu-lan enunciados verdaderos, los bribones siempre forrrn¡lan enunciadosfalsos, y cada habitante es un caballero o un biibón.

un hecho fundamental en esta isla es que a todo habitante le resultaimposible declarar que es un bribón, porqu-e un caballero nunca mentiríay.dirq que es un bribón, y un bribón nunca admiü ria verazmente que esuh bribón.

.Los cuatro problemas siguientes introducen los conectores lógicos y,o, sl-entonces, y sly sólo sl, que se tratarán de modo más formal eñ el cá_pítulo 6.

LA VISITA DE McGREGoR

Una ve-,, el empadronador señor McGregor rce\zó cierto trabajo deglmpo en la Isla de los caballeros y los Bribones. En esa isla ambibn sedenomina a las mupres caballeros y bribones. En esa visita Mccregordecidió entrevis[ar solamente a los matrimonios.

1. (Y)

McGregorllamó a una puerta; el marido la abrió a mediasyle preguntóa McGregor qué deseaba.

Hago un censo -respondió McGregor-, y necesito información sobreusted y su espo_sa. ¿CuáI, si alguno lo es, es un caballero, y orá1, si algunolo es, es un bribón?

-¡Ambos somos biibonesr- dijo el marrdo enojado mientras cenabala puera de un golpe.

- ¿De qgé dase es el marido y de qué clase e¡ l¡ rn. jer? (La solución vienedespués del problema 2.)

25

Page 15: SMULLYAN, R. Juegos por siempre misteriosos. España. Gedisa 2008

2. (O)

En la casa siguiente, McGregor le preguntó al marido: -¿Ambos son, bribones?- El marido respondió: -Por lo menos uno de nosotros lo es.

¿De qué clase es cada uno?

Solucíón delprcblema 1. Si el marido fuera un caballero jamás hubie-ra afirmado que él y la mujer eran bribones. Por lo tanto debe de ser unbribón. Dado que es un bribón su enunciado es falso; por lo tanto ambosno son bribones" Esto significa que la esposa debe ser un caballero. Por lotanto él es un bribón y ella es un caballero.

Solucíón delprcblema 2. Si el esposo fi¡era un bribón, enronces seríaverdad que por Io menos uno de los dos es un bribón, en consecuenciaun bribón hubiera formulado un enunciado verdadero, lo cual es imposi-ble. Por lo tanto el esposo debe ser un caballero. Entonces se sigue quesu enunciado era verdadero, lo que significa que él o su esposa es un bri-bón. Dado que él no es un bribón, entonces su esposa lo es. Y por lo tantola respuesta es la opuesta del Problema 1: él es un caballero y ella es unbribón.

El siguiente problema es más sorprendente que los dos anteriores (porlo menos para aquellos que no lo h¿n visto anteriormente). Conüene untema que vuelve a eparecer en algunos de los problemas más complejosque surgirán en capítulos posteriores.

3. 6i- entonces)

La siguiente casa que visitó McGregor resultó un mayor enigma. Unhombre algo introvertido abrió la puerta tímidamente. Cuando McGregorle pidió que dijera algo sobre sí mismo y sobre su esposa, lo único que dijoel esposo fue: -Si soy un caballero, entonces también lo es mi esposa.

McGregor se fire no muy complacido. -¿Cómo puedo deducir algosobre alguno de los dos a partir de una respuesta tan evasiva?- pensó.Estaba a punto de escribir, "Marido y Mujer ambos desconocidos", cuandorecordó súbitamente una vieja lección de lógica de sus días de estudianteuniversitario. Por supuesto -se dio cuenta-, puedo determinar de quéclase son attbos.

¿De qué clase es el marido y de qué clase es la mujer?

Solución Supongamos que el esposo es un caballero, entonces lo quedijo es verdad; es decir, que si es un caballero, también lo es su esposa y,en consecuencia, la esposa también debe ser un caballero. Esto demues-tra que sl el esposo es un caballero, también lo es la esposa. Bueno, esoes exactamente lo que dijo el esposo; dijo que si él es un caballero, enton-ces también lo es su esposa. Por lo tanto formuló un enunciado verdadero,y en conseoencia debe ser un caballero. Ahora sabemos que es un

26

F

, aballero, y ya hemos demostrado que si es un caballero, ambién lo es sucsposa. Por lo tanto marido y mujer deben ser caballeros.

I.aidee impllcita en el úlümo problema tiene ramificaciones más t¡as-r r:ndentes de lo que el lector podría advertir. Consideremos la siguiente va-r r a nte del problema: Supongamos que visitamos la isla en busca de oro. An-l(:s de empezat e e;r.cava:r, queremos descubrir si realmente hay oro en larsla. Hay que hacer la hipótesis de que cada nativo sabe si hay o no hayr ¡ro enla isla. Supongamos que un nativo nos dice: "Si soy un caballero, en-tonces hay oro en la isla". Entonces podemos fustific¿damente sacar la con-, lusión de que el nativo debe ser un caballero y que debe haber oro en larsla. El razonamiento es el mismo que el de la solución del problema 3;supongamos que el naüvo es un caballero; así es realmente cierto que sir's un caballero, hay oro en la isla yen consecuencia hay oro en la isla. Estotlcmuestra que si es un caballero, entonces hay oro en la isla. Dado quertijo eso precisamente, es un caballero. Por consiguiente, hay oro en la isla.

La solución del problema 3 y su variante son cílsos especiales delsiguiente hecho, que es suficientemente importante para regishado como'lcorema I.

Teorerna I Dada una proposición p, supongamos que un nativo de lai.sla de los caballeros y los bribones dice: "Si soy un caballero, entonces p."lintonces el nativo debe ser un caballero y p debe ser verdadera.

La solución del problema 3 es un caso especial del Teorema I, consi-derando que p es la proposición de que la esposa del nativo es un caballe-ro. La variante del problema I (el problema sobre el oro) también es uncaso especial del Teorema I, considerando que p es la proposición de quehay oro en la isla.

También se sigue a parür del Teorema I que ningún habitante de la islade los caballeros y los bribones puede decir: "Si soy un caballero, enton-ces Santa Claus existe" (a menos que, por supuesto, Santa Claus realmentesí exista).

4. (Si y sólo si)

Cuando el empadronador entrevistó ala anarte pareja, el esposo dijo:-Mi esposa y yo somos de la misma clase; o ambos somos caballeros oambos bribones.

(El esposo podria haber dicho alternativamente: -Soy un caballero siy sólo si mi esposa es un caballero. Es Io mismo.)

¿Qué puede deducirse sobre el marido y qué puede deducirse sobrela mu.ier?

Soluciott" No puede determinarse si el esposo es un caballero o un bri-bón, perosepuededeterminarde qué clasees la esposa, delsiguiente modo:

Si la esposa fuera un bribón, el esposo jamás podría afirmar que es de

27

Page 16: SMULLYAN, R. Juegos por siempre misteriosos. España. Gedisa 2008

la misma clase que su esposa, porque eso sería equivalente a declarar quees un bribón, lo cual no puede hacer.

Una forma alternativa de considerar el problema es ésta: El esposo esun caballero o un bribón. Si es un caballero, su enunciado es verdadero,en consecuencia é1 y su esposa realmente son de la misma clase, lo cualsignifica que la esposa también es un caballero. Por otro lado, si él es unbribón, entonces su enunciado es falso, en consecuencia él y su esposa sonde distinta clase, lo cual significa que la espeiSa, al contrario del esposo, esun caballero. Y por lo tanto, independientemente de que el esposo sea uncaballero o un bribón, la esposa debe de ser un caballero. (En este caso,la clase de esposo es 'indeterminada"; podria ser un caballero que afirmÓverazmente ser como la esposa, o podría ser un bribón, que afirmó falsa-mente ser como la esposa.)

Este problema es un caso especial de lo siguiente: Dada unaproposición p, supongamos que un habitante de la isla dice: 'Soy un caba-llero si y sólo si p es verdadera". pué puede deducirse?

Dos proposiciones se denominan equiualentes si ambas son verdade-ras o falsas: en otras palabras, si una de ellas es verdadera, también lo se-rá la ova. Dos proposiciones se denominan no equlual.entet si no sonequivalentes: enotraspalabras, siuna de ellas esverdaderaylaotra es falsa.Ahora, el habitante ha dicho: "Soy un caballero si y sólo si p es verdadera "Si suponemos que k es la proposición de que el habitante es un caballe-ro, entonces el habitante afirma que k es equivalente a p. Si es un cab¿-llero, entonces su afirmación es verdadera, en consecuencia k es realmenteequivalente a p; y dado que k es verdadera (él es un caballero), entoncesp también lo es. Por otro lado, si es un bribón, entonces su afirmaciÓn esfalsa; k no equivale realmente a p, pero dado que k es falsa (no es un ca-ballero), entonces nuevamente p debe de ser verdadera (porque todaproposición no equivalente a una proposición falsa es obviamente verda-clera). Y por lo tanto observamos que p debe ser verdadera, pero k esindeterminada. Vamos a registrar este caso como Teorema II.

Teotema ILDada una proposición p, supongamos que un habitantedice: 'Soy un caballero si y sólo si p". Entonces p debe ser verdadera,independientemente de que el habitante sea un caballero o un bribón.

Regresemos al problema de determinar si hay oro en la isla. Suponga-mos que un naüvo dice: "Soy un caballero si y sólo si hay oro en la isla."Entonces segfin el Teorema II (considerando que p es la proposición deque hay oro en la isla), vemos que debe haber oro en la isla, aunque nopodemos determinar si.el nativo es un caballero o un bribón.

Por lo tanfo observamos que sl un nativo dice: "Si soy un caballero,entonces hay oroen la isla", según el Teorema I, podemos inferir tanto quees urr caballero colno que hay oro en la isla. Pero si en cambio dice: .Soy

un caballero si y sólo si hay oro en la isla", entonces según el Teorema II,

28

lo único que podemos inferir es que hay oro; no podemos detennirr,ri E,'cl hablante es un caballero o un bribón.

El Teorema II constituye la base de un famoso acertijo Que invt'rrlft.' Iiilósofo Nelson Goodman. El acertijo puede plantearse del siguientc nrot l' 'Supongamos que vamos a la Isla de los Caballeros y los Bribones y {¡ttt:tt=mos descubrir si hay o no hay oro en la isla. Nos encontramos con u¡ ¡itl I v ' ry sólo se nos permite hacerle una pregunta, que debe responderse por.

-io por no. ¿Qué preguntaleharian?Una pregunta que sirve es: "¿Es cierto que usted es un caballero sr Y

sólo si hay oro en la isla?" Si la respuesta es sí, entonces según 6l Jsercrrr.ttI hay oro en la isla. Si responde no, entonces no hay oro en la isla (por9t t t'ól niega que el hecho de ser un caballero sea equivalente al hecho ie Qrtt'haya oro en la isla), 1o cual es equivalente a aftrmar que el heqho de s('run caballero es equivalente al hecho de que nohaya oro en la isla; por tt,tanto nuevamente según el Teorema II no hay oro en la isla.

ALGUNOS PROBLEMAS RELACIONADOS

5

¿Qué enunciado podría formular un nativo a partir del cual pudiéramosdeducir que si es un caballero, entonces hay oro en la isla, pero si es unbribón, entonces podría o no haber oro en la isla?

6

¿Qué enunciado podría formular un nativo para que pudiérarnosdeducir que si hay oro en la isla, entonces debe ser un caballero, pero sino hay oro en la isla, entonces podría ser un caballero o {¡n bribón?

7

Una vez visité esa isla y le pregunté a un naüvo: "¿l lay oro en e5¡¿ isla?'Lo único que dijo como respuest¿ fue: "Jamás he dich<¡ que haya oro enesta isla". Más tarde, descubrí que había oro en la isla. Ill nativo ¿era uncaballero o un bribón?

SOLUCIONES

5. Hay muchos enunciados que pueden servlr. I lr¡¡ r rlc ellos ¿9; "5' 'yun caballero y hay oro en la isla". Otio es, nHay oirr ert le lsla ! hay trl'rt¡en la isla." (Si el nativo es un caballero, enton('eli lr¡r €ul)r.rc¡ito hay t't"

:B

--_...-

Page 17: SMULLYAN, R. Juegos por siempre misteriosos. España. Gedisa 2008

-así como plata- pero si hay oro, el nativo no tiene por qué ser uncaballero, podría no haber nada de plata.)

6. Un enunciado que sirve es: 'O soy un caballero o hay oro en la isla".La frase " o-d slgnlfrca por lo ¡nenos uno y posiblemente arnbos. Y por 1otanto si hay oro en la isla, entonces indudablemente es cierto que o el nati-vo es un caballero o hay oro en la isla. Por lo tanto, si hay oro en la isla,entonces el enunciado del nativo e¡a verdadero, lo que a su vez impücaque el nativo debe ser un caballero. Esto demuestra que si hay oro en laisla, entonces el nativo es un caballero.

Por otro lado, el nativo podría ser un caballero sin que hubiera oro enla isla, porque si es un caballero, entonces es verdad que o es un caballeroo hay oro en la isla.

Otro enunciado que sirve es: !'O hay oro en la isla o hay plata en la isla".

7. Supongamos que el nativo era un bribÓn. Entonces su enunciado esfalso, lo cual significa que una vez sí afirmÓ que hay oro en la isla. Sudeclaración debe haber sido falsa (dado que es un bribón), 1o que signifi-ca que no hay oro en la isla. Pero yo les dije que habíaoto en la isla. Enconsecuencia no puede ser un bribón; debe ser un caballero.

/+

En busca de Oona

Existe todo un grupo de islas de caballeros y bribones en el Pacíficosur en les cuales algunos de los habitantes son mitad humanos y mitadpájtos. Estas personas-pljaro vuelan tan bien como los pájaros y hablancon tanta fluidez como los humanos.

Esta es la historia de un filósofo ---tn lógico, en redidad- que visitóese grupo de islas y se enamoró de una joven-pájaro llamada Oona. Secasaron. Su matrimonio era feliz, excepto que su esposa era demasiadoescu¡ridiza. Por ejemplo, él llegaba al hogar a la hora de la cena, pero siera una tardecita especialmente agradable, Oona se habfa ido volando aotra isla. Por lo tanto él terúa que remar por los alrededores en su cÍrnoade una isla a la otra hasta encontrar a Oona y llevada e c:.sa. Siempre queOona aterizaba en una isla, todos los naüvos la veían en el aire y sablanqueest¿ba aterrizendo. Sinembargo, unavez que descendla, era muy dificilencontrada, por lo tanto lo primero que hacía el marido cuando llegaba auna isla eraúe¡:,r de averiguar por medio de los naüvos en dónde habíaatenizado Oona. Lo que dificultaba tanto la tarea, por supuesto, era lJuealgunos de los naüvos eran bribones y no decían la verdad. Los siguientesson algunos de los incidentes que le ocurieron.

1

En una opornrnidad, el esposo llegó a una isla en busca de Oona yencontró a dos nativos, A y B. Ies preguntó si Oona habia a¡enizado enla isla. Obn¡vo las sigrientes respuestas:

A: Si B y yo somos caballeros, entonces Oona está en esta isla.B: Si A y yo somos caballeros, entonces Oona está en esta isla.

¿Está Oona en esta isla?

2

En otro opornrnidad, dos nativos A y B formularon los siguientesenunciados:

A: Si alguno de los dos es un caballero, entonces Oona está en esta isla.

30 3l

Page 18: SMULLYAN, R. Juegos por siempre misteriosos. España. Gedisa 2008

It I:s() cs verdad.

¿listá Oona en esta isla?

3No recuerdo los detalles del siguiente incidente con mucha daridad.

Sé que el lógico se encontró con dos naüvos A y B y que A dijo: uB es uncabá[ero, y Oona está en esta isla". Pero no r.*erdo &actamente lo quedijo B..O dijo; "A es un bribón, y Oona no está en esta isla'. O dijo: "Aesun bribón y Oon está en esta isla. ¡Ojalá pudiera recordarlo! De todos mo-dos lo que recuerdo es que el lógico pudo determinar si Oona estaba o noen la isla. ¿Esaba?

4

En el siguiente incidente, el lógico llegó a una isla muy pequeña consólo seis habitanúes. Interrogó a cada uno de ellos y, bastante curiosamente,todos dijeron lo mismo: "Por lo menos un bribón en esta isla ha visto a Oonaaterrizar aquí esta tarde.'

¿Algún nativo de esa isla vio a Oona ^terrizar

allí esa tarde?

5

En otro extraño incidente, cuando el esposo llegó a una isla buscandoa Oona, se encontró con cinco nativos, A, B, C, D, y E, quienes adiünaronsu propósito y le sonrieron con picardía al encontrarlo. Formularon lossiguientes enunciados:

A: Oona está en esta isla.B: Oona no está, en esta isla.C: Oona estuvo aqtÍ ayer.D: Oona no está aquí hoy, y no estuvo aquí ayer.E: O D es un bribón o C es un caballero.

El Iógico lo meditó durante un rato, pero no pudo lograr nada.-¿ Podría alguno de ustedes formular otro enunciado, por favor ?,

-suplicó el lógico. En ese momento A diio: --O E es un bribón o C es uncaballero.

¿Está Oona en esta isla?

SOLUCIONES

1. Voy a conformarme con soluciones más breves que las que di ante-riormente.

32

Supongamos que A y B son ambos caballeros. Entonces el enunciado

comúnque formulan es verdadero, a partir del cual se sigue que oona está

en la islá. por lo tanto si ambos son caballeros, entonces Oona está en la

isla. Esto es 1o que dijeron, por lo tanto ambos son caballefos. En conse-

cuencia Oona está en la isla.

2. Siuno de los dos es r¡n <::tl>;tll<:ro, entoncesel enunciado que formulÓ

es verdadero, a parir de lo <:r¡:rl st' sigttr: que Oona está en la isla' Por lo

tanto, si alguno de los dos (:s r¡rr r':rlr.¡ltt'ro, ()ona está en la isla. De este

mOdO el enUnCiadO qUe amlros lr ¡¡ t r rt t l.t r, 'r r l. s v<'r<laclcro, en COnSeCUenCia

ambos son cabal leros, pc)r l ( ) l l l t l { ) : , ( .F,ur,rr¡ ( ' l l l l t l l t ( ) Jrr l r lo menos es un

Cabal lerg" A part i r de eStO y Cl<: l : l v t ' t .1, r r l . ¡ ,1 , l ¡ I r ' ! r r ¡ r r r r . r ( l ( ) r ¡ r r t : formUlarOn,

se sigue que Oona está en la i.sl:r'

3.Esteesunejemplodel<l<¡rr<. t | r . t l l l t l t l l t r | | ,1| I , t t | | t t l t i , l I Jr , ' ' r . l t t f r t f .

maloquedi joB, peroencamll i< lst ' i l t l r l t t t l . ¡ l l | . f . t l i . t i I l | .1. . | . , , ¡ , r , . I t1r ' t l r t t

Primero les demostraré que si l) l¡ttlrtcr '¡ r l¡¡ lt' ' " '1 '

no está en esta isla", entonces cl l tr¡4ir {) n{r lr" ' l rrr

oroblema. Por lo tanto supongam()s t¡trc I l lo l t tr l r lg ¡ t

podría ser un caballero, porquc si l<¡ f i ¡ t ' r ' r , ' ' ¡ t l r . ' rr ' i ' :

tao-o di jo A), lo cual convert i r ía A A <' t t t r t r l ¡ I i l . r ¡ ' ¡ t { ' ¡ r t l " . l

arrraábr["r 'yOOnanOestéen la i .s l : r , . r ¡ r rc. l l . ¡ ( l t * l , r ¡ l ¡ t=rr ! i t r t ] . r¿ =-: '=

en la is la, y no hay forma de detcr l r l i t t ; t t t t t i r l rL ' ¡ ' t ' ' ' l ' r : ¡ ' r '= i l ' i l i i l+ ' l -= -=

la correcá. por lo tanto s i B hubiera dic: l to < 'sr¡ , t ' l l r ' ¡ r . t , r , t r ¡ ' I ' r , , l r i ¡ l t¿ l ' * '+

SabidOSi OOnaestabaenla iSla. PerOSe nt¡s t l t<t ' r ¡ t tc e ' l l r ' r ¡ : . l r Ú: l l ¡ ' +¡ l ¡ i¿

Lrr aonra*"t tc ia B no di fo eso. Tiene que halrcr t ¡< l t t ¡ "A r ' : t t r r I ' ,1" ' r ¡ r '

Oona está en esta isla". Ahora veamos lo que <>cut rt'

A debe ser un bribón por la misma rLz6A anleri()r. si ( )r ¡¡t.¡ ¡"¡l ;t ¡= ¡ I l'r

isla, obtenemoslasiguienté contradicción: Esentoncesvcrtl:tt l t¡ttt 'A r : rttt

UriÉOn y Oona estáln la isla, en consecr¡e-ncia B formuló u. ('trtrrr( lr¡l' '

""r¿"¿aro, lo cual convierte a B en un caballero. Pero entoncc.s A f ¡ ¡t rttttl¡'

un enunci;do verdadero al declatat que B es un caballero y Oorta csl:t crr

ü itf", opuesto al hecho de que e él r.'r1 bribón' La única salida tlt: l¡

ó"t "¿üiO"

es que Oona zoésté en la isla. Por lo tanto Oona no cstá ('tr

la isla (y, por supuesto, A y B son bribones)'

4. Dado que los seis nativos dijeron lo mismo, entonces o son todos

caballeros o t;dos bribones (todos caballeros, si lo que dijeron es verda-

á"-, v todos bribones de lo contrario)' Supongamos.gY" to,1 t*?t-ii:

balleros. Entonces sería verdad quc por lo menos un bribÓn en la $ta naDra

visto aterrizar a Oona, pero eso s-eríi imposible, dado que ninguno de ellos

es bribón. por lo tantotodos deben .ci l>.ibones. En consecuencia lo que

ó.ro" "r falso, lo que significa quc ni.gírn l>ribon de la isla vio aterrizar

33

Page 19: SMULLYAN, R. Juegos por siempre misteriosos. España. Gedisa 2008

a Oona esa tarde. Pero dado que todos los nativos son bribones, entoncesningún nativo vio aterrizat a Oona esa tarde.

5. Vamos a demostrar que si A es un bribón obtenemos una contra-dicción: supongamos que A es un bribón. Entonces zu segundo enunciadoera falso, en cor$ecuencia E debe de ser un caballero y C debe de ser unbribón.

Dado que E es un caballero, su enunciado es verdadero, y, entonces,o D es un bribón o C es un caballero. pero C no es un caballero, por lo tantoD debe de ser un bribón. De donde el enunciado de D era falso y, por lotanto, Oona está aquí hoy o estuvo aquí ayer. pero Oona no eshrvo aquíayer (porque C dijo que estuvo y C es un bribón), asl que ella está aqul hóy.Pero esto convierte el primer enunciado de A en verdadero, contrario alsupuesto de que A es un bribón. Entonces A no puede ser un bribón; debeser un caballero. Por lo tanto el primer enunciado de A era verdadero, yOona está en esta isla.

Un i,aberinto interplaneta'rio

En Garúmedes -un satélite de Júpiter- hay un club conocido comocl club marciano-venusino. Todos los socios son o de Marte o de Venus,aunque a veces se admiten visitantes. Un terrlcola es incapaz de distinguirpor li apariencia a los marcianos de los venusinos. Además, los terrícolasno pueáen distinguir a los varones marcianos o venusinos de las mujeres,dado que se visten iguales. Sin embargo, los lÓgicos üenen una ventaja,dado {ue las mujeres venusinas siempre dicen la verdad y los varonesvenusiños Siempre mienten. Los marcianos son lo opuesto; los varones mar-cianos dicen la verdad y las mujeres marcianas siempre mienten.

Un día Oona y su esposo lÓgico visitaron Ganlmedes y les comentaronsobre ese club: -Te apuesto a que pttado distinguir a los varones d,e lasmujeres y a los marcianos de los venusinos -le dijo el esposo orgullosa-mente a su esposa.

-Pómo?- preguntó Oona.-iremos al club esta noche, que es la noche de los visitantes, y te lo

demostraré --dijo el esposo (cuyo nombre, a propÓsito, era George)'

1

Esa noche visitaron el club. -Ahora, veamos qué puedes hacer -diioOona con un dejo de escepticismo-. El socio de allá. ¿Puedes descubrirsi él o ella es varón o muier? -Entonces George se acercó al socio y le hizouna sola pregunta por sl o no. El socio respondió, y entonces George de-terminó si el socio era varón o mujer, aunque no pudo determinar si el socioera de Marte o de Venus.

¿Qué pregunta Pudo haber sido?

2

-Muy inteligente! -dijo Oona, cuando George le explicÓ la solu-ción-. Ahora supongamos que en vez de querer descubrir si el socio eravarón o mujer, hubieras querido descubrir si era de Marte o de Venus.

¿Habrías podido hacedo formulando solamente una pregunla por sí o no?-¡Por supuesto! -dijo George-. ¿No imaginas cÓmo?Oona lo meditó un rato y repenünamente se dio c\¡enta. ¿CÓmo?

5

34 35

Page 20: SMULLYAN, R. Juegos por siempre misteriosos. España. Gedisa 2008

3-Si eres tealmente inteligente -dijo Oona-, debes rer capaz de

descr¡brir sólo con una pregunta si un socio dado es varón o mujer y dedónde proviene. A ver si haces ambas cosas de inmediato usando solamenteuna pregunta por sl o no.

-¡Nadie es tan inteligente! -respondió George. ¿Por qué dijo esoGeorge? CEsencialmente esto es una repeüción del último acertijo delcapltulo 2 de mi hbro To Mock a Uocktnpbnd.)

4

En ese preciso momento un socio pasó al lado de ellos y formuló unenunciado a partir del cual George y Oona (que en ese momento ya em-pzaba a entender las cosas) pudieron deducir que el socio debía ser unamujer marciana. ¿Qué enunciado pudo haber sido?

5

El siguiente socio que pasó cerca de ellos formuló un enunciado a par-tir del cual George y Oona pudieron deducir que el socio debía ser unamujer venusina. ¿Qué enunciado pudo haber sido?

6

¿Qué enunciado podría formular un varón marciano, una mujer mar-ciana, un varón venusino o una mujer venusina?

Las noticias sobre el ingenio con que George y Oona aplicabanlalógicapara determinar el sexo y/o raza de varios socios se difundieron rápida-mente por todo el club. El propietario del club, un Empresario norteameri-cano llamado Fetter, se acercó a la mesa de George yOona para felicitarlos.-Me gustaría ponedos a prueba con otros socios más -dijo Fetter-, yver qué pueden hacer.

7

En ese momento pasaron dos socios. -Acérquense -dijo Fetter, quelos presentó como Ork y Bog. George les pidió que le dijeran algo sobreellos mismos, y los dos formularon los siguientes enunciados.

ORK: Bog es de Venus.BOG: Ork es de Marte.ORK: Bog es varón.

36

BOG: Ork es muier.Con esta información, George y Oona pudieron identificar sucesiva-

mente tanto el sexo como la reza de cada uno de ellos. ¿Qué es Ork y quées Bog?

I

-Saben -dijp Fetter, cuando ya Ork y Bog se habían ido-, los mar-cianos y venusinos generalmente se casan entre sí, y tenemos varias pare-jas mixtas en este club. Aquí se acerca una pareja. Veamos si pueden descr.i-brir si es una pareja mixta o no.

No recuerdo los nombres de pila dela pareja, por lo tanto simplementelosl lamaréAyB.

-¿De dónde son? --Oona le preguntó a A.-De Marte -fue la respuesta.-¡Eso no es cierto! --dijo B.¿Es ésta una pareja mixta o no?

9

-Aquí viene otra pareja -4ijo Fetter. Nuevamente no voy a decirlessi es una pareja mixta o no. Veamos si pueden descubrir cuál es el esposo.

Llamaremos a los dos A y B. George preguntó: -¿Ambos son del mismoplaneta? Estas son las respuestas:

A: Ambos somos de Venus.B: ¡Eso no es cierto!

¿Cuál es el esposo?

to

-Aquí hay otra pareja --dijo ltrltcr. Nuevamente, no voy a decirlessi es una pareja mixta o no. Vearrl()s lo <¡ue sucede.

En este caso recuerdo los nonll¡rt:s: Jal y Tork.-¿De dónde son cada uno de ustt'tk:s? -preguntó George.-Mi cónyuge es de Marte -res¡rontlió Tork.-Ambos somos de Marte -difo.|alEsto les permiüó a George y Oonl r'l¡sifi<'arlos completamente. ¿Cuál

es el marido y cuál es la mujer, y de t¡rrtq ¡rlrutcta proviene cada uno?

solu(l loNlts

1. Lapregunta mássencilla quesllerF: "¡l1e trntnl rnarciano?" Suponga-mos que la respuesta es sí. O el habla¡tte rli¡ r= l¿ vcrtlad o miente. Si es el

37

Page 21: SMULLYAN, R. Juegos por siempre misteriosos. España. Gedisa 2008

l)rimer caso, entonces el hablante es realmente marciano, y al ser unrnarciano veraz, debe ser varón. Si el hablante miente, entonces el hablan-l(: rcalmcnlc cs venusino, en consecuencia es un venusino mentiroso, porlo lllrto tanrl>ión es varón. En consecuencia en ambos casos, una respues-tl sí irrclica <¡ue cl hablante es varón. Un análisis similar (que dejo a car-¡1o rlcl lcctor) demuestra que una respuesta negaüva indica que el hablantet 's rr t r r jcr .

I)<lr supuesto la pregunta "¿Es usted venusino?" resulta igualmente satis-firctoria; una respuesta afirmativa indica entonces que el hablante es mujer,y una respuesta negativa indica que es varón.

2. La pregunta "¿Es usted varón?" sirve. (Dejo la verificación a cargo dellcctor.) Alternativamente la pregunta, '¿Es usted mujer?" también sirve.

3. La razón por la cual es imposible diseñar una pregunta por sí o noque determine definidamente si un cierto socio es varón o mujery si el socioes marciano o venusino es que hay cuatro posibilidades de socios -va-rón marciano, mujer marciana, varón venusino, mujer venusina- pero só-lo hay dos respuestas posibles parala pregunta: sí o no. Y por lo tanto, só-lo con dos respuestas posibles no se puede determinar cuál de las cuatroposibilidades es válida.

4. Un sencillo enunciado que serviría es: "Soy un varón venusino."Obviamente el enunciado no puede ser verdadero, o tendríamos la con-tradicción de un varón venusino que formula un enunciado verdadero. Enconsecuencia el enunciado es falso, lo que significa que el hablante zo esun varón venusino. Dado que el enunciado es falso, entonces el hablantedebe ser una mujer marciana.

5. Este es un poco más intrincado: Un enunciado que sirve es: "Soy omujer o de Venus." (Nota: recordemos que o-o significaporlonenosunoo posiblelrrente amboq no significa exactamente uno.)

Si el enunciado es falso, entonces el hablante no es ni mujer ni de Ve-nus, en consecuencia debe ser un varón marciano. Pero un varón marcia-no no formula enunciados falsos, y por lo tanto tenemos una contradicción.Esto demuestra que el enunciado debe ser verdadero; en consecuencia elhablante debe ser o mujer o de Venus y posiblemente ambas cosas. Sinembargo, si es mujer, también debe ser venusina, y si es de Venus, tam-bién mujer, porque las mujeres veraces son venusinas y las venusinas ve-races son mujeres. Por lo tanto el hablante debe ser tanto de Venus comomuier.

Entre paréntesis, si el hablante hubiera formulado el enunciado másrotundo: 'Soy mujer y venusina', sería imposible determinar el sexo o laraza del hablante 0o único que podría inferirse es que el hablante no esun varón marciano).

38

6. Un enunciado de esa clase es: oO soy un var6n marcía o o u t la mujervenusina", o, aun más simple: 'siempre digo la verdad'. Cualqult:r men-tiroso o veraz podrTa decirlo.

7. Supongamos que Ork dijo la verdad. Entonces Bogsería tantt¡ v;trÓncomo veñusino; en consecuencia Bog debe haber menüdo. Supon¡'¡;trnos,por otro lado, que Ork minüÓ. Entonces Bog no es varón ni venusitto; cntonsecuencia Bbg debe ser una mujer marciana y por lo tanto nuevarrr('n-te Bog debe de haber mentido. Esto demuestra que independientctttr'rrtt:de que Ork haya dicho la verdad o no, Bog definiüvamente mintiÓ.

Dado que Bog mintiÓ, entonces Ork no es de Marte ni es mujer; cn t otrsecuencia Ork debe serunvarÓnvenusino. POrlo tantoOrk también n¡illli(),lo que significa que Bog debe ser una mujer marciana. Y por lo tartlr I l,rsolución es que Ork es un varÓn venusino y Bog es una mujer mart'l;ttt''(y los cuaro enunciados eran mentiras).

8. Dado que A afirmÓ ser de Marte, entonces A debe ser varÓn (t:orr¡r Ivimos en la solución del problema I), y en consecuencia B debe scr rrrtl

ier. Si A es veraz, entonces A realmente es de Marte, B mintiÓ, y al scr tttt'tmujer menürosa también es de Marte. Si A minüÓ, A realmente es dc Vtrtrt ts,B dijo la verdad, y al ser mujer también es de Venus. Por 10 tanto ésll trr I

es una pareja mixta ambos son del mismo planeta.

9. Si el enunciado de A es verdadero, entonces ambos son dc vt:trtts,en consecuencia A es de Venus y A debe ser mujer. Supongamos r¡rrc clenunciado de A es falso. Entonces por lo menos uno de ellos es dc M¡r'tr'Si A es de Marte, A debe ser mujer (dado que el enunciado de A cs tllso)Si B es de Marte, B debe ser varÓn (dado que el enunciado de l! cs vt'rdadero); en consecuencia nuevamente, A debe ser muier. Y por ltr llrrl()A es la esposa y B es el marido.

10. Supongamos queJal dijo la verdad. Entonces los dss ¡g¿l¡¡s¡t¡: .rottde Marte; en corlsecuencia Tork es de Marte y el enunciado de'lkrrk t¡rrt'afirma queJal es de Marte era verdadero. Por lo tanto nos enfrentafix)ti ( ( )rrla imposibilidad de una pareja del mismo planeta en la cual amtx¡s dit<'ttla veidad. Eso no puede ser, en consecuenciaJal debe haber menüd<1. l'orlo tanto al menos uno de ellos es de Venus.

SiJal es de Marte, entonces debe ser que Tork es de Venus. Pero ctttonces Tork dijo la verdad al afirmar que Jal es de Marte, por lo tanto J:rlno puede ser de Marte; Jal debe ser de Venus. Dado que Jal mintiÓ y t:'sde Venus, Jal debe ser varón. Además, dado que Jal no es de Marte, 'li rt kmintió. En consecuencia Tork es una mujer mentirosa, y por lo tanto (:$ (l('Marte.

En resumen, Jal es un varÓn venusino y Tork es una mujer marciart:t.

3()

Page 22: SMULLYAN, R. Juegos por siempre misteriosos. España. Gedisa 2008

Parte III

CABALLEROS, BRIBO}{ES YLOGICA PROPOSICIONAL

Page 23: SMULLYAN, R. Juegos por siempre misteriosos. España. Gedisa 2008

6

Un poco de lógica, proposicional

Los acertijos de mentirosos y veraces de los últimos tres capltulosadquieren una importancia adicional cuando se los considera en términosde la materia conocida como lóglca prcposlclonal (como veremos en elpróximo capítulo). En este capítulo examinaremos algunos de los puntosbásicos: los conectores lógicos, las tablas de verdad y las tautologías. Loslectores que ya estén familiarizados con estos conceptos pueden pasardirectamente al siguiente capínrlo (o quizá simplemente hojear estecapltulo como repaso).

LOS CONECTORES LOGICOS

La lógica proposicional, como el álgebra, tiene su propio simbolismo,que es relaüvamente fácil de aprender. En álgebra, las letras x, y, z tepÍe-sentan números no especificados; en lÓgica proposicional, usamos lasleuas p, q, r, s (a veces con subíndices) para representar proposiciones noespecificadas.

Las proposiciones pueden combinarse r,rsando los llamados conectoreslóglcos. Los principales son:

I1) - (no)2) & (v)3) v (o)4) > (si-entonces)D = (si Y sÓlo si)

Ahora vamos a explicados.

t) Negaclón. Para toda proposición p, -¡r ri¡¡rrif i<:^ lo opuesto o con-trario a p. Leemos -p como nno es cierto t¡ttr p", o, c:n forma más breve,"no p'. La proposición -p se denomina la negrlifrtr tlt: p; es verdadera sip es falsa y es falsa si p es verdadera. Podct¡¡oa tr'¡tttttir estos dos hechosen la siguiente tabla, que se denomina Tabla dc pp,(hklp n la negación.En esta tabla (como en todas las tablas que algtte,ttl, i¡riJr.(:mos la letra 'V"para representzr la uerd.ad y "F' para repreaellldr la fitlx'dad.

43

Page 24: SMULLYAN, R. Juegos por siempre misteriosos. España. Gedisa 2008

_ La primera fila de la lc,bla de verdad dice que si p tiene el valor V (esdecir, si p es verdadera), entonces -p tiene el valor F. La segunda fila diceque si p tiene el valor F, entontes -p üene el valor V. También podemosescribido del siguiente modo:

-V=F-F=V

2) conjunctón.Para proposiciones cuaresquiera p y q, la proposiciónque p y q son ambas verdaderas se escribe "p&q, (a veces ,,p^q,,). becimosque p-&q es la conjunción de p y q. Es verdadera si p y q son verdaderas,pero falsa si alguna de las dos es falsa. por lo tanto ténemos las siguientescuatro leyes de la conjunción:

V&V=VV&F=FF&V=FF&F=F

Por lo tanto la siguiente es la tabla de verdad para la coniunción:

Observamos que pvq es falsa sólo en el cararto ceso: (:tl¡lltl.r p y il á! rrlambas falsas.

4) Si-entOtrces. Para proposiciones cualesquiera P y (1, eÉ. t ll tlt tt¡t ¡ ' ,, ¡para significar que o p es falsa o p y q son ambas vertlstletel' Flr llrcspalabras, si p es verdadera, también lo es q. A veces inler¡lrFlelturs lt 'rlcomo "si p, entonces q", o 'p implica q", o uno es cierto que ¡) rea vct r lar llt.ty q sea falsa,. Llamamos a p)q el condicional de p y q. Parr el ¡ orri llr lilI¡¡ l,tenemos la siguiente tabla de verdad.

Observemos que prq es falsa sólo en la segunda fila: el t'l¡r I ('rI quep es verdadera y q es falsa. Tal vez esto requiera un poco de cx¡rlir'a<:iÓn:prq es la proposiciÓn de que no es cierto que p sea verdadcrl y r ¡ sr:a.fal-ia. ia únióa forma en que puede serfalsa es si escierto que p cs vr.rrlaclcray q es falsa.

5) Stysótbsl.Porúltimo supongamos que pEq es la proposi<'irrrr tlr: quep y q son ambas verdaderas o falsas, o, lo que es 10 mismo, quc sr :tl¡1unade las dos es verdadera, la otra también lo es. Ieemos pEq com() "l) (!s ver-dadera si y sólo si q es verdadeta' , o 'p y q son equivalentes"' (Recorclcmosque dos proposiciones se denominan equlualentes si ambas son vt:rtlade-ras o falsas.) La proposición pÉq a veces se denomina bicondlclonal dc py q. La siguiente es la tabla de verdad.

Paréntesis. Es necesario usar paréntesis para evifar la ambigüedad. Porejemplo, supongamos que escribo p&qvr. El lector no puede saber siquiero decir que p es verdadera y que q o r es verdadera, o si esto siSni-fica que p y q son ambas verdaderas o r es verdadera. Si quiero decir_lopriméro, debo escribir p&(qvr); si quiero decir lo segundo, debo escribir(p&q)w.

Tablas compuatas de aerdad.El ualor de uerdad de una proposiciónsignifica su veracidad o falsedad: que es V, si p es verdadera, y F si p es

3) Disrynción.Para proposiciones cualesquier^ p y g, sea pvq la pro-posición de que al menos una de las proposiciones p oq es verdadlera.Interpretamos pvq como 'o p o q", y posiblemente ambas. @xiste otrosenüdo de "o", es decir, (xactalnente una, pero no vamos a usar la palabra"o" en ese sentido. Si p y q resultan ser verdaderas, la proposiciónpvq seconsidera verdadera.) La proposición pvq se denominá diiyunción de p yq. La disyunción tiene la siguiente tabla de verdad:

44 45

Page 25: SMULLYAN, R. Juegos por siempre misteriosos. España. Gedisa 2008

\

fals,a. De este modo las proposiciones 2 + 2 = 4y Londres es la capital deIngftgrra, aynque son proposiciones diferentes, tienen el mismo valor deverdad: es decir, V.

Ahora consideremos dos proposiciones p y q. si conocemos el valorde verdad de p y el valor de verded de q, entonces podemos ¿"t"r-irr".los valores de verdad de p&q, pvq, p>q, y-Fq, y también el valor de verdadd" -p (""¡ como el valorde vérd;d ds-q). pói b tanto se sigue que dadacualquier combinación de p y-q (es decir, cualquier proposicién eipresableen términos e p I q-, usando los conectores lógicos), pbdemos d"L.-irr",el valor de verdad de esta combin¿ción unalez quetenemos el-valor de":ryq 9S q y q. Por ejem-plo, supongamos qrr-. e es h proposición(p=(q&p))>(-p>q). Dados lbs valores ?e ,ret¿a¿ de p y á, ód;;averiguar sucesivamente los valores de q&p, podemos-avlriguir zucesi-vamente los valores de q&p,_ p=(q&p), -p, (-prq), y por úlümo(p=(q&p))>(-p>9). Hay cuat¡o ¿istriuüciones fbsibres ,ié

"áró ¿"

""i¿"¿Pxa py e, y en cada uno de los cuatro casos podemos determinar el valorde verdad de A. Podemos hacerlo en forma sñtemática confeccionando lasiguiente abla:

Por lo tanto observamos que A es verdadera en los tres primeros casosy falsa en el cuarto.

Consideremos otro ejemplo: Supongamos que B = (p>q)>(_q)_p), yconfeccionemos una tabla de verdad pira B.

Observamos que C es fals¿ en los ocho casos; es lo corrt¡>lc:lamcnteopuesto de una tautología y un ejemplo dc lo que se denomina u¡nlracllc-c\ón. Nohay proposiciones p, q y r tal quc (p&(ppr)>(r&-p) se:a vt:rcla-clera. (PodriamoJ habedo observado sin una tabla de verdad usan<lo clsentido común. supongamos que p&(q>r) cs verdadera. Entonccs ¿<:(rrnclpodría (r&-p) ser verdadera, dado que -¡r t:s falsa)

si confeccionamos una tabla de verdad l)ara una expresiÓn dc cr¡alroincÓgnitas --digamos, p, q, r, y$- tenemo¡i r¡rrc considerardieciséis c:a.stts,y por lo tanto la tabla de verdad tendrá dic<:is{iis lilas. En general, para toclt Inúmero entero positivo n, una tabla de v<rrtlltl pafaúrra expresiÓn clc rrincógrritas debe tener 2" filas (cada vez qlr(' :rgrcgamos una incÓgnita, lacantidad de filas se duPlica).

TAUTOLOGIAS

Una proposición se llama tautologta sl ¡rr xlt'mos determinar que e$verdadera basándonoS solamente en las taltlps tlt' verdad para conectoreslógicos. Por ejemplo, supongamos que una f )r'r:'t )rt;l dice que mañana llo-,era y una segunda persona dice que ntl. lt¡lr li('amente no podemosesperar deScubrir quién üene razón usando tltr¡ l¡lrlil de verdad. Debemosesperar hasta mañana y entonces obsemar el llcir¡l)o. Pero supongamosque una tercera persona dice hoy: "O lloverá ¡¡t¡ñ¡r¡;t t¡ no lloverá". Bueno,eso es lo que yo Llamaria ¡una predicciÓn sqgllrrll Sin tener que esperarhasta mañána y hacer una observación, sattefi lt ¡¡ 1 t t tt I ura razÓz que debeestar en lo cierto. Su aseveraciÓn tiene la foflrr¡ I'v f) (en donde p es laproposición de que lloverá mañana), Y para ktd¡t pro¡nsiciÓn p, la propo-sición pv-p debe ser verdadera (como demo-rt ¡ a I á | t ( ilmente una tabla deverdad).

La definición más común de tautologh !rrlrr,rr(: el concepto de

fórmula. Fórmula significa una expresiÓn quo rF t I tr r:,1 ruye a partir de losiímbolos -, &, v, :, = y de las variables proporir lnl¡¡lcs p, q, r..., con lospaéntesis bien colocados. Las siguientes ltoll ld6 r''glas precisas paraconfeccionar las fórmulas.

Observamos que B es verdadera en los cuatro casos, y asÍ es uncjcmplo de lo que se denomina tautolagía.

También podemos confeccionar una tabla de verdad para unacr>¡nbinación de tres incógnitas proposicionales _p, q y r_ pero ahora<k'lrcrnos considerar ocho casos (porque hay cuatro d.isliibuciónes de V vli l);rr;r l) y q, y con cada una de estas cuatro distribuciones hay doá¡rrrsrlrili<lades para r). por ejemplo, supongamos que C es la expiesión{¡rt(t¡ rr)):(r&-p). Tendría la siguiente tabla de verdad:

FFFFVV

FrFFvFvF

FFFI;l i

l ¡I rl l

vFVFvFVF

vFvVvFVv

vFvvFFFF

FFFF

VvFFvvFF

47

Page 26: SMULLYAN, R. Juegos por siempre misteriosos. España. Gedisa 2008

1) Cualquier variable proposicional única es una fórmula.2) Dadas fórmulas cualesquiera X e y ya construidas, las expresiones

(X&D, CKvlD, QblO, o G=Y) son también fórmulas, y rambién ló es la ex-presión -X.

Debe comprenderse que ninguna expresión constituye una fór¡nula amenos que se construya según las reglas 1) y 2) ya citadas.

. Cuando se presenta una única fórmula, podemos eliminar los parén_tesis externos sin incurrir en ambigüedad: por ejemplo, cuando décimos"la fórmu,la p>q", significa "la fórmula <prq),'.

Una fórmula en sl misma no es ni verdadera ni falsa, sino que sólo seconvierte en verdadera o falsa cuando lntetpretarnosque las variables pro-posicionales representanpro_posiciones definidas. poi ejemplo, si yo pre_guntara: ¿Es verdadera_la fórmula (p&q)1, usredes protráUtementé (fcorrectamente) responderían:."Depende de qué próposiciones repre_sentanlas letras upo y oq": Y por lo tanto una fórmüla al cómo ,,p&q', a rráceses verdadera y a veces falsa. por otro lado, una fórmula tal cbmó "pv-p"slefnpre es verdadera (es verdadera sea cual sea la proposición qüe rL_presente la letra "p) y en conformidad se denomina iórmula tautoTóplca.Por Io tanto una fórmula tautológica es por definición una fórmulique11"-pf es.verdader¿, o lo que es lo mismo, una tabla de verdad parálafórmula sólo tendrá símbolos V en la última columna. Entonces pod..rrosrfecir que una prcposíclón es una tautología si se expresa po,

-edio d.

cierta fórmula tautológica según determináda interpréhció; de las varia-bles proposicionales. @or ejemplo, la proposición de que llueve o no llue_ve se expresa por medio- de la fórmula pv:p, si interpretamos que ,,p,, esla proposicién que está lloviendo.)

. .Impllcaclón y equtualencla lóglcas. Dadas dos proposiciones X e y,aeci.mg¡ que X implica lóglcarnente y, o que y es uña éorsecuencta lógr-ca dex si )t¡Y es una tautologia. Decimos que X es tagtcamente eqirual.ente a Y si )GY es una taurología; o, lo qué es lo misáo, si X impiicalógicamente Y e Y implica lógicamente X.

2) (p&(p>q)):q

Esto significa que si p es verdadera, y si p implica q, entonces q esverdadera. A veces esto se parafraseat 'Todo lo que implica una pro-¡'rosición verdadera es verdadero".

3) ((p>q)&-q)>-p

Esto significa que si p implica una proposiciónfalsa, enronces p debeser falsa.

a) ((p>q)&(p>-q))>-p

Esto significa que si p implica q y también p implica no q, entoncesp debe ser falsa.

5) (-p>q)&(-pr-q))rp

Este principio se conoce como reducclón al absurdo. Para demostrarque p es verdadera, basta demostrar que -p implica alguna proposición qasí como su negación -q.

6) ((pvq)&-p)>q

Este es un principio conocido de la lógica: Si por lo menos una de lasproposiciones p o q es verdadera, y si p es falsa, entonces debe ser q laverdadera.

7) ((pvq)&((p>r)&(q>r))¡r

Este es otro principio familiar conocido con¡<> dernostraclónporcasos.Supongamos que pvq es verdadera. Supon¡¡amos también que p

implica r y que q implica r. Entonces r debc scr vcrdadera (indepen-dientemente de que p o q sean verdaderas, o anlllas cosas a la vez).

El lector que no tenga mucha experiencia con la ló¡¡ica proposicionalpuede sacar provecho del siguiente ejercicio.

EJerclcto.l. Determinar cuáles de las siguientcri ri(,n ltrrrlolo¡¡ías,

a) (p>q)>(q>p)b) (p>q)=(-p>-q)c) (p>q):(-q>-p)d) (f¡=q)>(-p=-q)e) -(p=-p)f) -(r-p)8) -(P&q):(-P&-q)

ALGUNAS TAUTOLOGIAS

La tabla de verded es un método sistemáüco de verificar tautologías,pero muchas tautologías pueden reconocerse con mayor rapidezsi sJusaun poco de senüdo común. Estos son algunos ejemplos:

'

1 ) ((p>q)&(q>r))>(p>r)

-. Blo liS-nr-nÍa que si p implica q, y si q implica r, enronces p implica

r' I'lsto indudablemente resulta evidente poisí mismo (pero, por iupuésto,' pucdc verificarse con una tabla de verdad). Esta táutoiógia tiéne unnrrrrrbre: se denomina sllogísmo.

48 4t)

Page 27: SMULLYAN, R. Juegos por siempre misteriosos. España. Gedisa 2008

-(pvq)>(-pv-q)(-pv-q)>-(pvq)-(p&q)=Gpv-q)-(pvqF(-p&-q)(q=r)>(p>q)=(p>r))(p=(p&q))=(q=(pvq¡¡

Respuestcrs. a) No, b) No, c) Sl, d) Sí, e) ¡No!, g Sl, g) No, h) Sí, i) No,D Si, t¡ Sí, l) Sí, m) Sl (tanto p=(p&q) como (q(pvq)) son equivalantes aprq).

Con respecto a e), muchos principiantes piensan que ningunaproposición p puede implicar su negación. ¡Esto no es así! Si p es falsa,entonces -p es verdadera, en consecuencia en ese caso pl-p esverdaderaSin embargo, ninguna proposición puede ser equivalente a su propianegación, y por lo tanto D es en realidad una tautología.

Análtsls. La importancia de las tautologías radica en que no sólo sonverdaderas, sino que son lóglcarnmteclertas. No se necesita ningún expe-rimento científico para establecer su veracidad: pueden verificarse sobrela base de la pura razón.

Alternativamente se pueden caracterizar las tautologías sin recurrir alconcepto de fórmulas. Definamos un estada de cosas como alguna cla-sificación de todas las proposiciones en dos categorías: proposicio-nesuerdaderas y proposicioÁes fakas, con la restricóión de-qué ta cla-sificación debe saüsfacer las condiciones de laTabla de verdad para losconectores lógicos (por ejemplo, no podemos clasificar pvq como verda-dera si p y q se clasifican ambas como falsas). Una uutología, por lo tanto,es una proposición que es verdadera en toda estado de cosas poslble.

Esto se relaciona con el concepto de I¿ibniz sobre ot¡os mundosposibles. Leibniz sostenía que de todos los mundos posibles, éste era elmejor. Francamente, no tengo la menor idea si tenía l'azón o estaba equi-vocado, pero lo interesante es que tuvo en cuenta otros mundos posibles.A partir de esto en los últimos años ha sido desarrollada toda una rama delógica filosófica -principalmente por el filósofo Saul Kripke- conocidacomo semántica de los mundos posibles que analizaremos en uno de losúltimos capínrlos. Dado cualquier mundo posible, el conjunto de todas lasproposiciones que son verdaderas para ese mundo, junto con el conjuntode todas las proposiciones que son falsas para ese mundo, constituyen elestado de cosas que rige en ese mundo. Por lo tanto, una tautología esverdadera, no sólo para ese mundo, sino para todoslos mundos posibles.l,as ciencias flrsicas se interesan en el estado de cosas que vale para elrrrundo real, mienfras que la matemática y la lógica puras estudian todosIr¡.s cstados de cosas posibles.

Caballeros, briboncs y lógicaproposicional

RETORNO A LOS CABALLEROS Y BIUBONES

Ahora podemos introducir un método de traducciÓn simple pero fundamental por medio del cual un sinnúmero de problemas sobre mentiro'sos y veraces puede reducirse a problemas de lógica proposicional. Estcrmétodo será decisivo en varios de los capítulos que siguen'

Regresemos a la Isla de los Caballeros y los Bribones. Dado un nativoP, supongamos que k es la proposiciÓn de que P es un caballero. Ahorasupongamos que P afirma una proposiciÓn X. En general no sabemos siP és un caballero o un bribón, ni tampoco si X es verdadera o falsa. Perosí sabemos todo esto: Si P es un caballero, entonces X es verdadera,y alainversa, si X es verdadera, entonces P es trn caballero (porque los bribonesnunca formulan enunciados verdaderos). Y por 1o tanto sabemos que P esun caballero si y sólo si X es verdadera; en otras palabras, sabemos que laproposición leX es una proposiciÓn verdadera. Y por lo tanto traducimos'P afirma X" como'leX".

Lvecespartlclpan más de dos nativos: por eiemplo, supongamos quetenemos dos nativos P, y Pr. Supongamoq que k es la proposición de queP, es un caballero; supongamos que \ es la proposición {e qu9 P, es uncáballero. Si participa un tercer nativo P' supongamos que k, es la proposl-ción de gue Pr es un caballero, y asl sucesivamente, para todos los nativosque participaí. Por lo tanto traducimos nP, afirma X" como "kr=X";traducimos nP, afirma X" como -lq=X'; y así sucesivamente.

Ahora consideremos él primer problema del capítulo J. Participan dosnativos P, y P, (el marido y la mujer). Se nos dice quc P, afirma que P, YP, son ambos bribones; tenemo.s que determinar de <¡uó clase son Pry Pr,Ahora bien, k, es la proposición de que Pl es un caballcr<¡, en consecuencia-k,es equivalente a la proposición de que P, es un bribón (dado que todohabitante es un caballero o un bribón, pero no ambas c;osas). De modosimilar, -\ es la proposición de que P, e,s un bribÓn, l)or. lo tanto la pro-posición db que PryPrson ambos bribones es -k,&'-k' P, afirma laproposición -kr&-\. Entonces si usamos nuestro métotkr de traducciÓn,

7

h)i)Dk)l)m)

50 5l

Page 28: SMULLYAN, R. Juegos por siempre misteriosos. España. Gedisa 2008

lt n'allilarl dc la situación es que k,=(-k,&-k ) es verdadera. Por lo tantor.l ¡rrrrlr lt.rnir ¡rrrctk: ¡t lantcarse en los siguientes términos puramente¡ rr n¡ ror,ir ' iorr.¡lr 's, | )rrrlrrs rlos proposiciones k, y k, tal que k,=(-k,&-k) esr.rr l r ,L ' r , t , ¿r r r f l r .s s ' r I l ,s valores de verdadbe k,y kr? Si cóntecóionarnosrr,r t,¡ lr l¡ r lr. vr.rrl lrrl, podcmos observar que el único caso en quel,r:r lrr,\ i lr,) rt,srrlr:r vcrdadera es cuando k, es falsa y k, es verdadera.r i ,rrrrl ' i / 'rr lrr.nr( )s vis[() csto por medio del razonamiento óon senüdo co-r¡rur r u.ur(l{) rt.solvirnos el problema en el capítulo 3.) El resultado es queLr ¡,ro¡rosir ' i{rrr (k,=(*k,&-k)):-k, es una tautología, y también lo es lal , r . | )osr( i i rn (k,=(-kr&-k)) :kr .

'lixlo cl c:ontenido matemático de este problema es que para¡rr,¡xrsrcioncs oalesquiera kry k, la siguiente proposición es una tau-toh r¡.¡r;t: (kr=(-kr&-k2))r(-kr&k).

lil lcctor puede notar también que la proposición recíproca( k,&k):(k,=(-k1&-k)) también es una rautología; en consecuencia la¡rr'<r¡rosición k,=(-k,&-\) es lógicamente equtualente

^ l^ proposición

"k.&k^.' Ahor¿ consideremos el segundo problema del capítulo 3. En este caso

P, afirma que o P1 o P2 es un bribón. Llegamos a la conclusión de que p,es un caballero y P2 es un bribón. El contenido matemático de este hechóes que la proposición (k,=(-k,v-k))r(kr&-k) es una tautologla.

El lector puede adverür de pasada que lá recíproca también es ver-dader.a, y por lo tanto que la proposición (k,=(-krv-k))=(kr&-k) es unatautología.

La traducción del problema 3 es de particular interés teórico; en reali-dad, considerémoslo en la forma más general del Teorema I, capítulo 3.Tenemos a un nativo P que afirma sobre cierta proposición q que si p esun caballero, entonces q es verdadera (q podría ser la proposición de quela esposa de P es un caballero, o de que hay oro en la isla, o oalquierproposición). Supongamos que k es la proposición de que P es un caballe-ro. Por lo tanto P afirmala proposición k>q, y en consecuencia la realidadde la sin¡ación es que le(bq) es verdadera. A parür de esto debemos de-terminar el valor de verdad de k y q. Como ya vimos*k y q deben ser ambasverdaderas. Por lo tanto el contenido matemático del Teorema I, capítulo3, es que q<=¡toq))>(k&q) es una rautología. Por supuesto este hecho nodepende realmente de la naturaleza concreta de la proposici1nki para todaproposición p y e, la proposición (p=(p>q))>(p&q) es una taurología. Laproposición inversa, (p&q)¡(f(p=q)), también es una raurología, porquesi p&q es verdadera, p y q son ambas verdaderas, entonces poq debe servcrdadera; en consecuencia p=(p¡q) debe ser verdadera. Y por lo tantol;r si¡¡uiente es una tautología: (p=(p>q))=(p&q).

Ahora consideremos el problema 4, o mejor dicho el Teorema II, encl crr¡rítulo 3. En este caso P afirma que P es un caballero si ysólo sl q. Nue-v¡rn('ntc supongamos que k es la proposición de que p es un caballero,y rlc er.stc modo P afirmala proposición leq. por lo tanto sabemos queli+( li=r¡) cs verdadera. A partir de esto podemos determinar que q debe ser

verüdera, y por lo tanto el contenido matemático esencial del Teorema II,capítulo l, es que la siguiente es una tautología: (le0eq))>q.

Esta tautología es lo que yo llamaría tautologla de Goodman, dado quesurge del problema de Nelson Goodman, que se analiza en el capítulo 3.

Elerclcto.f . Consideremos tres habitantes P' P2 y P, de la Isla de losCaballeros y los Bribones. Supongamos que Pry F, fórmúlan los siguientescnunciados.

Pr: P, y P, son ambos caballeros.Pr: P, es un bribón y Po es un caballero.¿De qué clase son P' P, y Po?

Ejerctcio 2. a) La siguiente ¿es una tautología?((k,=(trg¡ ¡¡&Qq=(-k,&k J)r((-k r&-k2)&-k3

b) ¿Cómo se relaciona esto con el ejercicio 1?

Ejerctclo 3. Demostrar que la proposición p1 es una consecuencialógica de las dos proposiciones siguientes:

1) pr=_p,2) p¡(pr=-pr)

Ejerclcío 4. Supongamos que P,, P, y P" son tres habitantes de la Islade los Caballeros y los Bribones, y qüe fiy P, formulan los siguientesenunciados:

Pr: P, es un bribón.Pr: P, y P, son de clases diferentes.

a) P" ¿es un caballero o un bribón?b) ¿Óómo se relaciona esto con el Eiercicio 3?

LOS PROBLEMAS DE OONA

Muchos de los problemas de Oona del capítulo 4 también puedenresolverse por medio de tablas de verdad. Consideremos, por ejemplo, elprimero: Tenemos dos nativos Pry Pr P, afirma que si p, y P, son amboscaballeros, entonces Oona está en la isla. Prafirmalo misriro. Supongamosque O es la proposición de que Oona está en la isla. Si usamos nuesro mé-todo de traducción, sabemos que las dos proposiciones siguientes son ver-daderas.

1) k,=((k,,c¡19¡-9¡

53

Page 29: SMULLYAN, R. Juegos por siempre misteriosos. España. Gedisa 2008

2) k=(q&kJf,O)Tenemos que determinar si O es verdadera o falsa. Bueng, si con-

feccionamos una tabla de verdad parala conjunción de 1) y 2), es decir,para 0<r=(0<r&k)>O))&Qq=(l¡r&q)>O)), veremos que la úlüma columnacontiene V sólo en los lugares en que O tiene el valor V. Por lo tanto O debeser verdadera.

Ejerclclo 5. Supongamos que el marido va en busca de Oona y seencuentra con dos nativos A y B en una isla y éstos formulan los siguientesenunciados:

A: Si B es un caballero, entonces Oona no está en esta isla.B: Si A es un bribón, entonces Oona no está en esta isla.

¿Está Oona en esta isla?

Ejercicio 6. En otra isla, dos naüvos A y B formulan los siguientesenunciados:

A: Si alguno de los dos es un caballero, entonces Oona está en esta isla.B: Si alguno de los dos es un bribón, entonces Oona está en esta isla.

¿Está Oona en esta isla?

Ejercicio 7. En otra isla, dos naüvos A y B formulan los siguientesenunciados:

A: Si soy un caballero y B es un bribón, entonces Oona está en es-ta isla.

B: ¡Eso no es cierto!

¿Está Oona en esta isla?

RETORNO A LOS MARCIANOS Y "VENUSINOSMuchos de los acertijos Marte-Venus-mujer-varón del capínrlo 5 tam-

bién pueden resolverse con tablas de verdad, aunque el método de tra-ducción que se necesita es un poco más complejo, y esta clase de proble-ma no es considerado más en este libro.

Enumeremos a los socios del club en cierto orden P' p, p3, etc., y p racada número i, supongamos que \ es la proposición que P. és venusino,y supongamos que { es la proposición que P es mujer. Entonces P. es tnar-cianopuede escribirse -V,, y P, esuarón puede escribirse -F,. Ahora bien,P dice la verdad si y sólo si P, es o una mujer venusina o un varón marciano,lo cual puede simbolizarse (V,&F)V(-V,&-F), o en forma más sencilla,V,=F,. Y entonces si P, afirma una proposición X, la ¡ealidad de la situaciónc:s la siguiente proposición: CV¡F)=X.

54

Entonces éste es nuestro método de traducciÓn. Sienr¡rre que P, afirmeX, escribimos: (V=F)=X.

Consideremos, por ejemplo, el problemaT del capftulo 5 (página36):<rl caso de Ork y Bog. Supongamos que Pl es Ork y P, es llo¡¡. Entonces$e nos dan las cuatro proposiciones siguientes (después de aplicar el méto-do de traducción):

1) CVr=Fr)=\2) (Vr=F)=-V,3) (V,=F,ts-F,4) (V,=F)=F,

Entonces, los valores de verdad de V,, F,, V, Y F, pueden determinarsepor medio de una tabla de verdad. (Hay cuatro incógnitas \, Fr, \y F,aquí, ¡y por lo tanto hay que considerar dieciséis casos!)

Not4.' No todos los problemas de menürosos y veraces pueden resol-verse por medio de los métodos de traducciÓn de este capltulo. Estosmétodos resultan saüsfactorios para los problemas en los cuales se nos in-forma lo que se dice y luego debemos dcducir ciertos hechos acerca dequienes lo dicen. Pero para los tipos más dificiles de acertijos en los que¡.enemos que inventaruna pregunta o un cnunciado para hacer ciertztare ,se necesita pensar más. Existen otros mél()(los sistemáticos que a¡rdan enmuchos casos, pero ése es un tema quc sc aparta de la línea principal depensamiento en este libro.

RESPUESTAS DE I,OS IilIIRCICIOS

l' Los tres son bribones.

2. La proposición es una tautologfa.

3. Lo dejamos a cargo del lector.

4. P, es un caballero.

5. Oona no está en la isla (y ambos Rallv(,'r son caballeros).

6. Oona está en la isla (y ambos nat¡vlrfi non caballeros).

7. Oona está en la isla (y A es un cabellero; ll es un bribón).

55

Page 30: SMULLYAN, R. Juegos por siempre misteriosos. España. Gedisa 2008

8 5

Ahora llega un ser de Urano que sólo comprende el conector >.llntonces nos resulta imposible explicarle lo que & significa, ni lo que signi-fica -, pero es posible definir v en términos del único conector >. ¿Cómosc puede hacer? (I¿ solución no es en absoluto evidente. El hedro de quesea posible hacedo es parte del folklore de la lógica matemáüca, pero nohe podido encontrar al lógico que lo desq.¡brió.)

6

Resulta obvio que = puede definirse en términos de -, &, y v. Mostrardos formas diferentes de hacerlo.

7. Un problema especíal

Supongamos que un ser dcl cspacio exterior comprende los sig-nificados de > y =. Entonces nos rcsultará imposible explicade el signi-ficado de -, p€ro será posible explir:arlc &. ¿Cómo se puede hacer? (Estedescubrimiento es mlo, hasta donde yo sé.)

Dos conector* más. Lhora otr.scrvamos que todos los conectores lGgicos -, &, v, r y = pueden definlrse a parür de los conectores - e &, o al-ternativamente a partir de - y v, o alternativamente a partir de - y>. ¿Existevt-únlco conector lógico a partir del nral puedan definirse los cinco co-nectores? El lógico Henry M. Shcffer resolvió este problema en 1,91,3.Definió plq como que p y q no son lrrrlt¡s verdaderas. El slmbolo ul', seconoce como la "matca de Sheffer"; ¡'rrxlernos interpretar flq como "p y qson imcompatibles" (por lo menos una es fjrl.sa). Demostró que -, &, v, =,= pueden definirse a parür del sfmbr¡kr rlr. ll rnarca. Otro conector lógicoque da todos los otros conectores es l; e¡te sirnbolo se denomina símboloparala negaclón conJunta.ln¡erprctarrror ¡rlr¡ como 'p y q son ambas fal-sas", o 'ni p ni q son verdaderas".

I

¿Cómo pueden definirse -, &, V, f,, . ¡t ¡iutir de la marca de Sheffer?¿Cómo pueden definirse todas ellas a párilr rle l;r negación conjunta?

Ia corrstante lógtcat. Una propotlc.lf¡rr X ¡e dcnomina l.óglcammtecontradlctoña o lógicarnmte falsa sl óu ¡teg¡t¡ lf¡n *X es úna tautología.Por ejemplo, para toda proposición p, ll ¡rrn¡rosir ifrn (p&-p) es lógica-mente falsa. Por 1o tanto también lo er p. ¡r

Vamos a usar el símbolo ahora estÁndef " l" ¡r¡r'¡ representar algunafalsedad lógica particular (no importa cr,¡ál) Fsrr ¡ ¡rrrc:<lc considerarsé fi¡opara eI resto de este libro, y para cualquler ol¡l lllrr o rlt¡e puedan leer en

Clausura y consistencia lógicas

INTERDEFINIBILIDAD DE LOS CONECTORES LOGICOS

Ahora ya conocemos los cinco conectores lógicos básicos: -, &, v, )y =. Podríamos haber comenzado con menos y haber definido los otros entérminos de ellos. Esto pr.rede hacerse en varias formas, algunas de lascuales serán ejemplificadas en los siguientes acertijos.

I

Supongamos que un hombre inteligente de Marte desciende en laTierra y quiere aprender nuestra lógica. Afirma que comprende las palabras"no" e "y",.pero no sabe el significado de la palabra oo,'. pómo podríamosexplicársela, usando solamente los conceptos de rn e y?

Vamg¡ a expresar el problema en otra forma. Dada cualquierproposición p, él conoce el significado de -p, y dada toda proposición py q, él sabe el significado de p&q. Lo que busca el marciano es una formade escribir una expresión o fórmula con las letras p y q, vsando solammtelos conectores lógicos - y &, tal que la expresión seá lógicamente equi-valente a la expresión pvq. ¿Cómo se puede hacer?

2

.. Ahora supongamos que una dama del planeta Venus viene aquí y nosdice que entiende los significados de - y v, pero quiere una explicaciónde &. ¿Cómo podemos definir & en términos Ae - y f,i Ces decir, ¿qüé expre-sión en términos de p, q, - y v es lógicamente equivalente a p&q?)

-

3

En este caso desciende un ser de Júpiter que comprende -, &, y v, yquiere que le definamos > en términos de estos conectores. ¿Cómo puedehacerse?

4

Luego llega un ser de Saturno que curiosamente entiende - y I, perono comprende & ni v. ¿Cómo podemos explicade & y v?

56 57

Page 31: SMULLYAN, R. Juegos por siempre misteriosos. España. Gedisa 2008

el (lrre aparezc? estc símbolo. (El símbolo "JJ' se pronuncia oetn; es elal¡¡rlxrlo "' l '" e.gr:rito al rcvés.)

I'lra ltxh ¡rroposición p, la proposiciónJ-cp es una tautología (porqueI rs llrgicanrcntc falsa, por lo tanto -Lfp es verdadera, no importa que pser ve rrledcra o falsa). Por lo tanto toda proposidón es una consecuencialrrgir:a clc 1,. (Ils una suerte que I en sí misma no sea verdadera, porquesi lo fuera, todo sería verdadero ¡y el mundo entero estallariaD

Muchas formulaciones modernas de la lógica proposicional basan to-da su teoría simplemente en > y I, porque todos los otros conectoreslógicos pueden definirse a partir de esos dos (véase el problema 9 másabajo). Este es el camino que vamos a seguir porque es el que se adaptamejor a los problemas de este übro. Entonces T se define como l:I Nosreferiremos a I como falsedad lógtcay aT comotnrdad lóglca(obvirmenteT es una tautología).

9

¿Cómo se definen todos los conectores lógicos a parut de > y J?

CLAUSURA LOGICA

Una tná4ulna lógicamente callfcada. Para ilustrar el importanteconcepto de clausura lógica, imagínemos una computadora programadapara demostrar varias proposiciones. Cada vez que la máquina demues-tra una proposición, la imprime (más precisamente, imprime vne oraclónque expresa la proposición). La máquina, si se la deja sola, firncionará eter-namente.

Llamaremos a la máquina lógicamente calificada si satisface las doscondiciones siguientes:

1) La máquina demostrará toda tautología tarde o temprano.2)Para proposiciones cualesquierapy q, si la máquina llega a demos-

trar p y a demost¡ar prq, entonces tarde o temprano demostrará g. (Po-dríamos pensar que la máquina infiere qa parttr de las dos proposicionesp y prq. Por supuesto la inferencia es válida.)

Las máquinas lógicamente calificadas tienen una importante pro-piedad, sobre la cual el siguiente problema ilustra algunos ejemplos.

10

Supongamos que una máquina es lógicamente calificada.a) Si la máquina demuestra p, ¿demostrará necesariamente --p?b) Si la máquina demuestra p y demuestra q, ¿demostrará necesa-

riamente la proposición singular p&q?

Clausura lógica. En lógica, hay una anügua regla llamada modus

l,onens, que dice que depués de demostrarp y de demostrarprq, entoncessc puede inferir q. Se dice que un conjunto de proposiciones C es ceradol>aio modusponerusi para proposiciones cualesquier^ py g, si p y p>q es-tán ambas en el conjunto C, también lo está la proposición q. Ahora pode-rnos decir que un coniunto de proposiciones C es lóglcammte cen'ado sisc cumplen las dos condiciones siguientes:

Condíción /. C conüene todas las tautologías.Condlctón 2. Para proposiciones cualesquiera p y q, si p y p>q están

cn C, también lo está q.

Por lo tanto un conjunto lógicamente cerrado es un conjunto que con-tiene todas las tautologías y que es cerrado bajo mod.us ponens. (La razónde que se denomine lógicamente cerrado pronto resultará obvia.)

Decir que una máquina es lógicamente calificada equivale a decir quecl conjunto C de todas las proposiciones que la máquina puede demostrarcs un conjunto lógicamente cerrado. Sin embargo, los conjuntos lógica-mente cerrados también surgen en sin¡aciones en las que no participaninguna máquina. Por ejemplo, vamos a considerar sistemas matemáticosen los cuales el conjunto de todas la.s proposiciones demostrables en elsistema es un conjunto lógicamente cerrado. Además, gran parte de estelibro versa sobre lógicos cuyo conjunto de creencias es un conjunto lógi-camente cerrado,

Consecuencla lóglca. Hemos dcfinido que Y es consecuencia lógicade X si la proposición X>Y es una taut<llogía. Diremos que Y es consecuen-cia lógica de dos proposiciones X, y Xrsi cs consecuencia de la proposiciónXr&\: en otras palabras, si (K,&Xr)rY cs una tautología, o, lo que es lomismo, si la proposición \rO!>Y) cs un;r tautología. Hemos definido queY es una coruecuencia lógica dc X,, X,, y X. si (Q(&X)&X)=Y es unatautología, o lo que es lo mismo, si X, ,('X, (XrrY)) és uña taútología. Demodo másgeneral, para todo conjurrto f i¡rito dc proposiciones \, ,.., \,podemos decir que Y es consecuene:i:r lfrgica dc ese conjunto si la propo-sición (Xr&...&QrY es una tautol()¡{¡íI.

La importancia de los conjuntos l(r¡lt lrncntc cerrados es que poseenla siguiente propiedad:

Htlclplo I(Hnclplo de Clausurtt l,ó¡4IuD. Si C es lógicamente ce-rrado, entonces para n proposicionc.r t rrrlesr¡rricr, \,...,\ de C, todas lasconsecuencias lógicas de estas n pro¡roslr'iorr(.s también están en C.

Dlscuslón. Si volvemos a nuestr(, elcrrr¡rlo tlc: máquinas lógicamentecalificadas, el Principio L nos dice que el elgrrrru ve:z una máquina dem¡és-trauna proposiciónp, entonces tarde o lelit¡il.lru r (lcmostrará todas las con-secuencias lógicas de p; si demuestra rl¡¡rrrra vrz rkrs proposiciones p y q,

59

Page 32: SMULLYAN, R. Juegos por siempre misteriosos. España. Gedisa 2008

tarde o temprano demostrará todas las consecuencias lógicas de p y q, yasí siguiendo para cualquier número finito de proposiciones.

Lo mismo se apüca a los lógicos cuyo conjunto de creencias es lógica-mente ceffado. Si un lÓgico llega a creer p, tarde o temprano creerá todaslas consecuencias lógicas de p; si llega a creer p y q, creerá todas las conse-cuencias lógicas de p y q; etc.

11

¿Por qué es correcto el Principio L?

12

Otra propiedad.importante de los conjuntos lógicamente cerados esque si C es lógicamente cerado y contiene cierta proposición p y sunegación -p, entonces toda proposición debe estar en C. ¿Por qué?

CONSISTENCIA

El úlümo problema nos lleva al importante concepto de consistencia.Un conjunto C lógicamente cerrado se denomina lnconslstente si contie-ne J- y conslstente sí no contiene l-.

La siguiente es otra propiedad importante de los conjuntos lógicamen-te cerrados.

hlnclplo C. Supongamos que C es lógicamente cerrado. Entonces lastres condiciones siguientes son todas equivalentes (cualquiera de ellasimplica las otras dos):

1) C es inconsistente (C contiene -t).2) C contiene todaslas proposiciones.3) C contiene cierta proposición p y su negación -p.

Nota: Dado un coniunto S de proposicione$ que no es lógicamentecerrado, se dice que S es inconsistente si J- es una consecuütcla lógtcadealgún subconjunto finito 4,...,\ de proposiciones en S. CEntre paréntesis,esto equivale a decir que toda proposición es una corisecr¡encia lógica dealgún subconjunto finito de S.) Sin embargo, vamos e tratar casi exclusi-vamente con grupos que son lógicamente cerados.

Demostrar el Principio C.

13

SOLUCIONES

1. Decir que por lo menos una de las proposiciones p y q es verdadera

r:s decir que no es cierto que p y q sean ambas falsas; en otras palabras,rro es cierto que -p y -q sean ambas verdaderas. Y por lo tanto pvq esr:quivalente a la proposición -(-p&-q). Dado que el marciano compren-rlc - y &, entonces comprenderá -(-p&-q). Y en coruecuencia podemos<lccirle al marciano: "Cuando digo p o q, lo único que quiero decir es queno es cierto que no p y no q'.

2. Decir que p y qson ambas verdaderas equivale a decir que no escierto que o p o q sea falsa. En otras palabras, no es cierto que o -p o -qsca verdadera. Y por lo tanto p&q es lógicarriente equivalente a la proposi-r:ión -(-pv-q).

3. Esto puede hacerse de varias formas: Por un lado, poq es lógr-camente equivalente a -pv(p&q).También es equivalente a -(p&-q) (nocs cierto que p sea verdadera y q sea falsa), y a -pvq.

4.pvq, es lógicamente equivalente a -prq. Y por lo tanto podemosobtener v en términos de - y >. Una vez que tenemos - y v, podemosobtener & según la solución del problema 2. De modo más directo, p&qes equivalente a -(p>-q), como puede verificar el lector.

5. Este realmente es intrincado. La proposición pvq resulta lógicamen-te equivalente a (prq)>q, como el lector puede verificar con una tabla deverdad.

6. eq es obviamente equivalente a (p=q)&(q>p). También es equi-valente a (p&q)v(-p&-q).

7. Ya lo hemos resuelto en el último capíh-rlo, en conexión con el tercerproblema de caballeros ybribones; demostramos que p&q es lógicamenteequivalente a f(p>q).

8. Dada Lamarca de Sheffer, podemos obtener los otros conectores dela siguiente manera: En primer lugar, -p es lógicamente equivalente a flp.(La proposición plp es que por lo menos una de las proposiciones p o pes falsa, pero dado que las dos proposiciones p y p son la misma, estosimplemente quiere decir que p es falsa.) Ahora que tenemos -, podemosdefinir pvq como -(flq). @ado que plq es la proposición que por 1o menosuna de las proposiciones p o q es falsa, su negación -(Hq) equivale a decirque ambas no son falsas, es decir, que por 1o menos una es verdadera.) Unavez que tenemos - y v, podemos obtener & (según Problema 2), luego >y = (según los problemas 3 y 6). Por lo tanto obtenemos -, &, y, >, y = apartir de la marca de Sheffer.

Si comenzamos con la negación conjunta J, en lugar de la marca de

60

Page 33: SMULLYAN, R. Juegos por siempre misteriosos. España. Gedisa 2008

lihe l'fer, ¡rroc:eclcmo..i cle la siguiente manera: Primero definimos -p como¡rllr l,rre¡4o rlcflninro.s l)vq como -(pJq). IJnavez que tenemos - y &,{rnlr¡n('ra lrrxlrrnos ol¡tt:ncr todo el resto en la forma que ya ümos.

,r l.¡ lrrr ¡lx ¡sir:i(rn -p es lógicamente equivalente a pil, y por lo tantol'ulr.rr¡(),.i olrlcru:r - a partfu der yI. Unavez que tenemos -y->, podemOS¡ ¡l ¡tr'rrr'r v y & (por medio del problema 4). Entonces podemos obtener=J lur l r t l t : : y &.

I0. a) Supongamos cJue la máquina demuestra p. También demostraráf ) )--p (dado que es una tautología), en consecuencia imprimirá --p (porla scgunda condición que define la calificación lógic¿).

b) Supongamos que la máquina demuestra p y demuestra q. Ahorabien, la proposición p>(q>(p&q)) es una tautología; en consecuencia lamáquina lo demostrará. Una vez que la máquina ha demostrado p yp:(p(p&q)) debe demostrar p(p&q). Una vez que la máquina lo hademostrado, entonces dado que demuestra q, debe demostrar p&q.

@n realidad este problema no es más que un caso especial delsiguiente, como podrá ver el lector.)

11. Primero consideremos el caso n=1.: Supongamos que \ está en C,y que Y es una consecuencia lógica de \. Entonces Xr:Y es una tautología,en consecuencia está en C (según la condición 1). Dado que \ y X1>yestán ambas en C, también lo está Y (según la condición 2).

Ahora consideremos el caso n=2: Supongamos que \y4 están ambasen C, e Y es una consecuencia lógica de 4 V 4. Entonces (X'&X)>Y esuna tautología, en consecuencia \>O!>Y) és una tautología (cómó puedeverificar el lector) y por lo tanto está en C. Dado que \ V \)O!>Y) esránambas en C, también lo está 4pY (condición 2). Dado que 4 y \>Y es-tán en C, también lo está Y (nuevamente según la condición 2).

Para el caso n=J, supongamos que Y es una consecl¡encia lógica deXr, 4 y Xr. Entonces XrtQt=QLtY)) es una tautología: es lógicamenteequivalente a Oq&4&X)>Y. Entonces, si usamos la condición 2 tres veces,obtenemos sucesivamente \>(Xr>)D en C, frpY) en C, y finalmente yen C.

Debe resultar obvia la forma en que esta prueba se generaliza para¡o-do número entero positivo n.

Nota:Y^ hemos dicho que el problema 10 sólo es un caso especial deeste problema. La razór1 por supuesto, es que --p es una consecuenciaIógica de p; en consecuencia todo conjunto lógicamente cerrado que con-tenSa p también debe contener --p. Además, p&q es una consecuencialógica de las dos proposiciones py q, porlo tanto todo conjunto lógicamen-tc cerrado que contenga tanto p como q también debe contener p&q.

12. Supongamos que p y su negación -p están ambas en C y que C eslcrgicamente cerrado. Supongamos que q es cualquier proposición. La

(r2

¡rroposición (p&-p)>q es una tautología (como puede verificar el lector( on una tabla de verdad, o de modo más simple si se observa que dador¡ue p&-p debe ser falsa, entonces (p&-p)rq debe ser verdadera). Y porlo tanto q es una consecuencia lÓgica de las proposiciones verdaderas py -p. Entonces según el Principio L, la proposición q también debe estarr:n C. Y por lo tanto tada proposiaón q está en C.

1.3. Vamos a verificar que las tres condiciones son equivalentes demos-trando que 1) implica 2), que a su vez implica 3), que a su vez implica 1).

Supongamos 1). Dado que toda proposición es una consecuencialógica de J- y J- está en C, entonces toda proposición está en C (según elPrincipio L). Por lo tanto 2) e-s váLida.

Resulta absolutamente evidente que 2) implica J), porque si todaslasproposiciones están en C, entonces para toda proposición p, tanto p como-p están en C.

_ Ahora fl¡pongamos que 3) es válida; es decir, que para cierta p, tantop como -p están en C. Dado que Ies una consecuencia lógica de p y -p,entonces l- debe de estar en C (según el Principio L); es decir, C debe deser inconsistente.

63

Page 34: SMULLYAN, R. Juegos por siempre misteriosos. España. Gedisa 2008

Parte IV

SEAMOS CUIDADOSOS

I

I

Page 35: SMULLYAN, R. Juegos por siempre misteriosos. España. Gedisa 2008

¿Paradójico?Ahora tenemos la base para embarcarnos en nuestro üaje hacia la com-

¡rlicada problemática de Gódel sobre la consistencia, a la que llegaremost:n el capltulo 12, y encontraremos muchos problemas interesantes en elr:amino. Vamos aempezarconunproblema relacionadoestrechamente conr¡na de las variantes de la Paradoja de la Prueba Sorpresa del capínrlo 2.

Regresamos a la Isla de los Cabaüeros y los Bribones, en donde lassiguientes tres proposiciones son válidas: 1) los caballeros sólo formulancnunciados verdaderos; 2) los bribones sólo formulan enunciados falsos;.l) todo habitante es un caballero o un bribón. Vamos a referirnos colec-tivamente a estas tres proposiciones como las 'reglas de la isla".

Recordemos que ningún habitante puede afirmar que no es un ca-ballero, dado que ningún caballero formularía el enunciado falso de queno es un caballero y ningún bribón formularía el enunciado verdadero deque no es un caballero.

Ahora supongamos que un lógico visita la isla y se enq.¡entra con unnativo que le formula el siguiente enunciado: "Usted nunca sabrá que soyun caballé¡o"

¿Tenemos una paradoja? Veamos. El lógico empieza a razonar delsiguiente modo: "Supongamos que es un bribón. Entonces su enunciadoes falso, lo cual significa que en algún momentoyo sabré que es un caba-llero, pero no puedo saberqve es un caballero a menos que realmente losea. En consecuencia, si es un bribón, se sigtre que debe ser un caballero,lo cual es una contradicción. Por lo tanto no puede ser un bribón; debe serun caballero".

Hasta ahora todo va bien, hasta ahora no hay ningrna contradicción.Pero luego sigue razonando: "Ahora sé que es un caballero, aunque él dijoque nunca lo sabría. Entonces su enunciado era falso, lo que significa quedebe ser un bribón. iParadojar.

Pregunta. ¿Es ésta una paradoja genuina?

Aná,ltsís. Este problema da lugar a muchísimo análisis. Por empezar,la pttadoja (si realmente lo es) es lo que se llamaria para.doja pragm.áticamás que una puramente lógica, dado que supone no sólo conceptos lógicostales como verdad y falsedad, sino conceptos pragmáticos tales como

67

Page 36: SMULLYAN, R. Juegos por siempre misteriosos. España. Gedisa 2008

I lrtrth ét I 'ara rlr '¡¡r¡r ¡¡ l:¡ rr¡tkrl<' ¡)ra¡lmática de la cuesüón, ¿no es posibler¡i¡F arrld , ., rrr ¡r,'art,jl tk: at:ucrdo a quién se le dirija la afirmación?¡lr ' l¡r, l4l 'h-rr. 'rrr¡. ¡r! ( l rrrr '. j t:rrrgrlo extremol seguramenF;l nativo podría,l¡' rrqr'f r I d

" rrr('rr(), y

'o surgiría ninguna páradop. (señala el cá¿¿rre,

1' ,li,;. ".Nrrrr, ¿ srlx.á que soyun caballeio,,. Büeno, indudablemente tiener i.'rrr r, r'l r etlrvcr jamás sabrá realmente queél es un caballero, yporlotantor'l r r¡riv, r:s cn realidad un cabailero, ya que lo que dijo er.i.riá.'poo.o^or'l r':rrlávcr nunca lo sabrá, no zurge ninguna contradióción.) para coÁ¡¿"r",rrrr cjcmplo menos extremo, el nativo podría decirle

"ro " rr.r" ;;;;;;;;cstá viva pero es sorda y por lo tanto nó escucha el enu nci"ao ; ri.reu"rrreir.,

no surgiría ninguna paradoja.Por lo tanto debemos suponer- que el visitante de la isra estaba vivo yescuchó el enunciado, rero eito todaüa no es suficiente. úb;;;;;*

una cierta capacidad de razonamlmto de parte del visitante, poráu! ,i "fvisitante no-üene capacidad de razonamienio, no habrí" pl""tl-"átá

"rn"-m9":9 qg9 he presenrado. @l nativo diría: .Usted nunca sabrá q"",of'rr'caballero'. Entonces el visitante podría decir: 'eué i"terera"iJl i*ir, yjamás volver a pensar en el asunto. por lo tanto no surgiría ningunapandoja.) y en consecuencia debemos explicar qué üpo a."."p".iá"?i"razonamiento posee el lógico.

.Decimos que un individuo es vn ftEonador der ttpo 7 si comprende:lr:F:ry

la. lógica proposicional, es decir, si se cumple" lir-¿t,concuoones siguientes:

1) Cree todas las tautologlas.2-)-Para proposiciones cualesquiera X e y, si cree x y cree x)y, entonces

cree Y.

. con la terminología del capítulo g, el conjunto de creencias de un razo-narlc'r,de^l üpo 1 es lógicamenie cerrado. En'tonces, por el principio i áetilplrllrlti"lylta

que d.ado cualquier conjunto finito S ae p.opostáones enlas que cree' también debe creer todas las consecuenciaé logicas de s._ Hagamos ahora la hipótesis de que el visitante de la ist"

"J"" áro""¿o.

del üpo 1. Por-supuesb ésta es una ñipotesis srrÁamentre idealizada ya ouecomo hay rnfnitastautorogÍas, impliia argo asi como ta inmoiül¿?,ii"rnzonador. Sin embargo, pequeñeces corño éstaho nos preocupan en elreino atemporal infinito.dé h matemática. simplemente imaginamos al ra-3o1ad9¡ programado tal que 1) tarde o tr-pá.ro creerá cuálquier tauro-logia;2) si alg'na vez c.ee p y arguna"",

".á" p,,q, entonces tarde o tem-prano creerá q.

Todavla necesitamos más hipótesis. En primer término debemos su-poner cierta autoconciencia de parte del razonador; especificamente, Jeü-mos suponer que si el nzonador llega a saber algo, éntonces ,a¡e'q.re iosabe (de otro modo nunca podría haÉr dicrro, 'eñoia sñ;;,Ji;"i;r;ro", y mi argumento no funcionaría).

Considerando el problema con todos estos supuestos, ¿surge ahora unaparadoja genuina? Tod avia no, porque aunque tes ai,¡e d; É;óí;;;;ü

68

¡:,la son válidas (todo habitante es un caballero o un bribÓn; los caballerosrrrkr formulan enunciados verdaderos; los bribones sólo formulan enun-, r:rdos falsos), debemos hacer la hipÓtesis adicional de que el razonadori rr.e que las reglas de la isla son válidas. En realidad, es perfectamente

',,ronable que el razonador pueda creer esto al principio, pero después de

(.ncontrarse frente a una contradicción al seguir el argumento que yo

¡rrcsenté, tendría bases racionales para dudarde las reglas de la isla. (Merrrragino que ustedes y yo haríamos exactamente lo mismo después de(.rrcontrarnos en esa encrucijada.) Bueno, para hacerel problema realmenterrtcresante, finalmente hagamos la hipÓtesis de que el razonador cree yrl¡¡ue crqtmda en las reglas de la isla.

I

Ahora nos encontramos con un problema interesante. Según los su-

l)uestos adicionales que hemos formulado, el argumento del razonador der¡ue el nativo es un caballero y el nativo es un bribón parece perfectamenteválido. Sin embargo ¡el nativo no puede ser un caballero y un bribÓn al mis-rno üempo! Entonces ¿cuál es el error en el argumento del razonadot?

Soluctón. Formulé el problema anterior en forma muy engañosa. (¡Aveces siento que soyun poco tramposo!) Lo que está mal noes el argumentodel razonador; es que la situación que describÍ nunca podrÍa haberse pro-ducido. Si un razonador de tipo 1 llega a una isla de caballeros y bribonesy cree las reglas de la isla (y escucha todo enunciado que se le formule),entonces es lÓgicamente imposible que algún naüvo le diga: 'Usted nuncasabrá que soy un caballero".

Para demostrado, no necesitamos uno de los supuestos que hemos for-mulado; es decir, que si el razonador sabe algo, entonces sabe que lo sabe.Aun sin este supuesto, podemos obtener una contradicciÓn como ésta: Elrazo¡ador razona:. "Supongamos que es un bribón. Entonces su enuncia-do es falso, lo que significa que sabré que es un caballero, 1o cual implicaque realmente es un caballero. Por lo tanto el supuesto de que es un bribónlleva a una contradicción, entonces debe ser un caballero".

Sin ahondar más en el proceso de razonamiento del lÓgico, podemosinferir una cont¡adición. Hasta ahora el lógico ha razonado correctamentey ha llegado a la conclusión de que el nativo es un caballero. Dado queha razonado correctamente entonces el nativo realmente es un caballero,y por lo tanto el razonador saáe que el netivo es un caballero. Sin embargo,el nativo dijo que jamás lo sabría, en consecuencia el nativo debe ser unbribón. Por lo tanto el naüvo es un caballero y un bribón, lo cual es unacontradicción.

En vez de usar la palabra "saber" también podríamos haber dicho 'creercorrectamente' y nuestro argumento aún funcionaría. Decimos que unindividuo cree correctanmtevna proposición p si cree p y p es verdadera.

Ahora hemos demosrado el Teorema I. ,

69

Page 37: SMULLYAN, R. Juegos por siempre misteriosos. España. Gedisa 2008

Tntema I. Dada una isla de caballeros y bribones y un razonador deüpo 1-q]¡e cree las reglas de la isla (y escucha todo enunciado que se leformule), es lógicamente imposible que un nativo pueda decirÉ .ustednunca creerá correctamente que soy un caballero'.

A.y4lisLs: Ngonos lectores críticos podrían oponerse a la demostraciónque di del reorema I sobre la base de que le he cbnferido al razonador máscapacidad de la que le atribuí explícitamente: el razonador fornlula zupues-tos y posteriormente los desc¿rr^. En el caso específico que tenemos-entremanos' el razonador comenzó diciendo: "supongamos q.re es un bribón.Entonces...D. Bueno, realmente ésta es una cuestión de ionveniencia, node necesidad. Podría haber planteado el argumento del razonador en la si-guiente forma, más directa: 'si es un bribón, entonces su enunciado es falso.si su enunciado es falso, entonces creeré correctamente que es un caballero.si creo correctamente que es un caballero, entonces es un caballero. Reu-niendo estos tres hechos, si es unbribón, entonces es un caballero. A partirde este último hecho se sigue lógicamente que es un caballero.',

E-n lógica es una práctica usual demostraiuna proposición de la formap?q fsi p_entonces q) suponiendo que p es verdaderjy hego rrarando deinferir q. si esto se puede hacer, entoncés se establece la pro-posición p>q.En oüas palabras, si se supone que p como premisa lleit a q cb-oconclusión, entonces la proposición p)q se ha áemostrado. (nstaiécnicaes parye de lo que se conoce como deducción natural) El asunto es quetodo Io que puedg demostrarse usando ese método también pueoedemostrarse sin é1. (En lógica existe un conocido teorema en este sentidoque se denomina teorema de la dedu.cción.) y por lo tanto vamos apermitides a nuestros 'razonadores" usar la deduóción natural; un razo-nado¡ de tipo 1 que lo h-acg no puede demosrrar más hechos áe los quepuede demostrarsinusarla deducción natural, pero las demostraciones queusan la deducción natural generalmente son mhs breves y fáciles de seguir.Por lo tanto vamos a seguir permitiendo que nuestros razonadores usán hdeducción nan¡ral.

2. Unproblema duát

. . Seguimos suponiendo que el razonador es de üpo 1, que cree las reglasde la isla, y que oye todos los comentarios que se le hacen.

Supongamos que el nativo, en vez de decii, ,Usted nunca creerá correc_tam€nte que soy un caballero", dice, 'usted creerá correctamente que soyun bribón".

Entonces, ¿tenemos una contradicción? (El lector debe tratar deresolverlo antes de leer la solución.)

solución. El razonador razon así: "supongamos que es un caballero.Entonces su enunciado es verdadero, to cuál signiñca que yo creerécorrectamente que es un bribón, lo que a su vez implica que es un bribón.

70

l:n conseo.rencia el supuesto de que es un caballero lleva a una( ontradicción: por lo tanto debe ser un bribÓn".

En este púnto el razonador cree que el naüvo es un bribÓn, y ha

r¡zortado coirectamente, en consecuencia el nagvo es un bribÓn. Por otro

la <lo, dado qu e el razo¡ador cree correctamente que e! nativo es unbribÓn,,:l enunciadó del naüvo era verdadero, lo cual lo convierte en un caballero.I intonces, efectivamente, obtenemos una contradicciÓn'

ALGUNOS PROBLEMAS AFINES

Ahora vamos a deiar por un momento la Isla de los caballeros y los

Ilribones y vamos a consiberar un problema relacionado con la paradoja

rlel capítuio 3. Un alumno le pregunta a su profesor de teología: "¿Existe

,ealménte Dios?,, 81 profesor da la siguiente respr,¡esta curiosa: 'Dios existe

si y sólo si usted no cree correctamente que El existe"'

3

Supongamos que el alumno es un tazonador del tipo 1,.que el enun-

ciado bel [rofesoi es verdadero y que el alumno cree el enunciado.

Entonces, ¿obtenemos una Paradoia?

Soluctón. ¡Sí, así es! Por empezar, aun si olvidamos-que el alumno es

un razonador áe üpo 1 y que cree el enunciado del profesor, se sigue que

Dios debe exisür, porqué si Dios no existiera, entonces el alumno Cree-

ríA corcectamente que Dios existe, pero nadie puede creer corTectAtnente

una falsa proposici'ón. Por 1o tantó Dios debe efectivamente exisür (su-

poniendo-qué el enunpiado del profesor es. verdadero)' Ahora Éien, el alumno, al ser un razonador del üpo 1, conoce la lÓgicaproposicional tanto como ustedes o Yo, Y así también es capaz de razonar

iue'si el enunciado del profesor fuera verdadero, entonces Dios debe

ó<istir. pero también cree él enunciado del profesor; por lo tanto debe creer

que Dios existe. Y dado que hemos demostrado que Dios existe (según los

úer srrpr¡estos del problema), entonces el ,l,,Tto cree co'Tectamente que

Dios existe. Pero Dios existe si y sÓlo si el alumno no cree correctamen-ie que Dios existe. A partir de esto se sigue que Dios no existe si y sÓlo si

el ilumno sí cree correctamente que Dios existe. (Para toda proposición

p y q, la proposiciÓn F-q es iógicamente equivalente a la proposición

-p=o.) Oádo-que Dios no existe si y sólo si el alumno cree correctamen-

té qüe Dios existe, y el alumno sl cree correcamente que-Dios existe'

entónces se sigue que Dios no existe. De este modo las tres hipótesis del

problema lleván a ia pandoia de que Dios existe y no exis-te'Por zupuesto surgifía la misma paradoja si, en cambio, el profesor hu.

biera dicho: "Dios existe si y sÓlo si uited cree correctamente que El no e:<is-

te,,. Deiamos la demostración de este problema comoejercicio para ellector.

77

Page 38: SMULLYAN, R. Juegos por siempre misteriosos. España. Gedisa 2008

Ahora-es importanJe que nos demos cubnta de que la p atadoia anteriores esencialmente la misma gue la del problema 1 sobre ta tsta de los caba-llg-ros y los Bribones, aunque pueden parecer diferentes. ras ap"reniesdiferencias son: 1) En la paradoji anterior, el profesor formulo u" eriu"cia¿ode "si y sólo si", mientras que en el problemi 1, el nativo no lo hizo; él dijodirectamente que-el razonador nunca creería correctamente gue eínativoes un caballero. 2) En l.a-pandoja anterior, el alumno le cree al profásor,mientras que en el problema l, el razonador inicialmente no crée oue elnativo es un caballero. sin embargo, esas dos diferencias en cierto sentidose anulan mutuamente, como veremos ahora. La clave de esto es el métodode traducción del capítulo 7.

vi¡tualmente en todos los probremas que siguen, vamos a tratar sólocon dos individuos: el n-ativo de la isla qüe formulá el enunciado, y elrazonador que escucha el enunciado. vambs a usar continuamente la l-etrak para la proposición de que el nativo en cuestión es un caballero. Entonces,como vimos en el capíh¡lo ! siempre que el naüvo afirma una proposiciónq, la proposición leq es verdadera. Ahóra bien, el razon ador cree las reglasde..la isla !c1ee .que los caballeros formulan enunciados verdaderos y"lcsbribones, falsos) y nosotros suponemos que oye todos los enunciadoJ quele formulan. Por lo tanto, siempre que el narivo le afirma.rn" prolorio'ár,q,al razonador, el razonador ireelá proposición k=q. En ,eaü¿aá, des¿eahora, cuandoSe diga quevalenlasregras'de la isra, baü q".oorij.riñq"",para cualquier proposición q, si el naüvo afirma q, entonóes h prolosiáOnk;c9s verdadera. y cuando.se diga qrre er razonador cree las'reg'ias Je hisla (y oye todos los enunciados dirigicios a él) basta que eso signifilue, paracualquier proposición q, si el nativo re afirma q ar iazonadoí, enüncü elrazonador cree la proposición k=q.

. Para toda proposición p, sr¡p€ngamos que Bp es la proposición de queel nzonador cree (o creerá) p.-y supongamos que Cp ejta proposici'ánp&Bp. Interpretamos cp como .El rázonado r cree correctarnsnte o,,.,..-.

Ahg." bien, enel problema 1, el naüvo afirmó la proposicién _Ck(-usreo nunca creerá correctamente que soy un caballero,'). Dado que lasreglas de la isla son válidas, entonces la proposición le-ci e, ,re.dldl.a.Y como el razonador cree las_reglas de laisla, entonces cree la proposiciónk=-ck. Estos dos hechos resultan ser lógicamenüe incompatibrei; y'""ii".!.una paradoja.

- En el problema 3, sea g la proposición que Dios exisre. El profesor

fyO$re.ctamenre la proposicién g=-Cg. Según el supuesto ae que el pio_Iesor solo tormula enunciados verdaderos, la proposición gF_Cd debe- serverdadera. Dado que el alumno le creyó al profesor, entonces créyó la pro-posición F:Cg. Nuevamente, la verdad dé g-Cg resultó ser tógicaménteincompatible con el hechode que el alumno crea!-Cg (ya qr"ét

"t r*rro

es un razonador de tipo 1).Ahora vemos exactamente lo que las dos paradojas tienen en común;

en ambos casos tenemos una proposición p (que es k, para el problema1, y g para el problema 3) tal que p=-Cp es vérdadera y el iazonadbr h cree;

t,notraspalabras,elrazonadorlacree correctamente,yest()(:slógicamenteinrposible siel razonadores de tipo L. Porlo tanto ambas para<loias (omejorrlicho sus resoluciones en cuanto a que las condiciones dad:rs son lógica-rnente incompatibles) son casos especiales del siguiente tcorcrrra.

Teorema A. No existe ninguna proposición p tal que un razorr:rtlor detipo 1 pueda creer correctamente la proposición pE-Cp. En otra¡i ¡r:rl:rlrras,rro hay ninguna proposición tal que un razonador de üpo 1 pucl<l;r clr:cr(:orrectamente que: "La proposición es verdadera si y sólo si nr¡ t rt'ocorrectamente que es verdadera".

La demostración del Teorema A es poco más que una repetición de l, r:,rlos casos especiales ya considerados, pero puede servir de ayuda ¡r:rr,rr:onsiderado en un marco más general y p ra señalar algunas de sr¡si nteresantes características.

Por empezat, par^ toda proposición p y g, la proposicifrrr(e-(p&q))>q es una tautología (como puede verificar el lector). lirrparticular, la proposición (p=-(p&Bp)bp es una tautología. Supongamosque Cp es la proposición p&Bp, y por 1o tanto (pe-Cp)>p es una tautología.Ahora supongamos que un razonador de tipo 1 cree correctamente p=-Cp;cntonces obteoemos la siguiente cont¡adicción: Dado que el razonadorcree colnectctmente F-Cp, entonces p=-Cp debe ser verdadera. Además(p=-Cp)>p es verdadera (es una tautología), y por lo tanto p debe serverdadera. Ahora bien, dado que el razonador es de tipo 1, cree la tautolo-gía (tr-Cp)=p y también cree p-Cp (por hipótesis), y dado que es de ü-po 1, entonces creerá p. Y por lo tanto p es verdadera, y él cree p, enconsecuencia cree correctamente p. Así Cp es verdadera; y entonces -Cpes falsa. Pero dado que p"es verdadera y -Cp es falsa, no puede ser queF-Cp (dado que una proposición vcrdadera no puede ser equivalente auna proposición falsa), y obtenemos una contradicción a partir del supues-to de que el razonador de tipo 1 crcc correctamente p=-Cp.

También existe el siguiente "dual" dcl Teorema A cuya demostracióndejamos a c rgo del lector.

TeoremaA". No existe ninguna ¡rro¡rosición p tal que un razonador detipo 1 pueda creer correctamente f>¿:O(-p).

Ejercicio -1. Demostrar el Teorcr¡r:r A".

Ejerclcío2. Supongamos que l) c:s rrrrr operación cualquiera arbitraria,que asigna a cada proposición p cicrtx lir'()posición Bp. (No es necesarioespecificar qué es la proposición R¡>; ('n (:slc capítulo, hemos supuesto queBp es la proposición que el razona(lor ¡:r'rrr I); en un capínrlo posterior, enel que analizaremos sistemas matemátitr )s ('n vez de razonadores, Bp serála proposición que p es demostrab,/c trrr t'l sistcma. Pero por ahora, Bp noserá especificada.) Suponemos que (i¡r cs l;r ¡rroposición (p&Bp).

73

Page 39: SMULLYAN, R. Juegos por siempre misteriosos. España. Gedisa 2008

a) Demostrar que se puede inferir una contradicciÓn lígica a partir delos siguientes supuestos:

i) Todas las proposiciones de la forma BX, donde X es una tautología.ii) Todas las proposiciones de la forma (Bx&BOb)r)rBY.iii) Cierta proposición de la forma C(f-Cp).b) Demostrar que también obtenemos una contradicción lógica si

reemplazamos iii) por "Cierta proposiciÓn de la forma C(p=C-p)'.C)¿por qué el Teorema A es un caso especial del punto a)? ¿Por qué el

Teorema A" es un caso especial de b)?

10

El problemr,t se Profundiza

RAZONADOIIIiS I'IIIiSTJMIDOS

Volvemos a la Isla de losCaballt'rt,s y lr ¡:, Ilrilxrncs. Ahora supongarrro'sque el nativO, en vez de deCir: "lj¡itctl lrrrrr, ¡ r lr't'rll ('orrectamente quc s()y

uncaballero",formulaelsiguientcc:ttr trrt | t ' l ' t " I I t I t Inuncacreeráqucsrty

un caballe¡o".El naüvo ha omiüdo la palabra "t ilrrt'r l,r||r|ttlr"" y como resultado las

cosas se pondrán mucho más intt:tr'¡¡lrl.:i tr.,Hil|lil()s suponiendo quc cl

nat ivosedir igeaunfazonadordc t l ¡ r r r I r ¡ i i le=tr ' r ( ' ( ' lasreglasdelais la (y

también le hi oldo el enunclado) y t¡rrc l,re tr.gl.rr rk' la isla son realmcntc

válidas. Y ahora haremos la hip(ltr:sh arll¡ lilrt.rl rlr' <¡ue el razonador cs

siempre correcto en sus juicios; n() ('lef! t llt lgr | | r'l 1,r o¡xrsición falsa. ¿obtc-nemos todavia una Paradoja?

Bueno, supongamos que el nativr I ce r ¡ i i I r¡ ll ri rr r I i t rtonces su enunciado

es falSo,lO que SignifiCa que el raZOnatk lt { tFFt 4 r I r.= r': t¡n caballerO. YCOmO

el razonador eS Siempre COrregto en $tli llrlr lt ¡e, et tllr t¡ r's el nativo realmente

es un caballero. De eSte modO el fiu¡)tlciátr t t le ' ¡t

ir, rl nativO eS un bribÓn

lleva a una contradicciÓn; por lo tantr¡ el ttglli," rlr'lx' sef un caballero.

Ahora bien, el razonador es de tl¡ro I y irlc trrrt'r lógica como ustedes

yyo. ¿Quéle impideseguiradelante c(ttl el nrlártrr ¡ ¡'r 'u lso derazonamiento

que acábamos de rcalizar y llegar a la ule¡ttg t !'rrt lrr"ir)n' es decir, que el

nativo debe ser un caballero? por lo tanl0 el t4*r ¡rr*,l, n creeráqueel nativo

es un caballero, lo cual convierte en f*lert Fl et|trtt, l:rdo del nativo, en

consecuencia el nativo debe serunbril¡f¡ll Fettr ¡a lr¡'rr¡os demostrado que

el nativo es un caballero. ¡Paradojal

I

El argumento anterior es falaz. ¿Pua& Él lettil¡ 'tesc:ubrir la falacia?

@ista: A pesar del hecho de que el razonedof E*tf¡ÉJFF !a lrr¡4ica proposicionaltan bien como ustedes Y Yo, hay al¡¡er QÉÉ ilssiriila sabemos que elr zonadoÍ no sabe. ¿Qué es?)

74/>

Page 40: SMULLYAN, R. Juegos por siempre misteriosos. España. Gedisa 2008

Soluclón.lns dije que el razonador es siempre correcto; nunca dije queél sablaqveera exacto, siempre correcto. Si él lo supiera 0o cual en realidadno puedé saber), tendríamos una paradoja. Como se ve, parte de nuest¡ademosuación de que el nativo es un caballero utilizÓ el supuesto de queel razonador es siempre correcto; si el razonador formulÓ el mismo su-puesto, entonces también podría demostrar que el naüvo es un caballero,v asÍ converurÍa al nativo en bribón.

Eliminernos ahora la hipótesis de que el razonador es siempre correctoen sus juicios, pero supongamos que el razonadot cree que siempre escorrecto. Entonces para toda proposición p, el razonador cree que si llegaraa creer p, entonces p debe serverdadera. Decimos que un razoiador deesa clasé se denomina razonador presurnldo. Por lo tanto un razonadororesumido es el que cree que es incapaz de creer cualquier proposiciónialsa. Entonces, ¿obtenemos una paradoja? No, pero en su lugar tenemosel siguiente resultado más interesante.

Teorema /. Supongamos que un nativo de una isla de caballeros ybribones le dice a un razonador del tipo 1: "Usted nunca creerá que soyun caballero". Entonces si el razonador cree que siempre es correcto, caeráen una incorrección; es decir, tarde o temprano creerá algo falso.

2

Demosüar el Teorema I.

Soluctón. El razonador r^zon i "supongamos que es un bribÓn.Entonces su enunciado es falso, lo cual significa que creeré que es uncaballero. Pero si llego a creer que es un caballero, realmente debe sedo,porque no soy capaz de cometer errores [sic]. Así, si es un bribÓn es uniabailero, lo cual no es posible. Por lo tanto no es un bribón y es un ca-ballero".

En este punto el razonador cree que el naüvo es un caballero. Dadoque el naüvo dijo que el razonador nunca lo crée{ra, entonces el nativoefectivamente es un bribón. Y entonces ahora el nzonador tiene la falsacreencia de que el naüvo es un caballero.

Lo interesante es que si el razonador hubiera sido más modesto y nohubiera supuesto su propia infalibilidad, nunca hubiera caído en laincor¡ección de creer que el naüvo es un caballero. ¡El razonador ha sidocastigado con jusücia por presumido!

RAZONADORES PECULIARES

Digamos que un ruzonador es cornecto respecto de una proposiciÓndada p si su creencia en p implica que p es verdadera; en otras palabras,

76

ocurre que no cree fl p, o si no, cree en p y p e¡ ver¡l¡rler ¡ ! )ircmos queel razonador es incorrecto respecto de p si cree en ¡r y ¡r r',: l:rlsa.

Con respecto al.último problema resulta un her,lu¡ ¡rlt¡lrlt: que elrazonador fuera incorrecto con respecto a la mism¡ prolri !átr tt¡r¡ .sobre lacual creyó ser correcto: es decir, la proposición dc e¡ie el rr¡tivo es uncaballero. Al creer que era exacto con respecto a ega pf()lxrcir ¡f¡rr, final-mente llegó a creer que el nativo era un caballero, convlrtie rxh r r le , r.¡r modoal nativo enun bribón. Por supuesto nuest¡a demostraclón de r¡rrr. r.l rrat-ivoes un bribón se basó en nuestro supuesto de que las reglaa rlr, l¿ l.rl¡ cranválidas. Supongamos que eliminamos esta hipótesis pero sc¡li¡¡trr 15 rlrl)o-niendo que el razonador cree que las reglas de la isla son válldaa; ¿torl.rvíase sigue que el razonador creerá alguna proposición falsa? Por ntpt¡.fit( ) (:lrazonador aún seguirá creyendo que el naüvo es un caballero, ittltrr¡rrc r:lnaüvo dijo que nunca lo haría; pero si las reglas de la isla no son vllirl.¡r;,entonces el naüvo no es necesariamente un bribón. Sin embar¡¡o, ,rrrrrcuando nuestra demostración del Teorema I falle, si eliminamos la lrl¡rrrrr.sis de que las reglas de la isla son válidas, tenemos la siguiente prop<rglt.ifrrrmás sorprendente:

.Teorema.I/. Supongamos que un habitante de la isla le dice a rrn

razonador de tipo 1: 'Usted nunca creerá que soy un caballero'. Suporrgamos que el razonador crceque las rcglas de la isla son váüdas. Entoncc¡,independientemente de que las rcglas sean realmente válidas o no, si clrazonador es prezumido,llegará a crcer alguna proposición falsa.

.l

Con la hipótesis del Teorema tt, ¿<¡ué proposición falsa creerá elrazonador?

Solucíón.Segttn el mismo método clc (lc nlostración que seguimos parael Teorema I, el razonador llegerá a crclclr r¡ur: cl naüvo es un caballero. Estacreencia no es necesariamente falsa (daclo t¡uc las reglas de la isla no sonnecesariamente válidas), pero entonce¡¡ e I r¡zonador continf¡a: "Dado quees un caballero y dijo que yo nunca crccrl¡ (luc es un caballero, entonceslo que dijo debe ser verdadero, es declr, (Itc no creo (o sea, no es ciertoqueyo crea) que es un caballero. Por lo t¿¡nl() rr() (:reo que seaun caballero".

En este punto, el razonador cree que el nativo es un caballero y tam-bién cree que no cree que el nativo sea un c'all¡¡llcro. Por lo tanto üene lafalsa creencia de que no cree que el nativo re¡ r¡rr <:aballero (es falso, dadoque sícree que el nativo es un caballero).

La conclusión del anterior problemr ea rr¡lrrrcnte bastante rara. Elrazonador cree que el nativo es un caballero y larnllién cree que no creeque el nativo es un caballero. Ahora bien, crt(, rio lrrr¡rlica una inconsisten-cia lógica de parte del razonador, aunque l¡rthrtlrlrlerncnte sí implica unapeculiaridad psicológica. Decimos que un re:orrÉrftrr es pecullarsihay

77

Page 41: SMULLYAN, R. Juegos por siempre misteriosos. España. Gedisa 2008

cierta proposición p tal gue cree p y también cree que no cree p. Est¿condición implica por zupuesto que el razonador es incorrecto (porque creela proposición falsa de que no cree p). Y por lo tanto un razonador peculiares automáücamente incorrecto, pero no necesariamente inconsistente.

Decimos que un razon ador es ppullar con respecto a una propostctúndadasi cree p y también cree que no cree p. Entonces un razonador especuliar si y sólo si existe por lo menos una proposición p con respectoala Cj.tal es peculiar. Si un razonador es pecrrliar con respecto a p, enton-ces es incorrecto, no necesariamente con respecto a p, sino con respec-to a Bp.

Ahora observamos que aun si eliminamos el supuesto de que las reglasde la isla verdaderamente son válidas, si el razonador cree que se cumpleny es de üpo 1 y un nativo le dice que nunca creerá que el naüvo es uncaballero, entonces si el razonador cree que es correcto con respecto a laproposición de que el nativo es un caballero, su creencia lo obliga a serincorrecto con respecto a la proposición de que él crce que el nativo es uncaballero. Si las reglas de la isla realmente son válidas, entonces el rszona-dor también será incorrecto con respecto a la proposición de que el naüvoes un caballero: es decir, creerá que el nativo es un caballero, mientras queel naüvo realmente es un bribón.

Por supuesto este mismo problema puede formularse en el contextodel alumno y su profesor de teología. Supongamos que el alumno es unrazonador de üpo L y su profesor le dice: "Dios existe si y sólo si usted nun-ca creerá. que El existe". Supongamos también que el alumno cree que sillega a creer que Dios existe, entonces Dios sí existe. Esto significa: 1) Siel alumno no le cree al profesor, entonces terminará creyendo que Diosexiste y también creyendo que no cree que Dios existe. 2) Si el enunciadodel profesor también es verdadero, entonces Dios no eiste, pero el alumnocreerá que Dios sí existe.

Ambas versiones de este problema son casos especiales del siguienteteorema.

TeorcmaA.Supongamos que un razonador es de üpo 1 y que hay unaproposición p tal que él cree la proposición pE-Bp y también cree queBp>p. Entonces se sigue que:

a) Creerá p y también creerá que no cree p (será peculiar con respec-ro a p).

b) Si también F-Bp es verdadera, entonces p es falsa, pero creerá p.

Detnostracíón a) El cree F-Bp, entonces cree Bp>-p (que es unaconsecuencia lógica de e-Bp). También cree Bp>p (por hipótesis), enconsecuencia debe creer -Bp (que es una consecuencia lógica de Bpo-py Bp>p). Pero también cree p=-Bp, luego debe creer p (que es una corise-<r.¡encia lógica de -Bp y p=-Bp). Y por lo tanto cree p y también cree -Bp(cree que no cree p).

b) Supongamos t¿mbién que p=-Bp es verdadera. Dado que -Bp es

1t}

lalsa (sfcree p), entonces p, al ser equivalente a la proposición falsa -Bp,también es falsa. Por lo tanto cree p, pero p es falsa.

EJercfclo.Supngamos que un nativo le dice a un razonador presumidode tipo 1: 'Usted creerá que soy un bribón". Demostrar: a) Si el razonadorcree que las reglas de la isla son válidas, entonces creerá que el nativo esun bribón y ambién que no cree que el nativo es un bribón.

b) Si además las reglas de la isla son realmente válidas, entonces elnaüvo es efectivamente un caballero.

RAZONADORES DE TIPO 1*

Decimos que un razonador de ttpo -Z'es un razonador de tipo 1 conla propiedad agregada de que para proposicioncs cualesquiera p y q, sillega a creer la proposición prq, entonces creerá que si llega a creer p en-tonces también creerá q. En símbolos, si cree pf,q, cntonces también cree-rá Bo¡Bq.

bbr.h"-or qrre si un razonador de tipo 1 cree p:q, cntonces es ciertoque si llega a creer p, entonces creerá q: es decir, Bp:llq cs una proposi-ción verdadera (si cree prq). Lo que tiene un razonador dc tipo 1. que notiene un razonador de üpo 1 simplemente es que si cree prq, cntonces nosólo la proposición Bp>Bq es verdadera, sino que crce corrcctamenteBp=Bq. De este modo un razonador de tipo 1r tiene un poco rnhs de "auto-conciencia" que un razo¡ador que sólo es de üpo L.

Segrimos suponiendo que el razonador, que ahora es dc tiJxr 1., crcclas reglas de la isla y oye todos los enunciados que le formulan, y ¡xrr lt>tanto siempre que el naüvo afirme una proposición p, el razonatl()r (.rcola proposición lep, donde k es la pr<l¡losición de que el naüvr> cs r"rrrcaballero.

El siguiente hecho será bastante dcr:isivo:

Iemar 1. Supongamos que un nativo lt: li rrtru la un enunciado a un razo -nador de üpo 1'. Entonces el razonador t:rt:<:rh que si llega a creer quc clnativo es un caballero, también creerá kr r¡ut.rlijo el nativo.

Prcblema 4. ¿C6mo se demuestra cst(. lr.nrrr?

Solución. (En realidad, la demostrir( r()n (:s bastante simple.)Supongamos que el nativo le afirma la proJrosici(rr r ¡r al razonador. Entonceselrazonador cree la proposición lep. lintorr<r.s t.rr¡rlrién cree k=p, porque

r Un lerna es un¿ proposición que se demuestre ilr , t n trt r , ¡ ', ,r ri rnisma sino como awdapara demostru teorerlas ulteriores. Se podría decir {¡rrt' rril l.r r rr , s rrna proposición que noüene el "suficiente prestigio" para denominarse t(rxmr{ ll I

79

Page 42: SMULLYAN, R. Juegos por siempre misteriosos. España. Gedisa 2008

k>p es una consecuencia lógica de rep. gntonces dado que él es de tipo1', creerá BbBp.

He llamado e un razonador "presumido', si cree en su propia infa_libilidad. Dificilmente consideraríique una persona que cree que no especuliar actúa presumidamente; en realidad, zuponer qüe no se es peculiares perfectamente raLonable. Ni siquiera esto¡seguró de que sea psico-lógicamente posible que una persona sea peóliai. ¿lodría'efectirr

-errt"una persona creer algo y también creer que no lo cree? Lo dudo. sin embar-go n_o es lógicamente imposible que una persona sea peculiar.

De todos modos, confiar en la propia ño peculiaridid es mucho másrazonable que confiar en que uno es siempre correcto. por lo tanto elsiguiente teorema es un poco triste.

Teorema IIL Supongamos que un nativo le dice a un razonador de tioo1': 'Usted nunca crenrá que soy un caballero,,. Si el razonado. .r"" qrr. noes (y nunca será) peculiar, entonces se volverá peculiar.

Ptoblema 5 Demostrar el Teorema III.

soluctón La demostración de este teorema es un poco más elaboradaque cualquier otra hasta ahora.

Supongamos que el naüvo formula ese enunciado y que el razonadorcree que es incapaz de ser peculiar. Dado que el naiivo formuló elenunciado, _entonces según el l¿ma 1, er razonádor creerá que si llega acreer que el naüvo es un caballero, también creerá lo que di¡o el naüvo.Y por lo tanto el razonador razona: "Supongamos que ilégo a i"", q.r" .,un caballero. Entonces creeré lo que diie, ei decir, ir""riq.r"

"o oJo t r"

es_un caballero. Y por lo tanto enionces creeré que es rrn caball".o y t"--bién.creeré que no creo que sea un caballero. Esto significa quá serépeculiar. Por lo tanto, si llego a cteer que es un cabalÉro,

-"'"ál""iepeculiar. Dado que nunca seré peculiarisicl, entonces nunca creeré quees un caballero. Dado que él dijo que no lo creería, su enunciado

"r" '.rlr-

daclero, y por lo tanto es un caballero.,En este_punto el razonador ha llegado a la conclusión de que el nativo

:::l,:Piryro, y un poco anres llegó a la conclusión de que .ro

"."" qrl.

et natrvo sea un caballero. por lO tanto ha caído en peculiaridad.. Por supu-esto la citada proposición puede enunciárse y demostrarse dela siguiente forma más generáI,

Teorema B. Paru todo razonador de tipo f, si cree cualquierproposición de la forma p=-Bp ("p es verdadera si y sólo si nunca .r'oy acreer p"), entonce$ no puede creei que no es peculiár,

".""á, qu. .áíg"

cn la peculiaridad.

tt( )

MoraleJa. Si ustedes son razonadores de tipo 1r y desean creer que noson peculiares, pueden evitar volverse peculiares si simplemente se nieganx creer toda proposición de la forma "p si y sólo si nunca voy a creer p".

En particular, si alguna vez visitan la Isla de los Caballeros y los Bri-l>ones (o lo que les han dicho que es una isla de caballeros y bribones) yun nativo les dice que usteden nunca creerán que es un caballero, enton-ces la conducta más inteligente es negarse a creer que las reglas de la isla.son válidas.

Sin embargo, más adelante en este libro, cuando llegrremos al esh¡diode los sistemas matemáticos y hablemos sobre demostrabilidad en elsistema en vez de creencias de un razonador, veremos que la opciÓnanáIoga a no creer F-Bp no estará disponiblc.

81

Page 43: SMULLYAN, R. Juegos por siempre misteriosos. España. Gedisa 2008

Parte V

LA COMPLICADAPROBLEMATICA DE LA

s

CONSISTENCIA

Page 44: SMULLYAN, R. Juegos por siempre misteriosos. España. Gedisa 2008

1"1

Los lógicos que ra,zonan sobre sí mismosNos estamos acercando a la compücada problemáüca de Gó'del sobre

la consistencia. Pero primero, tenemos que considerar razonadores queposean niveles zuperiores de auto.conocimiento que los de tipo 1 simple-mente.

ETAPAS PROGRESIVAS DE AUTOCONCIENCIA

Ahgra definiremos razonadores de üpos 2, 3 y 4, que representanniveles*de autoconocimiento ceda rre, ttrayores. fos razonadoÉs de üpo4 desempeñan un papel importante en los dramas que se desarrollaráh.

Razonadates de tlpo 2. Supongamos que un razonador de üpo 1 creep y cree pnq. Entonces creerá q. Esto significa que la proposición(Bp&B(p=q)bBqes uudadra para un razonador de tipo 1. Sin embargo,el razonador ¡o sabe necesariamente que esa proposición es verdadera.Bueno, decimos lJue un razonador es de üpo 2 si es de üpo 1 y cree toüslas proposiciones de la forma (Bp&B(poq))=Bq. (¡t "sabe" que su coniuntode creencias -pasado, presente y futuro- es cerrado para el ttoduspotutts.Pera toda proposición p y q, cree: 'Si llego a creer p y a creer pDq,entonces tembién creeré q".)

Destaquemos que los razonadores de tipo 2 üenen cierto oautocono-cimiento' que no está necesariamente presente en los razonadores de üpo1. Un razonador de üpo I que cree p y cree poq tarde o temprano creeráq; los razonadores de üpo 2 ambién saben que si llegan a creer p y a creerp>q, creerán q.

Razonadorcs d" Up 3. Decimos que un razonador es normal si pe-ra toda proposición p, si cree p, entonce.s cree que cree p. (Si cree p,entonces ambién cree Bp.) Decimos que un razonador de üpo 3 es unrazontdor normal de üpo 2.

Los razonadores de tipo 3 tienen un nivel más de autoconocimientoque los de tipo 2.

Razonadotes defl&4. Un razonador r¡ormal no sabe necesariamenteque es normal. Si un razonador es normal, entonces para toda proposiciónp, la proposición Bp>BBp es uerdafum (si cree p, entonces cree Bp), pe-ro el razonador no conoce necesariamcntc la verdad de BpoBBp. Pues

85

Page 45: SMULLYAN, R. Juegos por siempre misteriosos. España. Gedisa 2008

bien, decimos lJue un razontdor cree que es normal si para todaproposición p, cr,99_l_a proposición BpoBBp. @ara toda proposición p, elrazonador cfee: nSi llego a creer p, entonces creeré que creb p,.)

- Un.razonador de üpo 3 de hecho es normal. Decimos que un razo-

nador de tipo 4 es un razonador de üpo 3 que sabe que es normal. De esteTodg para toda proposición p, un razonador detipo 4 cree la proposiciónBp=BBp.

Como ya hemos comentado, los razonadores de tipo 4 desempeñanun_papel importante en este libro. Ahora repasemos lai condicionés quedefinen avnrazonador de üpo 4.

la) Cree todas las tautologlas.lb) Si cree p y cree prq, entonces cree q.2) Cree (Bp&B(p>q))>Bq.3) Si cree p, entonces cree Bp.4) Cree Bp>BBp.

ALGUNAS PROPIEDADES BASICASDE LOS RAZONADORES AUTOCONSCIENTES

Ahora vamos a establecer algunas propiedades básicas de los razo_nadores de los üpos 2,3 y 4 que se utilizarán en el curso de los capítulosrestantes.

En primer lugar, una sencilla observación sobre los razonadores de9po_ _2:. Para proposiciones cualesquiera p y q, la proposición(Bp&B(p_>q)):Bq es lógicamente equivalénre a la pioposiciónS(p>q)?fPpr-Bq) porqug para proposiciones cualcsquíerax, y y Z,laproposición CX&\trZ es lógicamente equivalente a y;GrZ), como pue_de verificar el lector muy fácilmente, y por lo tanto todo razonador de tipo2 cree todas las proposiciones de la forma B(p>q)>(Bp>Bq). A la inveria,todo razonador de tipo 1 que cree todas las proposicionés de la formaB(p>q):(Bp>Bq) debe ser de tipo 2. Registremod esto como Hecho 1.

Hecho 1.Un razonador de tipo 1 es de tipo 2 si y sólo si cree rodas lasproposiciones de la forma B(p>q)>(Bp>Bq).

Ahora srrpongamos que un razonador de tipo 2 cree B(pq).Tambiéncree B(p>q)>(Bp>Bq), según el Hecho 1, y al ser de tipo i (da¿o que es99 tipg 2) entonces creerá. Bp>Bq -que es una consécuencia lógica deS(p)q) y B(p>q)>(BprBq)-. y así como consecuencia obvia deiHechcr1 tenemos el siguiente corolario: Si un razonador de üpo 2 cree B(poq),entonces creerá Bp>Bq.

Ahora llegamos a ciertos hechos menos evidentes sobre los tazona-dores de üpo 2.

T

Demostrar que para proposic:iottcs cualesquiera p, q y r, cualquielrazonador de tipo 2:

a) Creerl B(p>(q>r)>(Bp>(l)<l I llr)).b) Si llega a creer B(p:(q:r)), orrtonces creerá Bp>(BqrBr).

2

Demostrar que si un razonador tlt' ti¡r<> J 6¡6s p>(q>r), entonces creerlBp>(Bq>Br). Este hecho tendrá r¡lrrr'lras aplicaciones.

J. Ilctptltrridad

En el capítulo anterior, definirrlos ( lu(: un razonador es de üpo 1' si esde tipo 1 y si para proposicionc$ t'tt:tlt'st¡tticra p y q si llega a creer p.)q,también creerá Bp>Bq. Esta scf{trrt(l.r trlndiciÓn recibe un nombre.Decirgos que un razonador es rc¡uktr si su creencia en prq implica sucreencia en Bp=Bq.

.Demostrar que todo razonad<n'rlr li¡ro 3 es regular (y por lo tanto esde üpo 1').

,l

Demostrar que si un razona<k¡t tr'¡r,rtl.tt clc tipo 1 cree peq, entoncescreerá BpBq.

5

Existe una interesante conexi(rrt et¡ltc l.t rcgularidad y la normalidad.Para un razonador regular de üpo l, t t ttt t ¡t tr: haya una proposiciÓn q talque él crea Bq, entonces debe ser t¡or rtt.tl ¿l'or qué?

Cualquier razonador de üpo 1 <¡tte lt rr' ¡ ) y cree q cneerá p&q Qo cuales una consecuencia lógica de las dtl¡ ¡rt I r¡ x rsit:iones p y q). Por lo tantola proposición (Bp&Bq):B(p&q) es uuttl¡t¡h'nt ¡rara todo razonador de ti-po 1, y entonces es verdadera para t(¡(ht r¡?ottador de tipo 3.

Demostrar que todo razonadot cle tt¡xr 't r;ree(Bp&Bq)>B(p&q). CEIsabe que si llegara a creer p y a creer r¡, r rr'crl p,&q.)

CONSIS' I ' I iN(: IA

Decimos que un razonador es connlal(lttl.' ri cl conjunto de todas las

87

Page 46: SMULLYAN, R. Juegos por siempre misteriosos. España. Gedisa 2008

proposiciones que cree (o ha creído o creerá) es un conjunto consistentey decimos que es inconsistente si su conjunto de creencias es inconsistente.Pan todo rezonador de üpo 1, el conjunto de sus creencias es lógicamentecerrado; en consecuencia. se sigu.e a partir del principio c del capínrlo g quelas tres condiciones siguientes son equivalentes:

1) Es inconsistente (qee f).2) Cree cierta proposición p y su negación (-p).3) Cree todas las proposiciones.Decimos que un razonador crce que es consistente si cree -Bl_ (cree

que no cree l-). Decimos que cree que es inconsistente si cree que Bl_ (creeque cree J-). Aun un razonador de tipo 1 que sea inconsistente tambiéncreerá que es inconsistente (porque creerá todo), aunque un razonadorque cree que es inconsistente no sea necesariamente inconsistente(aunque puede demostrarse que por 1o menos debe tener una creenciafalsa).

Todo razonador de tipo 1 que cree cierta proposición p y su negación-p será inconsisrente: oeerá l_; y por 1o tanto la proposición (Bp&B-p):gIes uerdadera para un razonador de tipo 1.

7

__ ^D_emgsqar que todo razonador de üpo 3 cree la proposición

(Bp&B-p)>ga.

Notaj Este último problema es bastante decisivo para el próximocapítulo..Significa que para todq proposición p, un razonádor de tipo 3 sa_be que si llega a creer p y también a creer -p, entonces será inconsisten-te. (Por supuesto esto también se aplica a un rizonador de tipo 4, dado quetodo razonador de üpo 4 es también de tipo 3.)

Inconslstencla- y pecullañdad Recordemos que se dice que unrazonador es peculiar si cree cierta proposición p y también cre. que nocree p. Ya dijimos que un razonador peculiar no es necesariamente incon-sistente. Sin embargo todo tazonador peculiar de tipo 3 es inconsistente,como revelará el siguiente problema.

I

. Demostrar que todo razonador normal peculiar de üpo 1 debe serinconsistente (y en consecuencia todo razooádor peculiar ie tipo 3 debeser inconsisüente).

fJerclcfo,!. Según el problema anterior, para toda proposición p, laproposición (Bp&B-Bp)>B Les uerd.adera para un razoiad,ór de tipo I j enconsecuencia también es verdadera para un razonador de üpo 4. bemos-!!er que todo razonador de tipo 4 cree correctamente li proposición(Bp&B-Bp):BI (sabe que si llega a ser peculiar, será inconjisrente).

88

9. Sencíllo acertijo

Supongamos que un razonador de tipo 4 cree peBq. ¿Creerá necesa-riamente p>Bp?

CONCIENCIA DE LA AUTOCONCIENCIA

Los razonadores de tipo 4 poseen una mataüllosa propiedad que nocomparten los razonadores de tipos inferiores: saben qveson de tipo 4, enun senüdo que vamos a definir con precisión. Así, por ejemplo, un ra-zonador puede ser de üpo 3 sin saberlo, pero un razonador no puede serde tipo 4 a menos que lo sepa.

Decimosqueun razonador cree qtees de üpo 1 si cree todas las propo-siciones de la forma BX, donde X es una tautología, y cree todas tasproposiciones de la forma (Bp&B(pq)>Bq. Si también cree todas las pro-posiciones de la forma B(Bp&B(pq))rBq), entonces decimos que creeque ee de tipo 2. Si también cree todas las proposiciones de la formaBp>BBp, entonces decimos quecree que es de üpo 3. Si también cree todaslas proposiciones de la forma B(Bp>BBp), entonces decimos que cree quees de üpo 4.Pua cada uno de estos tipos, decimos que un razonador sábeque es de ese tipo si cree que es de ese tipo y realmente es de ese üpo.

No rezult¿ dificil advertir que un nzonador que sabe que es de tipoL es de tipo 2, y que todo razonador de tipo 3 sabe que es de üpo 2 (aunqueno sabe necesariamente que es de tipo 3). Además un razonaáor es de üpo4 si y sólo si sabe que es de tipo 3. El lector debe tratar de demost¡ar esioshechos como ejercicios.

El siguiente problema es más interesante.

10

Demostrar que un razonador de üpo 4 sabe que es de üpo 4.

El problema es interesante por varias razones. por un lado, demuestraque el hecho de ser de tipo 4 constinrye el último hito narural en nuestrajerarqula de razonadores (no tendría senüdo, por ejemplo, decir que unrazonador es de tipo 5 si es un razonador de üpo 4 que sabe que es de tipo4, dado que todo razonador de tipo 4 ya sabe que es de tipo 4, y por-lotanto no lograríamos nada nuevo).

En segundo lugar, todo lo que ustedes o yo podamos demostrar sobretodos los razonadores de tipo 4 usando simplemente la lógica proposi-cional, todo razonador de üpo 4 puede demostrarlo sobre sí mismo, dadoque también conoce la lógica proposicional y sabe que es de tipo 4.

EJercicto 2. Deck que un razonador es regular es decir que paraproposiciones cualesquiera p y q la proposición B(p>q)>B(Bp>Bq) esverdadera sobre el razonador. (La proposición B(poq):B(Bp:Bq) es la

89

Page 47: SMULLYAN, R. Juegos por siempre misteriosos. España. Gedisa 2008

proposición de que si el razonador cree prq,,entonces creerá Bp:Bq')b".i-ot que urr- razonador cree que es regular si cree todas las pro-

posiciones de la forma B(p>q):B(Bp:Bq).Demostrar que todo rizoÁadot deüpo 4 sabe que es regular (es decir'

es regular y cree que es regular).

Ejercicío 3. Demostrar que si un razonadot de tipo 4 cree p>(Bp>q),entorices creerá Bp>Bq. (La iolución de este ejercicio se dará en el capítulo'1.5; véasela página 113.)

SOLUCIONES

1. Supongamos que el razonador es de tipo 2' Consideremosproposiciones cualesquiera p, q y r.

Según el Hecho 1, él cree lo siguiente: 1)- B(p:(q:r])=(Bp>B(q>r))La ralón es que para proposiciones cualesquiera X e Y'

-él qtt"

sdbD:(sxrrirl, ño.lo tanio reemplazamos X por p e Y por (p>q)'

Nuevamente, según el Hecho 1, él cree: 2) B(Fr):(Bg:!r)La siguiente proposiciÓn es una consecuencia lÓgica de 2):

l) (Bp>B(q:r)):(Bpr(Bq>Br)). La raz6n es que' para proposicionescualesquiéra X,Y y Z, ú proposiciÓn Obl)=CK:Z) es una consecuencialógica áe Y>2, como puede verificar el lector' Reemplazamos X por Bp,

V por B(q>r) y Z por Bq>Br, y vemos que 3) es una consecuencia lÓgica

de 2).Ahora sabemos que el razonador cree tanto 1) como 2) y

B(p>(q=r))>(Bp>(Bq>Br)) es una consecuencia lÓgica de 1) y 2); por lotanto el razonador la cree. Esto demuestra a).

b) Supongamos que el razonadot cree B(p>(q>r))'Segun a) también cree B(p>(q>r)>(Bp>(Bq>Br)); en consecuencia

creerá-Bp>(Bq=Br), dado que es una consecuencia lÓgica de las dosproposiciones anteriores.

NotL: En adelante no daré argumentos tan detdllados. Ahora el lectorya defu.ria tener suficiente experiencia para entender argumentos másresumidos y completar los pasos que falten.

2. Ahora consideremos un razonador de tipo 3 que cree p>(q>r)' Dadoque es normal creerá B(p=(q>r)). Entonces según b) del problema anteriorcreerá Bp:(Bq:Br).

3" Supongamos que un razonador de üpo 3 cree p:q' Dado- que esrrrrmal, e.rto.tces creérá B(p>q). Entonces, según el corolario del HechoI, < rc<:rá Bp:Bq. Esto demuestra que es regular'

4. Srrpongamos que un razo¡ador regular de tipo 1 cree p=q' Entoncest.rr.r.rir t:tnto prq como g>p (dado que ambas son conseoencias lÓgicastle' ¡=r1). Al

'sei regular, éntonces creerá tanto Bp:Bq como Bq>Bp'

()( )

Entonces, como es de tipo 1, creerá Bp=Bq (que es una consecr.¡encia lÓ-gica de las dos últimas proposiciones).

5. Consideremos un razonador regular de üpo 1. Supongamos que qes cierta proposición tal que el razonador cree Bq. Ahora, supongamos quep es cualquiér proposición que cree el razonador. Tenemos que demostrarque creerá Bp.'

La propósición p:(q:p) es una tautología (c9mo el lector puedeverificai); en co.tsec,te.t cta él nzonador lo cree. También cree p (por hipÓ-tesis), en consecuencia creerá q>p. Entonces, dado que es regular, creeráBq:Bp. Entonces, dado que cree Bq, creerá Bp. Esto demuestra que esnormal.

6. Consideremos un razonador de tipo 3. Ahora bien, la proposiciÓnp>(q:(p&q)) es obviamente una tautoloSía; en consecuencia el razonadoria c.éeri. Entonces, según b) del problema 1, creerá Bpo(Bq>B(p&q)), enconsecuencia ueerá la proposiciÓn (Bp&Bq):I](p&q) lÓgicamente equi-valente. (Para toda proposiciÓn X, Y, Y Z, la proposiciÓn )bCl>Z) eslógicariente equivalente a CX&)O:Z).

7. La proposiciÓn (p&-p)>a es una tautoloSía, en consccuencia todorazonadoi de tipo 3 (o'au'de tipo 1) la creerá. Dado quc un razonadorde tipo 3 es regühr (según el problema l), entonces crecrá l|(p&-pbB*.También cree iBp&B-p)>s(p&-p) (según el problema 6). Al crccr cstasdos irltimas proposiciones, creerá (Bp&B-p)>Br, lo cual es una (:()nsc-cuencia lógica de las mismas.

Obseruactón Para toda proposiciÓn q, la proposición p:(-p>q) cs ttrtatautología. En consecuencia, según el argumento anterior aplicado a (l' cnvez de 1, un tazo.tador de tipo 3 creerá (Bp&B-p):Bq.

8. Esto es bastante obvio. Si un razonador normal cree p, creerá Bp.Si también cree -Bp y es de tipo 1, será inconsistente. Por lo tanto si unrazonador normal de üpo 1 cree p y cree -Bp será inconsistente'

Solución del eJerctclo 1. tJn razonador de tipo 4 creg-OppBg. f91-lotanto cree su consecuencia lÓgica (Bp&B-Bp)>(BBp&B-Bp). Tambiéncree (BBp&B-Bp)>Br (según el problema 7, dado que -Bp eslanegaciÓnde Bp). La proposiciÓn (Bp&B-p):Bl_ es una consecuencla IOStca Oe lasdos últimai próposiciones, y por lo tanto el razonador la creetl. En otraspalabras, un raionador de üpo 4 puede razonar así: "Supongamos queil.go

" cie.t p y también creo -Bp. bado que creeré p, también creeré Bp,

en-co.tsecueñcia creeré tanto Bp como -Bp, y entonces seré inconsistente.Y por lo tanto, si llego a creer p y -Bp, seré inconsistente'

9. Supongamos que un razonador de tipo 4 cree p=Bq' Es regular(según el'proÉlema 3, ya que también es de üpo 3), y por lo tanto según

9r

Page 48: SMULLYAN, R. Juegos por siempre misteriosos. España. Gedisa 2008

el problema 3 ueerá Bp=BBq. En consecuencia creerá BBq=Bp. Tambiéncree Bq>BBq (dado que es de tipo 4), de donde, según la lógicaproposicional, creerá BqrBp. A partir de pBq y de Bq>Bp, deduciráp>Bp. Por lo tanto la respuesta es sí.

L0. Supongamos que el razonador es de üpo 4. Entonces, cumple contodas las corrdiciones que definen avn razonador de tipo 4. Tenemos quedemostrar que cree todas esas condiciones.

1a) Consideremos cualquier tautología X. Dado que es de tipo 4 (y enconsecuencia de tipo 1), cree X. Entonces dado que es normal, cree BX.Por lo tanto, para toda tautología X, cree BX.

lb) A partir del hecho de que es de tipo 2 se sigue que cree todas lasproposiciones de la forma (Bp&B(Rre))=nq.

En este punto advertimos que sabe que es de tipo 1. (En realidad,según el argumento anterior, todo razonador normal de tipo 2 --es decir,todo razonador de üpo 3- sabe que es de ¡ipo 1.)

2) Dado que cree todas las proposiciones de la forma(Bp&B(p>q)!Bq, y es normal, entof\ces cree B((Bp&B(p¡q))>Bq).

En este punto vemos que sabe que es de tipo 2. (En realidad, todorazonador de tipo 3 sabe que es de tipo 2.)

3) Dado que el razonador es de tipo 4, entonces resulta inmediato queconoce todas las proposiciones de la forma Bp>BBp -es decir, sabe quees normal.

En este punto observamos que un razonador de üpo 4 sabe que es detipo 3. (Pero unrazonador de tipo 3 no sabe necesariamente que es de tipo3, porque puede no saber que es no¡mal.)

4) Dado que el razonador de tipo 4 cree Bp>BBp, y es normal, creeB(Bp>BBp).

Ahora vemos que un razonador de tipo 4 sabe que es de tipo 4 (esdecir, conoce todas las proposiciones que caracterizana un razonador deripo 4).

Sohtción del e1'erclclo 2. Consideremos un razonador de tipo 4. Dadoque es de tipo 1, cree B(p>q)>(Bp>Bq). Dado qüe es regular, entoncescree BB(p>q)>B(Bp>Bq). También cree B(p>q)>BB(p:q) ya que sabeque es normal. A partir de esto y del último hecho, se sigue que debe creerB(p>q)>B(Bp>Bq).

1,2

La sorprendente problernática de laconsistencia

Ahora el escenario está listo, y empieza el espectáculo en serio.Un razonador de üpo 4 visita la Isla de los Caballeros y los Bribones

y cree las reglas de la isla. Y así siempre que un naüvo formule unenunciado, el razonador creerá que si el naüvo es un caballero, el enun-ciado es verdadero y viceversa. De este modo, si un nativo afirma unaproposición p, entonces el razonador creerá lop (donde k es la propo-sición_que el nativo es un caballero), y también creerá prk. ¡dem¿i dádoque el rezonador es de üpo 4, es regular (como demostramos en elproblema 3 del último capíhrlo), y por lo tanto si el nativo afirma p, elrazonador creerá no sólo k>p, sino tembién BkoBp; es decir, creerá: "Sillego a creer que es un caballero, entonces creeré lo que dijo".

Regordgmos que en el último capltulo (problema 7) dijimos que todorazonedor de tipo 4, o aun de üpo l, sabe que si llega a cieer p f a creer-p, será inconsistente (p puede ser cualquier proposición).

Repasemos y registremos estos hechos.

Hecho 1. Supongamos que r¡n naüvo le formula un enunciado a unnzorudor de üpo 4. Entonces a) El razonador creerá que si el nativo es uncaballero, el enunciado debe ser verdadero (y a li inversa, que si elenunciado es verdadero, entonces el nativo debe ser un caballero). b) Elrazonador también creerá que si llega a creer que el naüvo es un caballero,entonces creerá lo que el naüvo dijo.

.. Hecho2.Par^todr proposición p,vnrazonador de üpo 4 sabe que sillega a creer p y también a creer -p, entonces será inco¡lsistente.

Con estos hechos en mente, estamos listos para embarcarnos. El pri-mer gran resultado al que llegamos es el siguiente teorema:

Teotetnc /. (Según el Teorema de la Consistencia de Gódel.) Supon-gamos que un nativo de la isla le dice a un razonador de tipo 4: 'Usted nun-ca creerá que soy un caballero'. Entonces si el razonador es consistent€,nunca puede sahr que es consistente; o, dictro de otr¿ forma, si el razona-dor llega a creer que no puede ser inconsistente, se volverá inconsistente.

9392

Page 49: SMULLYAN, R. Juegos por siempre misteriosos. España. Gedisa 2008

1

Demost¡ar el Teorema L

Sohrclón. Supongamos que el nzonador sl cree que es (y seguiúsiendo) consistente. Vamos a demostrar que se volverá inconsistente.

El razonador razonai 'Supongamos que llego a creer que el nativo esun caballero. Entonces creeré lo que dijo: creeré que no creo que es uncaballero. Pero además, si creo que es un caballero, entonces creeré quesí creo que es un caballero (dado que soy normal). Por lo tanto, si llegoa creer que es un caballero, entonces creeré tanto que sí creo que es uncaballero como que no creo que es un caballero, lo que significa que seréinconsistente. Ahora bien, nunca seré inconsistente [sic], en consecuencianunca creeré que es un caballero. El dijo que nunca creeria que es uncaballero, y lo que dijo era verdad; en conseclrencia es un caballero".

En este punto, el razonador cree que el nativo es un caballero, y dadoque es normal, entonces sabrá que cree esto. En conseoencia el razonadorcontinuará diciendg "Ahora creo que es un caballero. El dijo que yo nuncalo haría, en consecuencia formuló un enunciado falso; por lo tanto no esun caballero".

En este punto el razonador cree que el naüvo es un caballero y tam-bién cree que el nativo no es un caballero, por lo tanto ahora es in-consistente.

Análfsls. El importante contenido matemático del teorema citadopuede presentarse sin hacer referencie a caballeros y bribones. La funciónde la isla de caballeros y bribones fue proporcionar un método simple deobtener cierta proposición k (en este caso, que el naüvo es un caballero)tal que el razonador creyera "k es verdadera si y sólo si nunca voy a creerk". Cualquier otro método para Iograr tal proposición k serviría también.Por lo tanto el Teorema I sólo es un caso especial del siguiente teorema(que no üene nada que ver con caballeros y bribones).

Teorema G. Si un razonador consistente de tipo 4 cree cierfaproposición de la forma p=-Bp, entonces el razonador nunca puede saberque es consistente. Dicho de otra forma, si un razonador de tipo 4 creeF-Bp y cree que es (¡z seguirá siendo) consistente, entonces se volveráinconsistente.

Vamos a demostrar el Teorema G en forma más precisa.

Teoretna G' . Supongamos que un razonador normal de tipo L cree unaproposición de la forma p=-Bp. Entonces:

a) Si llega a creer p, se volverá inconsistente.b) Si es de tipo 4, entonces sabe que si llega a creer p, entonces se

volverá inconsistente; es decir, creerála proposición Bp>BI.

94

c) Si es de üpo 4y cree que no puede ser inconsistente, entonces sevolverá inconsistente.

Delnostraciórt a) Supongamos que cree p. Dado que es normal,entonces creerá Bp. Además, dado que cree p y cree p=-Bp, debe creer-Bp (dado que es de üpo 1). Creerá entonces tanto Bp como -Bp, y, enconsecuencia, será inconsistente.

b) Supongamos que es de tipo 4.Dado que es de tipo 1 y cree pe-Bp,también debe creer p)-Bp. Además es regular, en consecuencía creerientonces Bp>B-Bp. También cree Bp>BBp (dado que sabe que esnormal). Por lo tanto, creerá Bp:(BBp&B-Bp), lo que es una con-secuencia lógica de las dos últimas proposiciones. También cree(BBp&B-Bp)>BI (según el Hecho 2, dado que para toda proposición X,él cree (BX&B-)0=Bj-, y por lo tanto cree esto en el caso especial dondeX es Bp). Una vez que crea tanto Bp)(BBp&B-Bp) como(BBp&B-Bp)>BJ-, tendrá que creer Bp:B-L (dado que es de tipo 1).

c) Dado que cree Bp=B-l- (como acabamos de demostrar), entoncestambién cree -Bl:-Bp. Ahora, supongamos que cree -BI (cree que nopuede ser inconsistente). Dado que también cree -Bl-=-Bp (comoacabamos de ver), entonces creerá -Bp. Dado que tambiéen cree p=-Bp,creerá, p, en consecuencia será inconsistente según a).

El alumno y su prcfesor de teolagía. Ahora regresemos nuevamente alalumno y su profesor de teología que le dice:"Dios existe si y sólo si ustednunca va a creer que Dios existe". Si el alumno le cree al profesor, entoncescree la proposición F-Bg, donde g es la proposición Dios existe. Enton-ces, según el Teorema G, el alumno no puede creer en su propia consis-tencia sin volverse inconsistente.

Ya aludimos a este punto al final del capítulo 2, p.ro en ese momentono éramos capaces de enunciar 1o que corstifuía un conjunto "razonable"de supuestos sobre la capacidad de razonamiento del alumno. Ahorapodemos hacerlo. Las hipótesis simplemente son que el alumno es unrazo-nador de üpo 4.

Por supuesto el alumno tiene la opción de creer en su propiaconsistencia sin volverse incorxistente: simplemente puede negarse acreerle al profesor.

EJerctcio 7. Supongamos que en el Teorema I nos dan la informaciónadicional de que las reglas de la isla son realmente válidas. El nativo ¿es uncaballero o un bribón?

Ejercicio 2. En el ejemplo del alumno y zu profesor de teología,supongamos que el alumno sí le crcc al profesor y también cree en supropia consistencia. Si Dios realmcntc cxiste, entonces el enunciado delprofesor ¿era verdadero o falso? Si l)ios no existe, entonces el enunciadodel profesor ¿era verdadero o falso?

Respuesta al eJercíclo 1. Según cl 'lt:or<:ma I, el razonador será incon-

95

Page 50: SMULLYAN, R. Juegos por siempre misteriosos. España. Gedisa 2008

sistente, en consecl¡encia creerá todo. En parücular, creerá que el naüvoes un caballero. Dado que el nativo dijo que él no lo^creería, entonces elenunciado del nativo era falso. Por lo tanto, si las reglas de la isla son real-mente válidas, el naüvo debe ser un bribón.

Respuesta al ejercicio 2. Nuevamente, el alumno se volverá inconsis-tente y creerá todo. En particular, creerá que Dios existe (también creeráque Dios no existe, pero esto no üene importancia en este caso). Si g esla proposición Dios existe, la proposición Bg es por lo tanto verdadera, yasí -Bg es falsa. Si Dios sí existe, entonces g es verdadera, en consecuenciag-Bg es falsa, y el profesor estaba equivocado. Si Dios no existe, entoncesg es falsa, F-Bg es verdadera, y el profesor terúa nzón.

Ejercicío -1. En los capínrlos anteriores hemos demostrado los treshechos siguientes:

1) Si un nativo le dice a un razonador de tipo 1r: 'Usted nunca creeráque soy un caballero", y el razonador cree que nunca será peculiarentonces se volverá peculiar (Teorema III, capínrlo 10, página 80).

2) Un nzonador peculiar de tipo 3 es inconsistente (Problema 8,capírulo 11).

3) Un razonador de üpo 4 cree que si llega a volverse peculiar, seráincoruistente @jercicio 1, capítulo 11).

Usando estos tres hechos, podemos hacer una demostración delTeorema I de este capítulo mucho más rápida que la anterior. ¿Cómo?

Respuxta al eJercício 3. Supongamos que el nativo le formula esteenunciado: 'Usted nunca creerá que soy un caballero' a un razonador-detipo 4 y el razonador cree que nunca será inconsistente. Entonces elrazonador creerá que no puede ser peculiar (porque sabe que si es pecu-liar, será inconsistente). Sntonces según el Hecho 1 ya citado, se volverápeculiar, y según el Hecho 2 ya citado se volverá inconsistente.

EL DUAI DEL TEOREMA G

Ejercicío 4. Supongamos que el nativo envez de decir: 'Usted nuncacreerá que soy un caballeroo, diie: "Ustedcrcerá que soy un bribón". Supo-niendo que el razonador es de üpo 4 y cree en su propia consistencia, ¿sesigue ahora que se volverá inconsistente?

Soluctón.I-a respuesta es sl, y esto puede establecerse fácilmente comocorolario del Teorerna G, pero primero me gustaría esbozar un argumentodirecto. ürazonador razona: "Supongamos que es un caballero. Entonceslo que dijo es verdad, lo cual significa que yo creeré que es un bribón. Unavez que crea que es un bribón, creeré lo contra¡lo de lo que dijo: creeréque no creo que es un bribÓn. Pero si creo que es un bribÓn, también creeré

96

que sl creo que es un bribÓn (porque soy normal), y en consecuencia seréinconsistente. Dado que nunca seré inconsistente, él no puede ser uncaballero después de todo; debe ser un bribón. Ahora creo que es unbribón. El dijo que así sería, y por lo tanto es un caballero.

En este punto el rtzonador es inconsistente.

Lo que acabamos de demostrar es un caso especial del siguiente "dual"clel Teorema G.

Teorerna d. Si un razonador consistente de tipo 4 cree cierta pro-posición de la forma p=B-p, entonces nunca puede saber que es con-sistente.

2

El Teorema G" puede demostrarse "dualizando" el argumento delTeorema G, pero puede obtenerse en forma mucho más simple comocorolario del Teorema G. ¿Cómo?

Soluclón. Supongamos que el razonador cree p=B-p. Entonces creerá-p=-B-p. Supongamos que q es la proposición -p. Entonces elrazonadorcree q=-Bq, y por lo tanto el razonador cree una proposiciÓn de la formap=-Bp (justamente, q=-Bq), y asl el resultado se sigue del Teorema G.

EJerc,tcto S.Supongamos que en el Ejercicio 4 agregamos el supuestode que las reglas de la isla son realmente válidas y que el razonador creeen su propia consistencia. El naüvo ¿es un caballero o un bribón?

EJerclclo 6 Supongamos que un profesor de teología le dice a sualumno de tipo 4: 'Dios existe si y sólo si usted nunca va a cÍeer que Diosno existen. Supongamos que el alumno le cree al profesor y también creeen su propia consistencia. Demostrar que si el enunciado del profesor esverdadero, entonces Dios debe existi¡. Demostrar que si el enunciado delprofesor es falso, entonces Dios no existe.

EJerclclo 7. Supongamos que un nativo le dice a un razonador de tipo4: 'Usted nunca creerá que soy un caballero". (O, alternaüvamente, dice:'Usted creerá que soy un bribón".) Demostrar que el razonador sabe quesi llega a creer en su propia consistencia, se volverá inconsistente. CLasolución de este ejercicio será consecuencia fácil de resultados que sedemostrarán en capltulos posteriores.)

97

Page 51: SMULLYAN, R. Juegos por siempre misteriosos. España. Gedisa 2008

73

Sistemas gódelianos

Todos los rezultados qué hemos demostrado hasta ahora sobre razo-nadores y sus creencias son la contraparte de los resultados metama¡e-máücos sobre sistemas matemáücos y las proposiciones demostrables enellos. Antes de explayarnos sobre el tema, vamos a resumir los hechos mássignificaüvos que hemos demostrado en los últimos capltulos.

Rsurnen I. Supongamos que un razonador cree las reglas de la isla yun nativo le dice: 'Usted nunca creerá que soyun caballero'. O, de modomás general, supongamos que un razonador cree cierta proposición de laforma p=-Bp (p es verdadera si y sólo si ru¡nca voy a creer p). Entonces:

1) Para un razonador de üpo 1, si cree que es siempre correcto, enton-ces se volverá incorrecto; en realidad, se volverá pecuüar.

2) Para un razonador de tipo 1., si cree que nunca será peculiar, en-tonces se volverá peculiar.

3) Para un razonador de tipo 3, si cree que nunca será peculiar, enton-ces se volverá inconsistente.

4) Pare un razonador de üpo 4, si cree que siempre será consistenteentonces se volverá incoruistente.

Todos estos hechos, en particular el 4), se relacionan con importanteshechos metamatemáticos que analizaremos brevemente.

Los tipos de sistemas investigados por Kurt Gódel tienen las siguien-tes características. En primer lugar, hay un conjunto bien definido deproposiciones expresables en el sistema; las denominaremos propo-siciones del sistema. Una de esas proposiciones es l- (falsedad lógica), ypara toda proposición p y q del sisrema, la proposición (poq) también esuna proposición del sistema. Todos los conectores lógicos &, v, -, = puedendefinirse a p^rrír de t y I en la forma que se explicó en el capítulo 7.

En segundo lugar, el sistema -llamémoslo "S"- üene varios axiomasy reglas lógicas que hacen que ciertas proposiciones sean demostrables enel sistema. De este modo tenemos un subconjunto bien definido deproposiciones del sistema: el coniunto de proposiciones detnostrables delsistema.

En tercer lugar, para toda proposición p del sistema, la proposición de

98

que p es fumostrable en el sistema es en sí misma una proposición delsistema (puede ser verdadera o falsa, y puede ser demostrable en elsistema, o también puede no serlo). Supongamos que Bp es la proposiciÓnde que p es demostrable en el sistema. (Gódel introdujo el símbolo '8" para"demostrable"; repres€nta la palabra alemana bewelsbar.) Por una afor-tunada coincidencia, usamos el símbolo "B" en dos sih¡aciones estrecha-mente relacionadas. Cuando hablamos de razonadorcs, Bp significa quecl razonador cree p; cuando hablamos de slstemas matemáticos, Bpsignifica que p es demostrable en el sistema.

Ahora decimos que unsistema S cs de tipo 1., 1t, 2,3y 4, exactamentecn la misma forma en que lo hicimo.s ¡>ara los razonadores: Si todas lastautologías son demostrables en S, y si ¡rara proposiciones cualesquiera, py p>q ambas demostrables en S, q tanllrión cls demostrable en S -si estasdos condiciones son válidas- enton('(:s tl<:t:imos que S es de tipo 1. Siademás Bp>Bq es demostrable en S sicttt¡rrtr (¡rc prq es demostrable enS, entonces decimos que S es de tipo l'. Si ;¡tlt'rrtfs to<la.s las proposicionesde la forma (Bp&B(p>q))>Bq son dct¡toslt;tlrlq's t'tt S, cntonces decimosque S es de tipo 2. Si además S es nortrt¡tl (t's tlct'ir, ll¡r c.s demostrablesiempre que p lo es), entonces decinlos r¡trt' 5 t's tlt' ti¡ro ,¡i. l'or último, sitodas las proposiciones de la forma ll¡r rl l l l¡r sott clt:rrtoslrall les en S,entonces decimos que S es de üpo 4. Itttt stl¡rtt<:sl() l.(xlrls l<¡s rt:st¡lta<lr¡.s delcapÍtulo 11 que demOStramOS para l<ls r¡t¿ttnrttk¡ras tatrlbid:rr sort váliclospara los slstemas (donde ahora intcr¡lrcl:rttt().s "l]" comtl ¿lt'truts!rutl¡h' <'ttvezde creíble). Con respecto a los rc¡rrl l .t,L¡s dc los callíttrkls l{) y 12,necesilamos otra condiciÓn que ahorlt vllrrr()¡i íl f<>rmular.

Slstemas gódelianos. Gódel realizÓ e I rrol:¡blc descubrimicnto dt: <¡ttt:cada uno de los sistemas que investi¡¡(r terrir l;l ¡rr'opiedad de que hal>ia trnaproposición p tal que la proposiciÓn ¡='-ll¡r r'r':t tlcmostrable en el sistcnra.Decimos que esos sistemas se denontlrtart ilisl(:nlas gÓdeltanos.

La proposición F-Bp es muy [)artl('rrl.rt. At¡uí tenemos una proposi-ción p equivalente a su propia no tlt'¡¡ro¡lr.tltili<lad en el sistema. Puedeconsiderarse que la proposición p dicc; "Nt t soy clcmostrable en el sistema".La forma en que Gódel logró desculrrir t,rl ¡rro¡'ro.sición no debe preocu-parnos áhora, aunque vamos a consitlrr.rrl;l ('¡l uno de los últimos capí-tulos.

En analogía con los sistemas, pcxlerttos tk't'ir que un razonador es unrazonador gódeliano si por lo meno$ ltry tttt;t ¡rroposiciÓn p tal que creala proposición p=-Bp. Por supuesto hettlr rr¡ csl;tdo esn¡diando a los razona-dores gódelianos durante todo el tlhlttto t .t¡rtltrlo. (Si un razonadot llegaalalsla de los Caballeros y los Briboneg y r'tt't' las reglas de la isla, y si unnativo le dice: "Usted nunca creerá que ¡i()y r¡rr r':tballero", entonces el razo-nador cree la proposiciÓn k=-Bk; en ('{trl5('(tlcncia se convierte en unrazonador gódeliano.)

Ahora decimos que un sistema ptterle rlcrrtostrar su propia correcciÓnsi puede demostrar todas las proposicio¡tr's tlc la forma Bpop. Decimos que

99

Page 52: SMULLYAN, R. Juegos por siempre misteriosos. España. Gedisa 2008

É'

puede demostrar su propia no peculiaridad si puede demoshar todas lasproposiciones de la forma -(Bp&B-Bp). Decimos que el sistema puededemostrar su propia consistencia si puéde demostrai ra proposició; -Br.

Ahora volvamos a exponer nuestro resumen de ape-rtuá en términosde sistemas, en vez de razonadores.

Resumen /11) Si un sistema gódeliano de tipo 1 puede demostrar supropia corrección, entonces es incorrecto de hecho, peculiar.

?) Si y"- sistema gódeüano de tipo 1. puede dem-osrrar su propia nopeculiaridad, entonces es peculiar.

.. 3) Si un sistema gódeliano de üpo 3 puede demostrar su propia no pe-culiaridad, entonces es inconsistente.

. 4) -Si

un sist€ma gódeliano de tipo 4 puede demostrar su propia. con-sistencia, entonces es inconsistente.

El ltem 4) es el verdaderamente importante; es una forma generalizadadel famoso Segundo Teorema de la hicompletitud de Gódel.

Anállsls. En su artículo original de 1931, Gódel consideró un sistemaparticular (el sistema de mncela Mathenatlca de whitehead y Russell)y.demostró que el sistema, si era consistente, no podía demostar su pro-pia consistencia: Sostenía que su método se aplicaba no sólo a ese siste-ma en particular, sino a una amplia variedad de sistemas. En realidad, elmétodo se aplica a todos los sistemas gódelianos de üpo 4, como acaba-mos de ver.

Otro importante sistema gódeliano de üpo 4 es el sistema conocidocomo 'Aritmética' (de modo más completo, .A¡itméüca de peano dePrimer orden'). Esta es una formalización de la teoría de los númerosenteros ordinarios 0, 1, 2, ... Dado que la A¡itmética es un sistema gódelianode üpo 4, esfá, sujeta al Teorema áe la Consistencia de Gódel; án conse-cuencia si la Aritmétic¿ ers consistente (y así es, dado que sólo las propo_siciones verdaderas son demostrables en él), entonceino puede á"-'os-trar su propia consistencia.

Cuando el matemático André Veil se enteró de esto, hizo el famosochiste: 'Dios existe, dado que la Aritméüca es consistente; el Diablo existe,dado que no puede demostrarlo".

.Este chiste, aunque encantador, realmente es engañoso. No es que nopodamos demostrar la consistencia de le Aritmétical es que la Adtinéttcano prcde demostrar la corslstencla de la Añtmétlca. E] cierto qve no_sottos podemos demostrar la consistencia de la Aritmética, pero nuestrademostración no puede formalizarse en la propia ¡ritméticá.

realidad, ha habido mucha confusión popular con respecto alSegundo Teorema de Gódel (el Teorema de la

-Consistencia); esto se ha

debido en_parte a la iresponsabilidad de algunos periodistas áentíficos youos divulgadores. Cfo, por sr¡puesto, estoy completamente de acuerdocon la divulgación, siempre que la divulgación no sea incorecta.) Undivulgador en particular escribiór "El Teorema de Gódel significa que

100

nunca podemos saber que la Aritmética.es coosistente". Esto no üenesenüdo en absoluto. Para demostrar lo tonto que es, supongemos quehubiera resulado que la A¡itmética podía demostmr su propia consis-tencia; o para ser más realistas, supongamos que consideramos algún otrosistema. que fuede demostrar su propia consistencia. ¿Eso consütuirla algu-na garantia. de la consistencia del sistema? Por supuesto que no. Si el sis-tema fuera inconsistente, entonces al ser de üpo 1, podda demostrarcualqubr cosa, incluso su propia consistencia. El hecho de confiar en laconsistencia de un sistema sobre la base de que puede demostrada seríatan ingenuo como confiar en la veracidad de una persona sobre la base deque ésta afirma que siempre es veraz. No, el hecho de que la Aritméücano pueda demostrar su propia consistencia no arroja ni la mínima dudasobre Ia consistencia de la Aritrnética.

De hecho, en este libro vamos a construir varios sistemas gerdelianosde üpo 4, y denastrarenos su consistencia más allá de toda duda. Entoncesdemostraremos que, por ser consistentes, los sistemas no podrán demos-trar su propia consistencia.

EJerclcío. Consideremos un sistema S de tipo 4 (pero no necesaria-mente gódeliano). Recordemos que para toda proposición p, Iaproposición Bp es verdadera si y sólo si p es demostrable en S.

a) En primer lugar, demostrar que para toda proposición p, laproposición (B(p=-Bp)&B-Bl-))Br es verdadera (para el sistema S). Estoes muy fácil.

b) En segundo lugar, demostrar que (B(p=-Bp)&B-Bl-)>BI esrealmente demostrable en S. (¡No es tan fácilD

101

Page 53: SMULLYAN, R. Juegos por siempre misteriosos. España. Gedisa 2008

1,4

Má,s problemas con la consistencia

ALGUNOS PROBLEMAS PRETIMINARES

I

. Supongamos que untazonador cree que es inconsistente. ¿Es necesa_riamente inconsistente? ¿Es necesariamenie incorrecto?

2

. Supo_ngamos que un razonador cree que es incorrecto. Demostrar queüene razón.

3Supongamos que un^razonador de üpo l. cree que no puede ser in_

consistente (cree -Br). ¿creerá necesaria-mente que nunqr c¡eerá que esinconsistente? (Fs decir, ¿creerá necesariamente IS¡ID

- - -- 1-- -

MAS PROBTEMAS CON I.¿. CONSISTENCIA

Regresamos a la Isla de los cabalreros y los Bribones. seguimos suoo-niendo que el razonador es de üpo 4 y que cree las reglas áe U i"tr-'-

4

supongamos que un naüvo le dice al razonadorz 'si soy un cabalero,entonces creerá que soy un bribón'. Demostrar:

a) El razonador tarde o tamprano creerá que es inconsistente.b) si las reglas de la isla son realmente válidas, entonces el razonador

.sc volverá inconsistente,

t02

Nota: En los problemas de este capínrlo, zo suponemos que el razo-nador cree que es consistente.

5

Supongamos que el nativo dice en cambio: "Si soy un caballero,entonces usted nunca creerá que lo soy". Demostrar:

a) El razonador se volverá inconsistente.b) Las reglas de la isla no son realmente válidas.

Nota 1: Este problema tiene validez aun cuando el razonador sea detipo 3 solamente.

Nota 2: Parala versión "alumno y su profesor de teologla" de los dosúltimos problemas, véase el análisis que sigue a la soluciÓn del proble-ma-5.

6

Este es un curioso problema.IJ¡razonador de tipo 4llegaalo qlue creees una isla de caballeros y bribones (cree las reglas de la isla) y un nativole dice las siguientes dos cosas:

1) 'Usted creerá que soy un bribÓn'.2) "Usted siempre será consistente".Demostrar que el razonador se volverá inconsistente y que las reglas

de la isla no son válidas.

7

En este caso un nativo le formula los dos enunciados siguientes a unrazonador de üpo 4:

1) 'Usted nunca creerá que soy un caballero".2) 'Si llega a creer que soy un caballero, entonces se volverá in-

consistente'.Demostrar que el razonador se volverá inconsistente y que las reglas

de la isla no son válidas.

RAZONADORES TIMIDOS

Decimos que un razonador no debería creer p si su creencia en p lohace caer en una inconsistencia. Decimos que un razonador tene $eer psi cree Bp>BJ-; es decir, si cree que su creencia en p lo harl caer e! unaincoruisiencia. CEn otras palabras, teme creer p si cree que no debeíacreer p.)

103

Page 54: SMULLYAN, R. Juegos por siempre misteriosos. España. Gedisa 2008

Sabemos que todo razonador de üpo 4 que cree las reglas de la isla ya quien un naüvo le dice que él nunca creerá que es un caballero no deberíacreer en su propia consistencia. En general, sin embargo , nohay raz6n paraque un razonador de tipo 4 no crea en su propia consistencia. pero ahorasurge algo muy curioso: si por algún motivo, un razonador de tipo 4 temecreer en su propia consistencia, su mismo temor lo justifica. Esto significaque todo razonador de tipo 4 que teme creer en su propia consistencia,realmente no debería creer en su propia consis¡encia. Dicho aun de otromodo, si un razonador de tipo 4 cree que la creencia en su propia consis-tencia lo llevará a una inconsistencia, ¡entonces así serál

Por más sorprendente que pueda ser este hecho, no es dificil de-mostrarlo. Además, este hecho es válido para los razonadores normales detipo 1.

I

Demostrar que si un razonador normal de tipo 1 teme creer en supropia consistencia, entonces realmente no debería creer en su propiaconsistencia.

Corrrqntarlos.En el problema citado, observamos que para un razona-dor normal de tipo 1, su creencia en la proposición B-BJ_¡BI es auto-suficiente, en el sentido de que el creer esa proposición es condición su-ficiente para que la proposición sea verdadera. El tema de las creenciasautosuficientes desempeñará un papel importante en los próximos ca-pítulos.

9

Un razonador de tipo 4 también es un razonador normal de tipo1 ; en consecuencia, según el último problema, si teme creer en su pro-pia consistencia, entonces no deberla creer en zu propia consistencia.Esto significa que para un nzonador de tipó 4, la proposiciónB(B-BbBI-)¡(B-B-bBJ-) es verdadera. Demostrar que todo rizonadorde tipo 4 sabe que esa proposición es verdadera (sabe que si teme creeren su propia consistencia, entonces no debería creer en ella).

10

Supongamos que un razonador de üpo 4 cree que tetrc creer ensupropia consistencia. ¿Se sigue que realmente telme qeet ensu propia con-sistencia?

SOLUCIONES

1. Supongamos gue un razonador cree que es inconsistenüe. No veo

l( )4

por qué deba ser necesariamente inconsistente, pero debe ser incorrectopor las siguientes razones:

Un razonador que cree que es inconsistente tiene una creencia correctao incorrecta. Si su creencia es equivocada, entonces obviamente es inco-recto (tiene la creencia falsa de que es inconsistente). Si su creencia escorrecta, entonces realmente sí cree la proposiciÓn falsa J. En cualquiercaso, por lo menos tiene una creencia falsa.

2. Si estuviera equivocado, entonces sería correcto, lo cual es unacontrad.icción.

3. Supongamos que cree -Bl-. Entonces cree la proposición lógi-camente equivalente Bl-=l-. También es regular (dado que es de tipo 1f),y en consecuencia creerá BBJ-=B.j-; por lo tanto creerá la proposiciónlógicamente equivalente -Bb-BBl-. Ya que también cree -BI, entoncescreerá -BBl-.

4. Sabemos según el Teorema I del capítulo 3 que para toda propo-sición p, si un nativo de una isla de caballeros y bribones dice: "Si soy uncaballero entonces p", el nativo debe ser un caballero y p debe serverdadera. Ahora bien, todo razonador -aun de tipo 1- sabe esto igualque ustedes y yo,y por lo tanto si el razonador cree que las reglas de laisla son válidas, entonces si un nativo le dice: "Si soy un caballero, entoncesp", el razonador creerá que el nativo es un caballero y que p es verdadera.En este problema en particular, el nativo ha dicho: "Si soy un caballero,entonces usted creeráque soy unbribón", y por lo tanto el razonador c¡eeráque el naüvo es un caballero y también que él (el razonador) creerá queel nativo es un bribón. Y en consecuencia el razonador creerá k y tambiéncreerá B-k (k es la proposición de que el nativo es un caballero). Hastaahora, sólo hemos usado el hecho de que el razonador es de tipo 1. Sinembargo, es de tipo 4, y dado que cree k, también cree Bk. En consecuenciacreerá Bk y creerá B-k, pero sabe que (Bk&B-k)>BA (como demostramosen el capítulo 11., página 91). Por lo tanto creerá BJ-; es decir, creerá quees (o será) inconsistente. Además cree que el nativo es un caballero.

Hásta ahora, no hemos usado el hecho de que las reglas de la isla sonrealmente válidas; nuestro argumento precedente sólo usó el hecho de queel razonador cree que las reglas son válidas. Ahora bien, supongamos quelas reglas realmente son válidas. Entonces el nativo realmente es uncaballero y el razonador realmente creerá que el nativo es un b¡ibón (segúnel Teorema I, capítulo 3). Pero dado que el razonador también cree queel nativo es un caballero, se volverá inconsistente.

5. En este caso el razonador cree le(k>-Bk), en consecuencia cree ky cree -Bk, que son consecuencias lógicas de te(k¡-Bk). Dado que creek, creerá Bk, y al creer -Bk, se volverá inconsistente.

105

Page 55: SMULLYAN, R. Juegos por siempre misteriosos. España. Gedisa 2008

Si las reglas de la isla son realmente válidas, entonces le(k¡-Bk) nosólo es creída por el razonador, sino que realmente es verdadera, enconsecuencia k&-Bk (que se sigue lógicamente de ella) es verdadera, enconsecuencia -Bk es verdadera, opuesto al hecho de que el ruzonador sícree que el nativo es un caballero (y en consecuencia Bk, en vez de -Bk,es verdadera). De este modo las reglas de la isla no son realmente válidas.

Análisis. Consideremos los dos últimos problemas en la versión delalumno y su profesor de teología. Supongamos que el profesor dice: "Diosexiste, pero usted nunca creerá que Dios existe". Si el alumno es de tipo4 y cree al profesor, tendrá que creerse inconsistente. Si además el enun-ciado del profesorera verdadero, entonces el alumno realmente se volveráínconsistente.

Por otro lado, supongamos que el profesor dice: "Dios existe, perousted nunca creerá que Dios existe". Entonces, si el alumno es de üpo 4-o aun de tipo 3-y cree al profesor, se volverá inconsistente, y además,el enunciado del profesor es falso.

6. El razonadoÍ razonai "supongamos que es un b¡ibón. Entonces susegundo enunciado es falso, lo que significa que seré inconsistente, enconsecuencia creeré todo, en partiolar que es un bribón. Pero estovalidará su primer enunciado y lo convertirá en un caballero. Por 1o tantoes contradictorio suponer que es un bribón, en consecuencia debe ser uncaballero. Como es un caballero, su primer enunciado es verdadero, enconsecuencia creeré que es un bribón. Pero ahora creo que es un caballero;en consecLrencia seré inconsistente. Esto demuestra que seré inconsistente.Sin embargo, es un caballero y dijo que yo siempre seré consistente. PorIo tanto nunca seré inconsistente

En este punto el razonador h allegado ala conclusión de que se volveráinconsistente y de que no se volverá inconsistente, y por lo tanto aho¡a esinconsistente.

Como el razonador se ha vuelto inconsistente, el segundo enunciadodel nativo es falso. Además, dado que el ruzonador se ha vuelto incon-sistente, creerá todo, incluso que el nativo es un bribón. Esto convierte enve¡dadero al primer enunciado del nativo. Dado que el naüvo ha formu-lado un enunciado verdadero yun enunciado falso, entonces las reglas dela isla no son realmente válidas.

7. El razonador cree los dos siguientes enunciados:1) k=-Bk2) le(Bk¡Bl)

Como cree 1), entonces según b) del Teorema G*, capítulo 1-.2, el ra-z<>nador creerá Bk¡BI. Y, dado que cree 2), creerá k. Por lo tanto, segúna) del Teorema G", capítulo 12, se volverá inconsistente.

l (X)

Además se sigr.re que el primer enunciado del naüvo era falso y elsegundo enunciado era verdadero. Por lo tanto las reglas de la isla no sonválidas.

8. Suponemos que el razonador es norrnal y de tipo 1 y que creeB-Bl-tBI Debemos demostrar que si cree que es consistente, se volveráinconsistente (y por lo tanto que su temor es iustificado). Entonces,supongamos que sí llega a creer -Ba. Dado que es normal, entonces creeráB-Bl-. Esto, junto con su creencia en B-Bj:BI, lo harán creer Bl- (porquees de üpo 1). Y por lo tanto creerá tanto BI como -Ba (creerá que esinconsistente y que no es inconsistente), lo cual lo hará inconsistente.

9. El ruzonador razonai'Supongamos que llego a temer creer en mipropia consistencia. Esto significa que creeré que no debería creer en mipropia consistencia; es decir, creeré B-BbBl- Dado que soy de tipo 1 (porser de üpo 4), entonces creeré -Bl-=-B-Bl. Ahora supongamos quetambién creyeru que soy consistente; es decir, supongamos que crco -BJ-.Entonces dado que creeré -Bl¡-B-Bl-, creeré -B-BI. Pero tar¡rbiéncreeré B-B1 (dado que creeré -BI y soy normal) y en consecuencia seréinconsistente. Por lo tanto, si temo creer en mi propia consistencia,realmente no puedo creer en ella sin volverme inconsistente. Por lo tantola proposición B(B-Bl-:Bl-)¡(B-Bl-:Bf) es verdadera".

10. Acabamos de ver que el razonador cree la proposiciónB(B-Bl5Bl-)r(B-BbBJ-). Por lo tanto, si cree B(B-B-l-¡Bl-), creeráB-BbBl-, lo que significa que si cree que teme creer en su propia con-sistencia, entonces realmente temerá creer en su propia consistencia.

't07

Page 56: SMULLYAN, R. Juegos por siempre misteriosos. España. Gedisa 2008

Parte VI

LAS CREENCIASAUTOSUFICIENTES YEL TEOREMA DE LÓB

Page 57: SMULLYAN, R. Juegos por siempre misteriosos. España. Gedisa 2008

15

Creencia,s que se autocumPlen

Todos los problemas de este capítulo se relacionan con el Teorema deLób, famoso resultado que es importante pata el avance de este libro.

Ahora tenemos un cambio de argumento: Un razonador dc tipo 4piensa visitar la Isla de los Caballeros y los Bribones porquc ha oÍd<l clrumor de que los baños de azlttfrey las aguas minerales del lu¡¡ar put:tl<:nCurar Su reumatiSmO. Sin embargO, antes de embarCarse, analiz.a la sitrla'ción con su médico de cabecera. I€ pregunta al médico si la "cura" rc:t'mente sirve. El médico responde: "La c'tta es absolutamente psicol(r¡¡i<'a;la creencia de que sirve es autocumplidora. Slttstedctee que la cura x:nttrá, entonces smirá.".

El nzonador confta implícitamente en su médico, y por lo tanto va ala isla con la creencia previa de que si cree que la cura servirá, entonccsla cura servirá. Se somete ala anra, que dura solamente un día, pero qucse supone que no sirve para varias semanas, si es que sirve para algo. Aldía siguiente empieza a preocuparse por la situaciÓn y piensa: "Si pudierac¡eer qve la cura sirve, entonces servirá'. Pero ¿cÓmo sé si llegaré a creerque sirve? No tengo evidencia racional de que la cura servirá, ni tengoninguna evidencia de que llegaré a creer que la cura servirá. Hasta dondeyo sé, podría no creer nunca que la cura servirá, y en consecuencia la curapodría no servir.

Un naüvo de la isla pasa a su lado y le pregunta al tazonadot por quéestá tan afligido. El razonador le expüca toda la situaciÓn, luego la resumediciendo: 'Si llego a creer que la cura servirá, entonces así será, pero ¿lle-garé a creer que la cura servirá? El naüvo responde: "Sl llega 4 creer que soyun cabaueno, entonces creerá que la cura seruírd'.

Al principio, eso no le pareció muy üanquilizador al razonador. Elpiensa: "¿De qué me sirve eso? Aun si lo que dice es verdad, eso sÓlo redu-ciría el problema a determinar si llego a creer que es un caballero. ¿CÓmosé si llegaré a creer que es un caballero?Y aun si lo hago, él puede ser unbribón y su enunciado puede ser falso; en consecuencia aún puedo nocreer que la cura servirá". Pero luego el razonador meditÓ más sobre elproblema, y después de un rato suspirÓ aliviado. ¿Por qué?

Bueno, como veremos, lo sorprendente es que el razonador creeráque la cura servirá y, suponiendo que el médico tiene razÓn, ¡la cura servirá!fodríamos notar que las reglas de la isla no tienen que ser realmente válidas

111

Page 58: SMULLYAN, R. Juegos por siempre misteriosos. España. Gedisa 2008

para qr¡e- funcione el argumento; es suficiente que el razona dor ctea queson válidas.

- Este problema Ee relaciona estrechamente con el importante teorema

de M. H. Lób, que analizaremos más adelante. pero primeio, consideremosun problemaun poco más simple, que se acerca aun más al argumento ori-ginal de Lób. Supongamos que el nativo, envez de decir lo citádo anterior-mente, dice: 'Si llega a creer que soy un caballero, entonces la cura servirá',.

1 (Según Lób)

Demostrar que según las condiciones ya citadas, elrazonador creeráque la cura servirá (y en consecuencia, si el médico tenia raz6n, entoncesla cura servirá).

Solución. Resulta más fácil dar la solución en parte con palabras y enparte con símbolos. Supongamos que k es la proposición que el nativo esun caballero, y supongamos que C es la proposición que la cura servirá.Al principio, el razonador cree la proposición BC)C.

El razonador razonai "supongamos que llego a creer que es un caba-llero. Entonces creeré lo que dice: creeré que Bk>C. Además, si llego acreer que es un caballero, creeré que creo que es un caballero: creeé Bk.Y por lo tanto, si llego a creer que es un caballero, creeré tanto Bk comoBk>C, en consecuencia creeré C. De este modo, si llego a creer que es uncaballero, entonces creeré que la cura sirve. pero si llego a creer qúe h curasirve, entonces la cura servirá (como me dijo mi médico). y poi lo tanto,si llego a creer que es un caballero la cura servirá. Bueno, ejo es exacta-mente lo que él dijo: que si llego a creer que es un caballero, entonces lacura servirá, ¡y tenía razón! En consecuencia, es un caballero',.

En este punto el razonador cree que el 4ativo es un caballero, y dadoqrre el razonador es normal, dice: "Ahora creo que es un caballero. ya hedemostrado que si creo que es un caballero entoñces la cura servirá, y dadoque creo que es un caballero, la cura seryirá,'.

- En este punto el razonador cree que la cura se.rvirá. Entonces, supo_

niendo que el médico tenía raz6n, la cura servirá.

La solución del problema 1 podría haberse est¿blecido en forma másrápida de haber demostrado primero el siguiente lema, que también tendráotras aplicaciones.

. .1",*q l.Dada cualquier proposición p, supongamos que un nativo dela isla le dice a un razonador de üpo 4: 'si llega á creer que soy un caballero,entonces p es verdadera". Entonces el razonador creerá: "Si llego a creerque es un caballero, entonces creeré p". De modo más general, para dosproposiciones_cualesquiera k y p, si un razonador de tipo 4 creé la pro_posición k=(Bkrp), o aunque crea, la proposición más débil b(Bbp),entonces creerá Bk>Bp

r12

EJercicío 1. ¿Cómo se demuest¡a el Lema 1? (Este es el mismo problemaque se encuentra en el Ejercicio 3, capírulo 11.)

Eje?f,icio 2. ¿En qué forma facilita el uso del Lema 1 la solución delproblema 1?

Respuesta al e1'ercicio l. En primer lugar lo demost¡amos en la versiónde cabaileros y bribones. Supongamos que k es la proposición que elnativo es un caballero. El nativo ha afirmado la proposición Bk>p. Elrazo¡ador tazonai "Si llego a creer que es un caballero, entonces creeré loque dice: creeré Bk:p. Pero si creo que es un caballero, también creeré Bk(creeré que creo que es un caballero). Unavez que crea Bbp y que creaBk, entonces creeré p. Y por lo tanto, si llego a cieer que es un caballero,entonces creeré p".

Por supuesto la forma más general puede demostrarse esencialmentedel mismo modo, o alternativamente así: Supongamos que un razonadorde tipo 4 cree k>(Bk)p), lo cual ciertamente creerá, si cree la proposiciónmás fuerte le(Bbp). Entonces, según el problema 2, capí0.rlo 11., cree-rá Bk>(BBk>Bp). También cree Bk>BBk. Como cree esas dos proposicio-nes, creerá Bk>Bp, que es una conseq.tencia lógica de las mismas. (Paraproposiciones cualesquiera X, Y y Z, la proposici1¡X>Z es una conse-cuencia lógica de )b(ftrZ) y )bY. En particular, esto es así si X es laproposición Bk, .Y es la proposición BBk y Z es la proposición Bp.)

RnpuestaaleJerctclo2.El razonador cree le(Bk>C), porque el nativoafirmó Bk>C. Entonces, según el Irma 1, el razonador creerá Bk>BC.También cree BC>C, en consecuencia creerá Bk>C. Entonces creerá k(dado que cree tanto Bk>C como k=(llk¡C)), en consecuencia creerá Bk(porque es normal). Ahora que cree llk y cree Bk>C, creerá C.

El resultado del problema L es el siguicnte teorema.

Teorerna I (según Lób).Para toda ¡rro¡ro.sición k y C, si un razonadorde tipo 4 cree BC>C y cree Bk y cree kr(llk::C), entonces creerá C.

El Teorema I da el siguiente resultado r'rrrioso.

EJerclclo3.Supongamos que un estuctianr(: (tc teología está preocupa-do por cosas tales como la existencia de l)ios y su propia salvación. Lepregunta a su profesor: "¿Existe Dios?" y "¿St,r'(' redimido?" Entonces elprofesor formula los siguientes enunciadcls:

1) "Si cree que será redimido, entoncen ¡¡c¡ii rcdimido".2) "Si Dios existe y cree que Dios existc, {.nt()nces será redimido".3) "Si Dios no existe, entonces creerá (lue l)it¡s cxiste".4) "Serl redimido sólo si Dios existe".Suponiendo que el esn:diante es un ra?(¡n{tl rr rlc tipo 4 y le cree al

profesor, demostrar: a) El estudiante creerá {¡rre. x.rfr redimido; b) Si los

1L3

Page 59: SMULLYAN, R. Juegos por siempre misteriosos. España. Gedisa 2008

enunciados del profesor son verdaderos, entonces el esh-rdiante seráredimido.

Soluclón. Supongamos que g es la proposición que Dios existe y su-pongamos que S es la proposición que el estudiante será redimido. Enton-ces el estudiante cree las siguientes cuatro proposiciones:

1) BS>S2) (g&Bdrs3) -g2Bg4) Srg

La proposición -Bg>g es una consecuencia lógica de 3). Estaproposición, junto con 4), t.iene como consecuencia lógica la proposición(-Bgvs)rg.Además -BgvS es lógicamente equivalente a BpS, en conse-cuencia (-BgvS)>g es lógicamente equivalente a (Bg>S)>g. Ademásgp(Bg>S) es lógicamente equivalente a 2), y g-=(Bg:S) es una consecuencialógica de (Bg¡S)>g y g¡Gg>S). Por lo tanto g=(BpS) es una consecuen-cia lógica de 1), 2) y 3). Dado que el estudiante cree 1),2),y 3), tambiéncreerá g(Bg=S). Ya que según 1) también cree BS>S, entonces según elTeorema I, creerá S. De este modo BS es verdadera, y si el profesoi teníarazón, BSrS es verdadera, en consecuencia S es verdadera, lo cual significaque el estudiante será redimido.

Ahora regresemos al problema 1 y su proposición; ,,Si llega a creer quesoy un caballero, entonces la cura servirá".

2

Supongamos que un razonador de tipo 4 nuevamente cree BGC, peroen este caso el nativo dice: "Si llega a creer que soy un caballero, entoñcescreerá, que la cura sirve". Demostrar que el razonador creerá nuevamenteque la cura servirá.

Soluclón. En este caso el nativo afirma BkrBC (en vez de Bk:C).Entonces según el Lema 1, el razonador creerá BIoBBC (en vez deBbPC). Sin embargo, el razonador cree BC>C, y como es de tipo 4 esregular y creerá BBC=BC. Al crer esto y Bk>BBC, creerá Bk>BC. Timbiéncree k=(Bk:BC), entonces creerá k. y asÍ creerá Bk y como creerá Bk>BC,creerá BC. Pero también cree BC,cC, en consecuencia creerá C.

Por supuesto este argumento se generaliza a{t:

Teorcma II. Dadas proposiciones cualesquiera k y C si un razonadorde tipo 4 cree BC>C y cree k=(Bk>BC), entonces creerá C.

7t4

3Nuevamente un razonador de üpo 4 üene la creencia previa de que si

cree que la cura servirá, en[one$ la ct¡ra servirá. En este caso el naüvo ledice: "Tarde o temprano creerá que si soy un caballero, entonces la curaservirá" Veremos que otra vez el r-azomdor creerá que la cura servirá.

De modo más general, vamos a demostrar el siguiente teorem¿.

Tnrcma IILPara proposiciones cualesquiera k y C, si un razonador detipo 4 cree BC>C y cree leB(k>C), entonces creerá C.

Ios dos lemas siguientes, interesantes ensl mismos, facilitan la demos-tración del Teorema IIL

Lema2.supongamos que para cierta proposición q, un naüvo le dicea un razonador de tipo 4: 'Usted creerá q". Entonces el razonador creerá:'Si él es un caballero, entonces creeré que si es un caballero, creeré quees un caballe¡q". @idro en forma más abstrac'ta, para proposiciones k yq, si un razonador de tipo 4 cree leBq, entonces creerá k>Bk.)

I¿ma3. Dada cualquier proposición p, supongamos que un nativo ledice a un nzonador de üpo 4: "Usted creerá que si soy un caballero,entonces p es verdadera". Entonces el.razonador creerá: "Si él es un caba-llero, entonces creeré p". (Dicho en forma más abstracta, si un razonadorde üpo 4 cree leB(k>p), entonces creerá k:Bp.)

¿Cómo se demuestran los I¿mas Z y 3 y el Teorema III?

Demostraclón &l Isma 2. Este es el problema 9 del capítulo 11, queya hemos resuelto, pero quiero present¿r una versión de caballeros ybribones de la demostración, que es particulamente intuiüva. El nativo hadictro: 'Usted creerá q'. Entonces el razonadorrazona: "Supongamos quees un caballero. Entonces crceré q. Y asl creeré que creo q; y creeré lo quedijo, es decir, creeré que es un caballero. Por lo tanto, si es un caballero,creeré que es un caballero".

Demostractón ful Lema 3.Vamos a usar el l.ema 2 para facilitar estademostración.

El naüvo ha didro: 'Usted creerá que si soy un caballero, entonces pD.Sea q la proposición: "Si soy un caballero, entonces p'. El nativo ha dichoal rezonador que creerá g, y, según el I¡ma 2, el razonador creerá que siel nativo es un caballero, entonces él (el razonador) creerá que el naüvoes un caballero. Y así el razonador razorr 2'Supongamos que es un caba-llero. Entonces creeré que es un caballero. Entonces creeré lo que dice:creeré Bk>p. También creeré Bk (creeré que creo que es un caballero). Por1o tanto, si es un caballero, creeré Bk>p y creeré Bk, en consecuenciatambién creeré p. Y, por lo tanto, si es un caballero creeré p".

115

Page 60: SMULLYAN, R. Juegos por siempre misteriosos. España. Gedisa 2008

DenasfraclÚn del Teorema III. Yoy a presentar una versiÓn decaballeros ybribones de la demostración. El nativo ha dicho: 'Usted creeráque si soy un caballero, entonces la cura servirá". Por el Lema ), el tazo'nador creerá: oSi es un caballero, entonces creeré que la cura servirá".Luego el razo¡zdor dice: "Además, si creo que la cura servirá, entonces lacura servirá. Por lo tanto, si es un caballero, la cura servirá. Ahora creo quesi es un caballero la cura servirá. El dijo que creeria eso, luego es un caba-llero. Por lo t¿nto, es un caballero y además (como he demostrado), si esun caballero, entonces la cura servirá. Por lo tanto, la cura servirá'.

En este punto el razonador creerá que la cura servirá.

Arrátlsis. También puede obtenerse el Teorema III como fácil corolariodel Teorema I con el siguiente argumento. Supongamos que se nos dan lasproposiciones k y C tal que un razonador de üpo 4 cree leBQoC) y cree'SC;C.

Tenemos que demostrar que creerá C. Como cree kB(k>C)'también debe creer (loCts(BCkrC)>C)), que es una consecuencia lÓgicade leB0oC)). Entonces cree la proposiciÓn k<Bk'rc), donde k' es laproposiciÓn k>C. Pero como también cree BCcC, entonces creerá C segúnel Teorema L

El siguiente ejercicio curioso ilustra la autorreferencia llevada hasta elextremo.

EJercicio4.Supongamos que un naüvo de la isla le dice avntazonadorde üpo 4: "Tarde o temprano creerá que si soy un caballero, entonces creeráque lo soy'.-

a) Demostrar que el razonador creerá que el naüvo es un caballero.b) Demostrar que si las reglas de la isla son válidas, entonces el naüvo

es un caballero.

Solución. Esta es una consecuencia fácil del I¡ma 2.a) El nativo ha afirmado BQ<>Bk). Por lo tanto hay una proposiciÓn q

tal que el nativo ha afirmado que el razonador creerá q. Entonces segúnel lema 2, el razonador creerá k>Bk. Entonces, co?no es normal, creeráB(k)Bk), o sea, creerá lo que dijo el naüvo. En consecuencia creerá queel naüvo es un caballero.

b) Dado que el razonador cree Bk, indudablemente cree lcBk; enconsecuencia él enunciado del nativo era verdadero. Y por lo tanto elnativo es un caballero (si las reglas de la isla son realmente válidas).

El contenido matemático esencial del citado eiercicio es que para todaproposición k, si un razonador de üpo 4 cree l=B(k>Bk), entonces tambiéncreerá k. Si también k=B(bBk) es verdadera, entonces k lo es.

FORMAS DUALES

Los problemas 1,2y 3 (de modo más general, los teoremas I, II y III)tienen sus formas "duales", que son bastante curiosas.

116

1" (Dual del problema L)

Nuevamente, un razonador de tipo 4llega a la isla creyendo de an-temano que si cree que la cura servirá, entonces la cura servirá. Se en-cuentra con un nativo que le dice: "La csra no servirá y usted creerá quesoy un bribón".

Demostrar que el razonador creerá que la cura servirá.

2" (Dual del problema 2)

Igual que el problema 10, excepto que el naüvo dice: "Usted creerá quesoy un bribón, pero nunca creerá que la cura servirá". Demostrar que sesigue la misma conclusión (el razonador creerá que la cura servirá).

3" (Dual del problema 3)

En este caso el nativo dice: uNunca creerá que si soy un bribón, en-tonces la cura servirá". (Alternativamente, podría decir: "Nunca creerá queo soy un caballero o que la cura servirá".) Demostrar que se sigue la mismaconclusión.

Soluclón del prcblema 7". PodrÍamos demostrado directamente, peroresulta más fácil sacar provecho del Teorema I, que ya hemos demostrado.

El nativo ha afirmado (-C&B-k), y por lo tanto el razonador creek=({&B-k). Pero sabemos que le(-C&B-k) es lógicamente equivalentea -k=-(-C&B-k), que a su vez es lógicamente equivalente a -le(B-k>C).Por lo tanto el razonador cree -HB-k>C), o sea, cree una proposición dela forma p=(Bp>C) -p es la proposición -k- y según el Teorema I, sicree BC>C, creerá C.

Las soluciones de los problemas 2" y 3o también pueden obtenersecomo corolarios de los Teoremas II y III, respecüvamente. Deiamos laverificación en manos del lector.

EJerrlclos. Supongamos que un nativo le dice a un razonador de üpo4: "Si llega a creer que soy un caballero, entonces será inconsistente". ¿I€resulta posible al razonador creer en su propia consistencia sin volverseinconsistente? (Pista: Usar el Teorema II.)

E|erclcto 6. Supongamos que un razonador de tipo 4 cree que si creeque la cura servirá, entonces servirá. Supongamos que ahora tenemos unnaüvo que le dice: "Si llega a creer que creerá que soy un caballero,entonces la cura servirá".

¿Creerá necesariamente el razonador que la cura servirá?

EJerclclo 7. Supongamos que el nativo dice en cambio: "Usted creeráque si llega

^ creeÍ que soy un catrallcro, cntonces la sra servirá".

117

I

Page 61: SMULLYAN, R. Juegos por siempre misteriosos. España. Gedisa 2008

¿Creerl necesariamente el nzonador que Ia cura servirá?

Ejercicio 8. El siguiente diálogo oorre entr€ un alumno y su profesorde teología.

Alumtn: -Si creo que Dios existe, entonces ¿creeré también que seréredimido?

Prcfesor: -Si eso es verdad, entonces Dios existe.Alumno: -Si creo que Dios existe, entonces ¿seré redimido?Prcfaor -Si Dios existe, entonces eso es verdad.

Demostrar que si el profesor es correcto y si el alumno le cree al pro-fesor, entonces Dios debe existir y el alumno será redimido.

EJercício 9. La siguiente versión más fuerte del Teorema III puededemostrarse. Un razonador de tipo 4llega a la isla para someterse a la curay tiene la creencia previa de que si llegara a creer que la cura servirá,entonces así será. t€ pregunta a un nativo: "¿Llegaré a creer que si ustedes un caballero, entonces la cura servirá?" El nativo responde: nSi eso noes así, entonces la cura serviráo. (Alternaüvamente, podda haber respon-dido: 'O eso es asl, o la cura servirá'.) El problema es demostrar que elrazonador creerá que la cura servirá. (Dicho en forma más abstracta, si unrazonador de tipo 4 cree le(C v B(bC)), y cree BC=C, entonces creerá C.)La demostración de esteejercicio se facilita demostrando primeramente losdos hechos siguientes como lemas:

1) Dadas proposiciones cualesquiera p y q, supongamos que un nativole dice a un razonador de tipo 4: "O p es verdadera o usted creerá q'.Entonces el razonador creerá:'Si el nativo es un caballero entonces creeréque o p es verdadera o el naüvo es un caballero".

2) Dadas proposiciones cualesquiera p y q, supongamos que un nativole dice a un razonador de tipo 4: 'O p es verdadera o si no usted creeráque si soy un caballero, entonces q es verdadera". Entonces el razonadorcreerá:'Si el naüvo es un caballero, entonces o p ed verdadera o yo creeréque q es verdadera".

EJerc,tclo lO. El úlümo ejercicio tiene el siguiente dual. De nuevo unrazonador de tipo 4 cree que si creyera que la cura servirá, entonces servirá.Ahora se encuentra con un nativo que le dice "La cura no sirve y ustednunca creerá que o yo soy un caballero o que la cura sirve". Demostrar queel razonador creerá que la cura servirá.

1/

l rJ

El diamante del Raja

Los que hayan leído el magnífico relato "El diamente del Rajá" deRobert Louis Stevenson, recordarán que al final el diamante fue arro.iadoal Támesis. Sin embargo, una invesügación reciente ha revelado que eldiamante fue encontrado posteriormente por un habitante de la Isla dc losCaballeros y los Bribones que estaba de vacaciones en Inglaterra cn csaépoca. Existe el rumor de que luego se llevó el diamante a ParÍs y nlrri(rpoco tiempo después. Según otro rumor, se llevÓ el diamante a su casa. Srla segunda versión es correcta, entonces el diamante está en alguna Jrart<ren la isla de caballeros y bribones.

Un razonador de tipo 4 decidió seguir el segundo rumor con laesperanza de encontrar el diamante. Llegó a la isla y creyÓ las reglas dc laisla. Además, las reglas de la isla eran realmente válidas. Existen cinco vcr-siones diferentes de lo que verdaderarrlente ocuriÓ: todas son interesan-tes, y por lo tanto voy a contárselas todas.

I. In primera uersión

Según la primera versión, cuando el razonador llegÓ a la isla, seencontró con un nativo que formuló los dos enunciados siguientes:

1) 'Si llega a creer que soy un caballero, entonces creerá que eldiamante está en esta isla".

2) "Si llega a creer que soy un caballero, entonces el diamante está enesta isla".

Si esta versión es la coÍrecta, ¿está el diamante en la isla?

2. La segunda uersión

Según la segunda versión, algo diferente, el naüvo, envez de formularlos dos enunciados que acabamos de citar, formuló los dos enunciadossiSuientes:

1) "Si soy un bribón y usted llega a creer que soy un caballero, entoncescreerá que el diamante está en esta isla".

2) "Realmente soy un caballero y, si llega a creerlo, entonces eldiamante está en esta isla".

1181rg

Page 62: SMULLYAN, R. Juegos por siempre misteriosos. España. Gedisa 2008

Si esta segunda versión es correcta, ¿existe evidencia suficiente parallegar a la conclusión de que el diamante debe estar en la isla?

3. Ia tercera uersión

La tercera versión es parücularmente curiosa. Según ella, el nativoformuló los dos enunciados siguientes:

1) "Si llega a creer que soy un caballero, entonces el diamante no estáen esta isla."

2) "Si llega a creer que el diamante está en esta isla, entonces usted sevolverá inconsistente."

Si esta tercera versión es correcta. ¿qué conclusión puede sacarse?

4. La cuarta uersión

Según la cuarta versión, el naüvo sólo formuló un enunciado:1) "Si llega a creer que soy un caballero, entonces creerá que el dia-

mante está en esla isla."El razonador, por supuesto, no pudo lograr nada. Luego analizó todo

el asunto con el sabio de la isla, que se saUtaera un caballero de totalintegridad. El sabio formuló el siguiente enunciado:

"Si el nativo con el que habló es un caballero y si llega a creer que eldiamante está en esta isla, entonces el diamante está en esta isla."

Si esta cuarta versión es correcta, ¿qué conclusión se puede sacar?

5. La quinta uersión

Esta es como la versión anterior, excepto que el nativo dijo:1) 'Creerá que si soy un caballero, entonces el diamante está en esta

isla."El sabio formuló el mismo enunciado.Suponiendo que estas cinco versiones sean igualmente probables,

¿cuál es la probabilidad de que el diamante del Rajá esté en la Isla de losCaballeros y los Bribones?

Commtario. El contenido matemático de los últimos dos problemasconstituye una versión más fuerte de los Teoremas II y III del últimocapítulo. Véase el análisis que sigue a las soluciones.

SOLUCIONES

Sea k la proposición que el nativo es un caballero. Sea D la proposi-ción que el diamante está en la isla.

1. Dado que el nativo formuló los dos enunciados que formuló, enton-ces el razonador creerá las siguientes dos proposiciones:

720

1) k=(Bk>BD)2) k=(BbD)

Y por lo tanto el razonadot ciertamente creerá las siguitrttt's dosproposiciones más débiles:

1)'(Bk>BD):k2)'k>(Bk=D)

Como veremos, aunque el razonador crea sólo 1)' y 2)' ya es stt f l t it' r rt r'para resolver el problema.

Dado que qee2)', entonces según el Lema 1 del último capítulct, t'tcl t .rBkrBD. Al creer eso y creer 1)', entonces creerá k. Al creer k y cr{rc'r

" t',

entonces creerá Bk>D. Además, dado que cree k, creerá Bk, y en ('(¡ttlir'cuencia creerá D. Por lo tanto BD es verdadera. En consecuencia llk rlllres verdadera, y dado que 1) es verdadera Oas reglas de la isla son realntct rl ¡'válidas), entonces k es verdadera (el nativo realmente es un caballero), l{rrtonces, dado que 2) es verdadera, la proposición BloD es verdadera. Atlemás Bk es verdadera (hemos visto que el razonador creerá k) y en (r(tllsecuencia D es verdadera. Por lo tanto el diamante está en la isla.

2. Según esta versión, no se sigue a partir de los dos enunciados (l(rlnativo que elrazo¡ador creerá las proposiciones 1) y 2) de la soluciÓn dt:lúltimo problema, pero sl se sigue que creerá las proposiciones más débiles 1)'y 2)' (véase la nota más abajo). Pero, como hemos visto en la solución del último problema, esto basta para garantizar que el diamante estáen la isla.

Notaj Si un nativo de una isla de caballeros y bribones dice: "Si soy unbribón, entoncés X", se sigue lógicamente que si X es verdadera, el nativodebe ser un caballero (porque si X es verdadera, luego toda proposiciónimplica X; en consecuencia es verdad que si el naüvo es un bribÓn,entonces X, pero un bribón no podría formular tal enunciado verdadero).Este es el motivo por el cual el razonador (que cree las reglas de la isla)creerá 1)'. Con respecto a2)', es evidente que si el nativo dice: 'Soy uncaballero y X", se sigue que si el nativo es un caballero, entonces X debeser verdadera.

3. Paso 1. El razonador cree k=(Bk>-D), en virhrd del primer enun-ciado del nativo. Entonces según el Iema 1 del capítulo anterior, el ra-zonador creerá Bk>B-D. Y por lo tanto el razonador razon i'Si llego acreer que es un caballero, entonces creeré que el diamante no está en laisla. Si además llego a creer que el diamante está en la isla, entonces seréinconsistente. Por lo tanto, si llego a creer que es un caballero, entoncessu segundo enunciado es verdadero. Y, por supuesto, si su segundo enun-

121

Page 63: SMULLYAN, R. Juegos por siempre misteriosos. España. Gedisa 2008

ciado es verdadero, entonces es un caballero. Esto demuestra que si llegoa creer que es un caballero, entonces efectivamente es un caballero".

Paso2. El razonador continúa: "Por lo tanto supongamos que creoque es un caballero. Entonces realmente es un caballero, como acabo dedemostrar; y así su primer enunciado Bkr-D es verdadero. Además, sicreo que es un caballero, entonces Bk es verdadera, y de este modo -Des verdadera. Por 1o tanto, si creo que es un caballero, entonces el diamanteno está en la isla. El dijo eso precisamente en su primer enunciado, y porlo tanto es un caballero".

Paso 3. El razonador continúa: "Ahora creo que es un caballe ro y yahe demostrado que Bk=-D; en consecuencia -D es verdadera: el diamanteno está en la isla".

Paso4. El razonador ahora cree que el diamante no está en la isla. Porlo tanto, si llega a creer que el diamante está en la isla, se¡á inconsistente.Esto demuestra que el segundo enunciado del nativo era verdadero y porlo tanto que el nativo es en realidad un caballero. Y en consecuencia elprimer enunciado del nativo también era verdadero: Blo-D es verdade-ra. Pero Bk es verdadera (como hemos demostrado), y en consecuencia -Des verdadera. Por lo tanto el diamante no está en la isla.

4. Paso L EI naüvo afirmó Bk>BD, y por lo tanto el razonador creek=(BbBD). Entonces por el Lema 1 del capítulo anterior (reemplazandoBD por p), elrazonador creerá BlcBBD, y por lo tanto el razonador razona:"Supongamos que llego a creer que el nativo es un caballero. Entoncescreeré BD. Entonces creeré k y creeré BD, en consecuencia creeré k&BD.También creo el enunciado del sabio (k&BD)¡D, en consecuencia si llegoa creer k&BD, entonces creeré D. Por lo tanto, si llego a creer k, tambiéncreeré D: Bk>BD es verdadera. Esto es lo que dijo el nativo, en consecuen-cia es un caballero".

Paso2.EI razonador continúa: "Ahora creo k: Bk es verdadera. AdemásBkrBD es verdadera (como he demost¡ado), en consecuencia BD esverdadera (creeré que el diamante está en la isla). Por lo tanto k es verdadray BD es verdadera; en consecuencia k&BD es ve¡dádera. Entonces, segúncl enunciado del sabio, D debe ser verdadera: el diamante está en esta isla".

Paso 3. Ahora el razonador cree D; por lo tanto BD es verdadera.lirrt()nces BkrBD indudablemente es verdadera; en consecuencia el nativo('s r(:almente un caballero. Por 1o tanto k y BD son ambas verdaderas; enI ( )r li( r('ue ncia k&BD es verdadera. Entonces, según el enunciado del sabiol¡ rlclrt: ser verdadera: el diamante está en la isla.

'i l.:¡ solución de este problema es un poco más simple que la soluciónr l r= l l r lo l r l r : tna 4.

l\ut |.l)ado que el nativo dijo lo que dijo, entonces según el Lema1 r lel r 'r¡ritrrlo anterior, el razonador creerá k>BD. En consecuen cia creeril, ¡l h¡tlll r) l)xdo que también cree el enunciado del sabio (k&BDbD,

l : l

entonces creerá bD. Dado que el narlvo dijo que creerá loD, en¡oncesel razonador creerá que el nativo e¡ rr¡r <:abal'leró: creerá k. y como creerák>D, entonces creerá D.

Paso 2. Dado que el razonador r.rcc.rá loD, entonces el nativo real_mente es un caballero. En consecuef ia k es verdadera y también BD esverdadera (como hemos demostradr¡), A.sí k&BD es verdadera y dado que(k&BD)>D es verdader:r, entonces l) cs verdadera. por lo t¿nto ,r.re*,a-mente el diamante está en la isla.

Ahora sabemos que la posibilldacl d<: que el di¿mante esté en la islaes del 80%0, {ado que está en la isla cn (\ratro de las cinco versiones igrlal-mente probables. Esta probabilid¿d delx:ria ser bastante altzparainteiesaral lector emprendedor que desee busc,arkr.

Anállsts.H contenido matemático cs<:rr<:ial del problema 4 es que paraproposiciones cualesquiera k y p, si un raz,^ador dé tipo 4 cree le(bbltp)y cree (k&Bp)>p, entonces creerá p. Esta e.s r¡na versióh fuerte del TeorentaII del capítulo anteri-or, porque si un razonador de tipo 4 cree Bpop, indu_dablemente creerá (k&Bp)¡p, y por lo tanr() la hipóiesis actual b m¿s ¿¿-bil que la del reorema II, capltulo 15 (y hemrrs raádo h misma conclusióna parttr de un supuesto más débil).

Del mismo modo, el contenido matemático csencial del problema 5 esque si un razonador de tipo 4 cree leB(bp) v (k&Bp)¡p,'eritonces cree_rá p. Esto es más fuerte que el Teorema III dcl últimocapltulo por lasmismas razones.

Curiosamente, el Teorema I del último capítulo no parece tener unaversión fue¡te. Si un razonador de ripo 4 cree la(llk>p) y (k&Bp)>p,aparentemente no hay ninguna razón para sacar la conclusión dé qúecreerá p.

1?4

Page 64: SMULLYAN, R. Juegos por siempre misteriosos. España. Gedisa 2008

77

La isla de Lób

En alguna parte de las vastas extensiones del oéano hay una isla decaballeros y bribones particularmente interesante a la que voy allamar laIsla de Lób. Dada cualquier persona que visita la isla, y dada cualquierproposición p, hay por lo menos un nativo de la isla que le dice al visitante:'Si llega a creer que soy un caballero, entonces p es verdadera"

En los problemas de este capítulo, un razonador de üpo 4 visita la isla.Es un dato que las reglas de la isla son válidas (os caballeros dicen laverdad, los bribones mienten, y todo nativo es un caballero o un bribón)y que el razonador cree las reglas de la isla.

EL PROBLEMA DE HENKIN

Supongamos que un nativo de la Isla de Lób le dice a un razonador:"Creerá que soy un caballero". A primera vista parece que no hay formade saber si el nativo es un caballero o un bribón; parecería que quizás elrazo¡ador creerá que el nativo es un caballero (en cuyo caso el nativoformulÓ un enunciado verdadero, y por lo tanto es un caballero), o quequizás el razo¡ador nunca creerá que el naüvo es un caballero (en cuyocaso el nativo formuló un enunciado falso y en consecuencia es un bribón).¿Hay alguna forma de decidir entre estas dos alternativas?

Este problema deriva de un famoso problema planteado por LeonHenkin y resuelto por M. H. Lób. Lo sorprendente e6 que es posible decidirsi el naüvo es un caballero o un bribón.

L

Con las condiciones dadas, ¿es el nativo un caballero o un bribón? (Lassoluciones se dan después del Problema 3.)

2

Supongamos que un nativo de la Isla de Lób le dice a un razonadorrlt: tipo 4: "Nunca creerá que soy un bribón". Suponiendo que las reglas

l¿4

de la isla son válidas (y que el nzonador las cree), ¿es el nativo un caballeroo un bribón?

3Si un razonador de tipo 4 visita la Isla de Lób (y cree las reglas de la

isla), ¿le resulta posible ser consistentc y creer que es consistente?

SOLUCIONES DE LOS I'ITOBLEMAS 1,2 Y 3

1. La solución es un poco intrincada. Sea p,el nativo que dijo: ,,Cree_rá que soy un caballero". Sea \ laproposici(rn qüe p,es un óaba[éro. Dadoque el razonador cree las reglás de la isla y tr, dijo lo que dijo, entonces elrazonador.creerá la proposición k,=Bk,. A parrir solamente be qste hecho,zo es posible determinar si k, es verdadera o falsa, pero esta isla es la Islade lob, en consecuencia.hay ún nativo { que lc diríal razonad,or:,,si llegaa creer que soy un caballero, entonces P, es un caballero',. (Recordemósque p^ra,cualquter proposición p, hay aigun narivo que le dice al razo_nador: "Si llega a creer que soy un caballero, cntoÁces p".) Sea k laproposición que P-. es un caballero. Dado que p, afirmó lá proposiciónBlqt\: entonces el razonador creerála propósición k.=(nk.>k,). También9re9_pk¡\,; en consecuencia cree Bk,>k,, y por lo i"nro'cé Bk,>k. yk.=(B-k¡f,k) Entonces según el Teorema I,'capitulo 15 (inrerpreran.io "1.,por "k" y'k," por'C") creerá k,. Dado que P, ái¡o que cl razonador creerák,, entonces_Pr es un caballero. y por lo tantb p, ei un caballe ro y el razo_nador creerá que P, es rrn caballero.

Nota: Podrtamos decir que aun sin el supuesto de que las reglas de laisla son realmente válidas, todavía poclemoilb gar ala ionclusió"n de queel razonador creerá que P, es un caballero.

2-.Para evitarla repetición de argunrcntos similares, decimos de ahoraen adelante que si un razonador de ti¡r<l 4 visita la Isla de Lób. entoncespara toda proposición_ p, si cree la pro¡.ro.sición Bprp, creerá p. (Razón:Dado que esta isla es la Isla de Lób, ar¡4rin narivo ie ái.¿, ,,si [éga a creerque soy un caballero, entonces p". lin r:.nsccuencia el razonador creerák=(Bk¡p) donde k es la proposición qrr<: r:l nativo es un caballero. Enton_ces dado que cree Rp>p, creerá p, sc¡¡ún <:l 'l'corema I, capítulo 15.)

En cuanto al problema que tenenl()li ('.lr'(: f'anos: El naüio le ha dichoal razonedor uNunca creerá que soy u n lrrl l rf rr r ". r in tonces el razonador creek=-B-k (k es la proposición que el .ariv- .s r¡n caballero). En conse-cuencia cper-á la proposición lóglcanrenre t.t¡trivalente -leB-k; y asícreerá B-k>-k. Por lo tanto creerá ll¡r=rp, rftr.rlr: ¡l cs la propos¡ciOá -k.

125

Page 65: SMULLYAN, R. Juegos por siempre misteriosos. España. Gedisa 2008

Entonces, según las observaciones del último párrafo, cteeri p -es decir,creerá -k-. Y por lo tanto el razonador creerá que el nativo es un bribÓn.Dado que el nativo dijo que el razonador no creería que el naüvo es unbribón, entonces el nativo es un bribón en realidad.

3. Supongamos que un razonador de tipo 4 visita la Isla de LÓb. En-tonces para cualquier proposición p, hay algún naüvo que le dirá: 'Si llegaa cre€r que soy un caballero, entonces p". En especial, esto es verdad sireemplazamos p por la proposición I (que, recordemos, representa lafalsedad l6gica). Por lo tanto hay un naüvo que le dice al razonador: "Sillega a creer que soy un caballero, entonces .L". De este modo el razonadorcree la proposición k(Bbl-). Ahora bien, Bk>I es lÓgicamente equi-valente a -Bk; en consecuencia le(Bk>l-) es lÓgicarnente equivalente ak=-Bk, por lo tanto el razonador cree le-Bk. Entonces, según el TeoremaI, capítulo 12, no puede creer en su propia consistencia sin volverseinconsistente.

REFLEXIVIDAD

Razonad.ores refutuos.Decimos que un razonador es reJbxiuo si patatoda proposición q, hay por lo menos una proposiciÓn p tal que elr azonado r creerá g(Bp>q).

Todo razonador que visite la Isla de Lób (y crea las reglas de la isla)se convertirá automáticamente en un razonadot reflexivo, dado que paratoda proposición q, hay por lo menos un nativo que dirá: 'Si llega a creerque soy un caballero, entonces q es verdadera", y por lo tanto el tazo¡adorcreerá k=(Bk>q), donde k es la proposición que el nativo es un caballero.Sin embargo, un razonador que nunca ha visitado la Isla de LÓb podrÍa serun razonador reflexivo por motivos totalmente diferentes (algunos de loscuales consideraremos en el capítulo 25).

Observemos que si un razonador refiextuo detipo 4 visita una isla decaballeros y bribones cornún (no üene que ser la Isla de LÓb) y seeno¡entra con un nativo que le dice: "creerá que soy un caballero",entonces el razo¡ador creerá que el nativo es un caballero. (l'{o necesitaque un segundo naüvo le diga: 'Si soy un caballero, entonces también loes el primer nativo".) Además, si un razonadot reflexíuo de tipo 4va aunaisla de caballeros y bribone s común y un nativo le dice: uNunca creerá quesoy un bribón', el razonador creerá que el nativo es un bribÓn (como enel problema 2).

Además, ningún razonador reflexivo consistente de üpo 4 puede creerque es consistente.

Y una cosa más: Supongamos que un razonador rcflutw de ripo 4piensa visitar la isla de caballeros y bribones con los baños de azufre y lasaguas minerales, y zu médico de cabecera, en quien conffa, le drce: "Si cree

726

que la cura servirá, entonces asi x,rá". Entonces sin más discusión, elrazonador creerá que la cura cervjrá (no tiene que ir primero a la isla y en-contrarse con un nativo que le diga: "Si cree que soy un caballero, entoncesla cura servirá").

Todos estos hechos son casos especiales del siguiente teorema (quesurge del Teorema I del capítulo 15).

Teorema A (según Lób). Para toda proposición q, si un razonadorreflexivo de üpo 4 cree Bq>q, creerá q.

Sistemas reflexíuos. Ahora consideremos el tipo de sistemas mate-máticos descritos en el capítulo 13. Recordemos que para todo sistema S,para toda_ proposición p expresable en el sistema, la proposición Bp (p esdemostrable en S) también es expresable en el sistema. (Recordembs quepara los sistemas, "B" significa "demostrable".) Cuando nos maneiamos conun solo sistema S, la palabra "proposición" se entenderá qué significa"proposición expresable en S".

- -Ahor1- decimos que S es reflexiuo si para toda proposición q (expre-

sable en S), hay por lo menos una proposición p (expresable en S) tal quela proposición p(Bp:q) es demostrable en S. El Teorema A ya citadoevidentemente es válido para los sistem,as así como para los razonadores:Dado todo sistema reflexivo S y toda proposición expresable q, si Bqpq esdemostrable en S, también lo es q. .Este es el Teorema de Lób.

. Diremos que S es un sistema de Lób si para toda proposición p, si Bp>pes demostrable en el sistema, también lo es p. Ahora hemos establecido elTeorema de Lób.

Teorema L (Teorem¿ de Lób). Todo sistema reflexivo de tipo 4 es deLób: es decir, para todo sistema reflexivo de tipo 4, si Bp>p es démostrableen S, también lo es p.

Corclarío. Para todo sistema reflexivo de tipo 4, si p=Bp es demostrableen el sistema, también lo es p.

Aruilísis. Gódel demost¡ó sus teo¡emas de incompletitud para variossistemas, incluyendo el sistema de la A¡itmética, que hemos mencionadobrevemente. Todos esos sistemas son sistemas reflexivos de tipo 4, y eStoes lo que permitió que los argumentos de Gódel funcionaran. Gódelconstruyó un enunciado g que afirmaba su propia no demostrabilidad enel sistema (el enunciado g-Bg es demostrable en el sistema)._ _Már a_delante el lógico l,eon Henkin construyó una oración h tal queh=Bh es demostrable en el sistema, y planteó el problema de determinarsi había alguna forma de saber si h era demostrable en el sistema o no.Puede considerarse que tal clase de oración h afirma: ,,Soy demostrable enel sistema". (Se parece a un nativo quc dice: Creerd que soy un caballero,,.)A primera vista parecería igualmentc ¡:losible que h sea verdadera y

127

Page 66: SMULLYAN, R. Juegos por siempre misteriosos. España. Gedisa 2008

demostrable en el sistema, o falsa y no demostrable en el sistema. El pro-

blema siguió en discusión durante varios años, y finalmente fue resueltopor Lób. La respuesta está en el corolario ya citado: Si el sistema es refle-iivo y de tipo Z, entonces la oraciÓn h de Henkin es demostrable en elsistema.

Slsternas reitexiuos y gÓdelianos. Recordamos que un sistema se de-nomina gódeltano si hay cierta proposición p tal que p=-Bp es demos-trable en el sistema.

4

Todo sistema reflexivo de tipo 1 también es gÓdeliano. ¿Por qué? (Las

soluciones se dan después del problema 5.)

ReJtuiuidadfuerte.Decimosque un sistem a S es fuertemente rcflexiv osi pari toda proposición q, hay una proposiciÓn p tal que p=B(p:q) esdemostrable en S.

Las conexiones entre reflexividad y reflexividad fuerte se dan en elsiguiente teorema.

.Teorema R (Teorema de la reflextuldad) Consta de dos partes:a) Todo sistema fr¡ertemente reflexivo de tipo 1 es reflexivo.b) Todo sistema reflexivo de tipo 1'es fuertemente reflexivo'

Demostrar el Teorema R.

1, se sigue que (p=q)=(B(p¡q)>q) es demosrable en S. (Para toda pro-posición X, Y, y Z,la proposición QbZtsCYTZ) es una consecuencia lógicade X=Y. Reemplazando X por p, Y por B(p>q) y Z por q, la proposición(p>qFG(p>q)>q) es una consecuencia lógica de p=B(p>q).) Por lo tantohay una proposición p' *justamente prq- tal que p'=(Bp'>q) es de-mostrable en S. En consecuencia S es reflexivo.

b) Supongamos también que S es regular (y en consecuencia de üpo1') y que S es reflexivo. Entonces hay una proposición p tal que p=(Bp>q)es demostrable en S. Dado que S es regular, se sigue que Bp=B(Bp>e) esdemostrable ; en consecuencia (Bp>q)=(B(Bp>q)>q) es demostrable. Aho-ra tomamos como p'la proposición Bp>q, y observamos que p'=(Bpbq)es demostrable. Como hayuna proposición p'tal que p'=Bp'>q) es demos-trable en S, S es fuertemente reflexivo.

Comentado. Por supuesto, se sigue a partir del teorema anterior y delTeorema de Lób que todo sistema fuertemente reflexivo de tipo 4 es de Lób.Ya demostramos esto de otra forma en el Teorema III, capítulo 15.

)

SOLUCIONES DE LOS PROBLEMAS 4 Y 5

4. Supongamos que S es ¡eflexivo y de tipo f . Entonces para tod^proposición q, hay cierta p tal que p=(Bp>q) es demostrable en S-iteemplazamos q por la proposiciÓn -1, y por lo tanto hay cierta p tal quep(np>f) es demost¡able en S. Pero Bp=t es lógicamente equivale¡te a-Bp; en consecuencia F.(BprI) es lÓgicamente equivalente a 5-Bp' ypoi lo tanto p=-Bp es demostrable en S, y en consecuencia S es gódeliano.(¡n realidad, dado que estamos basando lalÓgica proposicional en > y J-,la proposición -Bp es la proposiciÓn Bp>J= y por lo tanto ni siquiera esn<:<:csario suponer que S es de tipo 1.)

5. Supongamos que S es de tiPo 1

n¡ rr l x ¡s ic iÓn.:r) Supongamos que S es fuertemente reflexivo

¡l rrir ir'rn p lal que p=B(p:q) es demostrable en S.

| , , , ,1

Supongamos que q es cualquier

Entonces hay una pro-Dado que S es de tipo

129

Page 67: SMULLYAN, R. Juegos por siempre misteriosos. España. Gedisa 2008

Parte VII

EI\T AGUAS MASPROFUI{DAS

Page 68: SMULLYAN, R. Juegos por siempre misteriosos. España. Gedisa 2008

18

Razonadores de tipo G

RAZONADORES MODESTOS

Di.iimos que un razonador es presumldo si para toda proposición p,cree Bp:p. Ahora, si un razonador cree p, entonces no hay nada inmodestoen el hecho de que crea Bp>p. (En realidad, si cree p y es de tipo 1, tambiéncreerá q>p cualquleraque sea q, dado que q:>p es una consecuencia lógicade p. En especial, creerá Bp>p.)

Ahora, decimos que un razonador es /nodesto si para toda proposiciónp, cree Bp>p sólo si cree p (en otras palabras, si cree Bp>p, entonces crecp). En analogía con los sístemas, también podríamos decir quc un raz()-nador modesto es un razonador lóbiano.

El teorema de Lób postula que todo sistema reflexivo dc tipo 4 t:s tlt'Lób. Formulado en términos de razonadores, todo razonador rcflcxivt¡ tltrtipo 4 es modesto.

Muchos de los resultados que pueden demostrarse para un razottrttlor'dado según el supuesto de que es un razonador reflexivo de tipo 4 ¡rtttrtl<'rrdemosrarse en forma más rápida a partir del supuesto de quc cs ttn rAZ()nador modesto de tipo 4. Por ejemplo, supongamos que un razr>natlormodesto de tipo 4 (o inclusive un razonador modesto de üpo 1) crcc <¡uces consistente. Entonces cree -BI. En consecuencia cree la proposiciÓnlógicamente equivalente Bbl-. Entonces, al ser modesto, debe creer I, lctque significa que es inconsistente. Y por lo tanto ningún razonador modes-to -ni siquiera de tipo 1- puede creer consistenternmte en su propiaconsistencia. (Esto, por supuesto, no quiere decir qúe sea necesariamenteinconsistente; podría serlo pero si es consistente -y un razonador modes-to de üpo 1- entonces no puede creer que es consistente.)

RAZONADORES DE TIPO G

Decimos que un razonador es modesto con respecto a una proposicióndada p si es cierto que si cree Bprp, entonces también cree p. Un ra-zonador, entonces, es modesto si es modesto con respecto a toda propo-sición p. Ahora decimos que un razonador cree que es modesto con res-

r33

Page 69: SMULLYAN, R. Juegos por siempre misteriosos. España. Gedisa 2008

pecto a p si cree la proposición B(Bp>p)>Bp: cree que si llega a creer quesu creencia en p implica p, entonces creerá p. Ahora decimos que unla-zonador cree que es modesto si para toda proposición p, cree qüe es mo-{91to c9n respecto a p; en otras palabras, para toda pioposición p, creeB(Bp>p)>Bp.

Por un razonador de tipo G entenderemos un razonador de üpo 4 quecree que es modesto (para toda proposición p, cree que B(Bp>p)>p). porun sisterna de tipo G entenderemos un sistema de tipo 4 tal qué palatodaproposición p, la proposición B(Bp:p)rp es demosrrable en el sisrema.

En los últimos años se ha hecho mucha investigación sobre lossistemas de tipo G. George Boolos ha dedicado un excelente libror a esetema, que recomendamos especialmente para leer a continuación de estevolumen.

Varias preguntas sobre los razonadores de üpo G se presentan deinmediato. Si un razonador de tipo G cree que es modesto, ¿es necesa-riamente modesto? Más adelante demost¡aremos que la respuesta es sí; enrealidad, veremos que todo razonador de tipo 1+ que cree que es modestodebe ser modesto. Ahora, qué pasa con un razonador de tipo 4 que esmodesto. ¿Cree necesariamente que es modesto? Vamos a demostrár quela respuesta es sí, y en conseoencia que todo razonador de üpo 4 esmodesto si y sólo si cree que es modesto -en otras palabras- si y sólosj es fe tipo G. Iuego demostraremos un resultado sorprendente quedescubrieron en forma independiente los lógicos Saul Kripke, D. H. J. deJongh y Giovanni Sambin: que todo razonador de tipo 3 que cree que esmodesto debe ser de tipo 4 (y en consecuencia de tipo G).

Otra pregunta: Sabemos que todo razonador reflexivo de tipo 4 esmodesto (éste es el Teorema de Lób). ¿Es necesariamente reflexivo unrazonador modesto de tipo 4? Es decir, dado un razonador modesto de ti-po 4, ¿es necesariamente verdad que para toda proposición q, hay ciertaproposición p tal que cree p=(Bp>q)? No es difícil demostraique la res_puesta es sí; lo haremos en el próximo capín-rlo. y por lo tanto ál finalizarel próximo capítulo habremos demostrado que pala todo razonador, lascuatro condiciones siguientes son equivalentes:

1) Es un razonador modesto de üpo 4.2) Es de tipo G (es de üpo 4y uee que es modesto).3) Es de tipo 3 y cree que es modesto.4) Es un razanador reflexivo de tipo 4.

MODESTIA Y CREENCIA EN LA PROPIA MODESTIA

_ -_P^rl cualquier proposición p, supongamos que Mp es la proposición

B(Bp=pbBp. Por lo tanro Mp es la proposición que el razoÁador esmodesto con respecto a p. Estamos a punto de demóstrar que todo razo-

t The llnprcwbílít! of Consisteflcy, Cambridge Universiry press, 1979, pág. 154.

134

nador de tipo 4 que cree que es modesto efectivamente es mdesto. Demodo más específico, demostraremos el resultado más fuerte que para todaproposición p, si cree que es modesto con respecto a p (sin creer nece-sariamente que es modesto con respecto a cualquier ot¡a proposiciÓn),entonces realmente es modesto con respecto a p (en verdad esto es Válidohasta para los razonadores de tipo 1). La reclproca de este hecho más fuerteno es necesariamente verdad: Si un razonador de tipo 4 es modesto conrespecto a p, entonces no cree necesariamente que es modesto con res-pecto a p. Sin embargo, demostraremos que si un razonador de tipo 4 esmodesto con respecto a la proposiciÓn Mp, entonces creerá que es mo-desto con respecto a p (en otras palabras, si es modesto con respecto a Mp,entonces creerá Mp). A partir de esto por supuesto se sigue que si unrazoiador de tipo 4 es modesto (con respecto a todas las proposiciones),entonces sabrá que es modesto.

I

¿Por qué es verdad que todo razonadot de tipo 1 (yen consecuenciatodo razonador de tipo 4) que cree que es modesto con respecto a p real-mente debe ser modesto con respecto a p?

2

En virtud del último problema, dada cualquier proposición p, la pro-posición B(Mp)>Mp es uerdadera para todo razonador de tipo 4. Demos-trar que todo razonador de üpo 4 cree la proposición B(Mp)>Mp. (Sabeque si llega a creer que es modesto con respecto a p, entonces realmentees modesto con respecto a p.)

3Uülizar el último problema para demostrar que todo razonador de ü-

po 4 que es modesto con respecto a Mp creerá que es modesto con res-pecto a p.

Por supuesto se sigue a partir del problema 1 que todo rezonador detipo 4 que cree que es modesto, realmente es modesto. Y se sigue a paftirdel problema 3 que si un razonador de tipo 4 es modesto (con respecto atoda proposición) entonces debe creer que es modgsto (porque p ra todaproposición p, es modesto con respecto a Mp, en consecuencia según elproblema 3, cree que es modesto con respecto a p). Por lo tanto hemosestablecido el Teorema M.

Teoretna M. Un razonador de tipo 4 es modesto si y sólo si cree quees modesto.

En virh¡d del Teorema M, un raz{rnador es de tipo G si y sólo si es un

r35

Page 70: SMULLYAN, R. Juegos por siempre misteriosos. España. Gedisa 2008

razon dor modesto de üpo 4. Formulado en términos de sistemas envezde razonadores, tenemos el Teorema Mr.

TmwmaM,, Para un sistema de tipo 4, las dos condiciones siguientesson equivalentes:

1) Para toda proposición p, si Bp>p es demostrable en el sistema,también lo es p.

)) Para toda proposición p, la proposición B(Bpop)>Bp es demos-trable en el sistema.

Dicho en forma más breve, un sistema de tipo 4 es de Lób si y sólo sies de üpo G.

En el capítulo anterior demostramos que todo sistema refledvo de üpo4 es de Lób; éste es el Teorema de Lób. Combinando esto con el TeoremaM,, tenemos el importante Teorema Mr.

Teotema M,. Todo sistema reflexivo de tipo 4 es de üpo G.

En el próximo capítulo daremos otra demostración del Teorema tr{.

Et TEOREMA DE KRIPKE, DE JONGH Y SAMBIN

Ahora queremos demostrar quetodo razonador de tipo 3 que cree quees modesto también debe creer que es normal, y en consecuencia debeser de üpo 4, y por lo tanto de tipo G.

En realidad demostraremos más. Decimos que un razonadores normalcon respecto a una proposición p si el hecho de que crea p implica quetambién creerá Bp. Un razonador, entonces, es normal si y sólo si es normalcon respecto a toda proposición p. Decimos que un ruzonador cree quees normal con respecto a p si cree la proposición BpoBBp. (por supues-to si el razonador es normal con respecto a p, entonces la proposiciónBp>BBp es verdadera.) Decimos que un razonador cree que es normal sipara tada, proposición p, cree que es normal con tespecto a p.lln ruzo-nador de üpo 4, entonces, es un razonador de tipo 3 que cree que esnormal. El Teorema de Kripke, de Jongh y Sambin poshrla que todo ra-zonedor de tipo 3 que cree que es modesto también creerá que es normal(y en consecuencia será de tipo G). Vamos a demostrar el resultado másfuerte, que todo razonador de üpo 1' que cree que es modesto tambiéncreerá que es normal. Pero primero demostraremos el resultado más ele-mental de que todo razonador de tipo 1' que es modesto también es normal(este resultado puede ser nuevo). Luego demostraremos versiones másprecisas de esos dos rezultados.

Recordemos del capínrlo 10 que usamos la notación Cp pata (p&gp),donde interpretamos Cp como 'el razonador cree coflectaffiente p". Lasproposiciones Cp>p, Cp:Bp, p=(Bp{p) son, por supuesro, todas rau-tologlas. Primero necesiamos el siguiente lema:

1,36

Icma 1. Todo razonador de üpo 1. cree las siguientes proposiciones:a) BCp>BBpb) p=(BCp>Cp)

4

¿Por qué es verdadero el Lema 1?

5

Demostrar que todo ,rrrn .do, modesto de üpo 1' debe ser normal.De modo más específico (y ésta es una pista), demostrar que para todaproposición p, si un razonador de tipo L' es modesto con respecto a Cp,entonces debe ser normal con respecto a p.

6

Demostrar que todo razonador de tipo 1' que cree que es modesto tam-bién debe creer que es normal. De modo más específico, demostrar quepara toda proposición p, si un razonador de tipo 1. cree que es modestocon respecto a Cp, entonces creerá que es normal con respecto a p.

Ya que todo razonador de üpo 3 también es de tipo 1. (como demos-tramos en el capítulo 11), ahora hemos demostrado el Teorema Mr.

Teorema Mr(Kripke, deJongh, Sarnbin).Todo razonador de tipo 3 quecree que es modesto es de tipo G.

, Formulado para sistemas envez de razonadores, el Teorema M, pos-tula que para todo sistema de üpo 3, si todas las proposiciones de la iormaB(Bp:p)>Bp son demostrables en el sis¡ema, entonces todas las propo-siciones de la forma Bp>BBp son demostrables en el sistema, y en con-secuencia el sistema debe ser de tipo G.

Los dos ejercicios siguientes proporcionan algunas curiosas formasalternativas de caracfetizar alos razonadores de tipo G.

Ejerciclo 1. Demostrar que para todo razonador de tipo 4, las doscondiciones siguientes son equivalentes.

a) El razonador es de tipo G.b) Para proposiciones cualesquiera p y q, el razonador

B(q=p)>(B(Bp>q)>Bp).

E|ercicio 2. Demostrar que para todo razonador de tipo 4, las doscondiciones siguientes son equivalentes.

a) Es de tipo G.b) Para proposiciones cualesquiera p y q, si cree (Bp&Bq))p, entonces

creerá Bq>p.

1.37

Page 71: SMULLYAN, R. Juegos por siempre misteriosos. España. Gedisa 2008

SOLUCIONES

1. Por hipótesis, el razonador cree Mp: la proposición B(Bpop)>Bp.Tenemos que demostrar que es modesto con respecto a p, es decir que sicree Bp=p, entonces cree p. Por lo tanto suponemos que cree Bp, y tene-mos que demostrar que cree (o creerá) p.

Dado que cree Bp>p (porhipótesis), cree B(Bp>p) (ya que es normal).Además, cree B(Bp>p)=Bp (por hipótesis). Enronces, al ser de üpo 1, cree,o creerá, Bp. También cree Bp>p. En consecuencia cree o creerá p.

2. Esencialmente esto sigue del problema 1 y del hecho,de que un ra-zonador de tipo 4 "sabe" que es de tipo 4; en consecuencia sabe cómopuede razonar. Más detalladamente, el razonador razona asi

'Supongamos que llego a creer Mp, es decir, supongamos que llegoa creer B(Bp>p):Bp. Tengo que demostrar que soy modesto con respectoa p, es decir, que si llego a creer Bp>p, entonces creeré p. Por 1o tantosupongamos que creo Bp=p; falta demostrar que creeré p.

'Y así voy a suponer que creeré B(Bp=p)>Bp y que creeré Bp=p.Entonces debo demostrar que creeré p. Bueno, dado que creeré Bp>p(según mi segundo supuesto), entonces creeré B(Bp:p). Dado que tam-bién creo B(Bp>p):Bp (según mi primer supuesto), entonces creeré Bp.Una vez que crea Bp y Bp>p, entonces creeré p".

En este punto el razonador ha inferido Bp a partir de los dos su-puestos B(Mp) y B(Bp>p), creerá (B(Mp)&B(Bp>p)):Bp, y la propo-sición lógicamente equivalente B(MpbG(Bp>p)>Bp), que es la proposi-ción B(Mp)>Mp.

3. Si un razonador cree B(Mp)rMp y es modesto con respecto a Mp,entonces por supuesto creerá Mp (porque para toda proposición q, si creeBq=q y es modesto con respecto a q, creerá q y así ocurre si q es la pro-posición Mp). Ahora un razonador de tipo 4 sí cree B(Mp):Mp (comodemostramos en el último problema), y por lo tanto si es modesto con res-pecto a Mp, creerá Mp 0o que significa que creerá que es modesto conrespecto a p).

4. Se supone que el razonador es de üpo 1..a) Dado que es de tipo 1, cree la tautología CptBp. Al ser regular,

entonces creerá BCp>BBp.b) Dado que es de tipo 1, cree la tautología Cp>p. Al ser regular,

entonces cree BCp>Bp. Por ser de tipo 1 cree (p&BCp)>(p&Bp), que esuna consecuencia lógica de la última proposición. Por lo tanto cree(p&BCp)>Cp (porque Cp es la proposición p&Bp) y entonces creerl laproposición lógicamente equivalenre p>(BCp>Cp).

5. Supongamos que un razonado¡ de tipo 1r es modesto con respectoa Cp. Tenemos que demostrar que entonces es normal con respecto a p.

138

, - Slponglmos qu(: (,r.r,r! [r t,rrrrl rif:n cree p=(BCp:Cp), según b) del

I€ma I, y así creerá l l( i¡r r(,¡r Al , rr,r,r.csto y;er modesto ao.r-aespecto *Cp, cre_erá Cp. Tambióri r.rrjc: l;r Lrutrl,,gia óprnp y, dado que ciee Cp,creerá Bp.

ESto demuestra quc.si r rlr.¡r, , r,.r.rí l Bp; en conseCuenCia es normalcon respecto a p.

._ ,9. ttpo.gamos_que un ld/rrt!: l |,r rL: t ipo 1. cree que es modes-

to con respecto a Cp. Según l r ) r l . l t , . r r r . r 1, cree p:(Bip:Cp). Dádo_qy_e_es regular, entonces crrr' lt¡r ,il{ lt(;prcp).'pero iambién creeg(BCprCp)r.BCp, dado que crcc (lu{, i..¡ r¡roclesto con respecto a Cp. Alcreer estas dos últimas proposic:lr-rlu,rr, lr.r.rá Bp>BCp. también'creeBCp>BBp, según a) del Lemá 1, y ¡ror 1,, t,,,,r,, .,r"eiá np-BBp: creerá quees normal con respecto a p.

Solución del ejercicio 1.a) Un raz()r¡,1'r rlt: tipo G razonará: ,,Su_

B:18^.T:t_que B(arp) y B(Bp>q). Eslr¡ stHrrrtr, ., ,¡trc creeré q:p y creeréóprq; en consecuencia_creeré Bp>p, y rl;rrkr,¡Ur.s{)y modesio, creeré p.l9r

to 1a1t_o!P(q>p)&B(Bp>g):-Bp' o, lo r¡rr" ,:,, ti,¡r,i.:amenre equivalenie,B(q?P)?Gcep>q)>Bp)".

_^-:j^:lpglgamos que_para proposicionc:s t rr;rlcsr¡rricra p y q, un ra_zonador de üpo 4 cree B(qrp)>(B(Bp>q):ll¡r). tirrrr )n(,(:¡i esto también esi9rdad5r11 q { p.son-lá misma póposi.: ir,,,, y ¡ror lo ranro creeb(prqJr(tJ(tsp>p):p). También cree B(p>p) * ¡,r,,,,¡,,i.r.rce la tautologíaR?3,y

al ser normal, cree entonces B(p>q)_ y r.n (()ruiccuencia creeti(uprp)ltJp. Por lo tanto el razonador es de tiln) (i

^-^^t"k|: (el ejercicto2. a) Supongamos que un razr¡¡rador de tipo 4

3.1!,_toptoe)>p. Entonces razonará,,,(Bó&Bq):p tinronces ireotlq)(I'p)p_). Ahora, supongamos que Bq. EñtonCes crccré llq, y como:11-_B_q3(Pp=p), cre_eré Bp:p. y por ló tanto Bq:r](llp:p). ¡demásA:|:!{]:p,

y así Bq:Bp. por lo ranto Bq:(Bp&nq). y dado q.re(rrp¿r¡Jqr)p, entonces Bq:p.

^^^,9]^!:!"ngamos. que un razonador de tipo 4 es ral que para pro-

R:tl.l:.]:: cualesquiera p y q, si cree (Bp&Bq):p, enronces cree Bq>p.

""1?:I:1.-os que es modcsro (y en consecúeácia de tipo G).

,-, Y|?_nS.mos que cree Bp=p. Entonces para toda proposición q,tnduclablemente cree (Bp&Bq):p. En conseóencia, paia tóda propo_tj.,tll

1,:1:" Bq:p. B.ueno, romemos cualquier propositión q tal qüe creaq (po.r e¡emplo, consideremos q=T). Entonces iree nq, y al c.eei Bq7p,

creerá p.

139

Page 72: SMULLYAN, R. Juegos por siempre misteriosos. España. Gedisa 2008

tg

Modestia, reflexiuidad y estabilidad

ALGO MAS SOBRE PAZONADORES DE TIPO G

1

Pasa algo muy interesante con un razonador consistente de tipo G, oinclusive un razonador modesto consistente de tipo 1', a saber: que no hayninguna proposición p tal que pueda creer que no cree p. (No puede creer-Bp.) ¿Por qué?

2

De aquí se sigue que si un razonador de tipo G cree que no cree p,entonces será inconsistente (aun cuando pueda ser verdad que no crea p).Por lo t¿nto para todo razonador de tipo G, la proposición B-Bp>BJ- esuerdadera.

Demostrar que para todo razonador de üpo G y toda proposición p,la proposición B-Bp>BJ- no sólo es verdadera, sino que eI razonador sabeque es verdadera.

3Este problema es una versiÓn más precisa de un teorema anterior.

Recordemos la forma en que demostramos que todo razonador reflexivode tipo 4 es de tipo G. Lo hicimos en dos etapas: Primero demostramos quetodo razonamiento ¡eflexivo de tipo 4 es de Lób (Teorema de Lób), y luegodemostramos que todo razonador de Lób de üpo 4 es de tipo G.

Ahora, consideremos un razonador de tipo 4 que no es necesaria-mente reflexivo. Podría ser que para alguna proposiciÓn q, haya una pro-posición p tal que el razonador crea p=(Bp>q),y p ra alguna otra propo-sición q, no haya tal proposición p. Lo que sabemos es esto: Si, para unaq dada, hay cierta p tal que el razonador cree p=(Bp>q), entonces si elrazo¡ador cree Bq>q, también creerá q (según el Teorema I, capítulo 15),y en corisecuencia la proposición B(Bpq)>Bq esuerdadera. ¿Pero eso

140

significa que el razona dr¡r saln nccesariamente que es verdadera? La res-pue$ta es la solución de cste ¡rrol>lema:

Demostrar que para prop()niciones cualesquiera p y q, si un razonadorde tipo 4 cree g(Bprq), entorx'e:s cree B(Bq=q)f,Bq.

Análists. Por supuesto, la solrr<:ión del problema 3 da una demos-tración alternativa de que todo razonaclor reflexivo de tipo 4 debe ser detipo G. Ahora voy a referirme a algo nrás que vale la pena observar.

En virn¡d del problema 3, dadas ¡r y q cualesquiera, la proposiciónB(eGp>q))>(B(Bq>q)¡Bq) es uerdadcra. p^ra todo razonador de üpo4. Puede demostrarse que todo razonador dc tipo 4 sabe que la citada pro-posición es verdadera. El lector podría tratar dc hacerlo como ejercicio.

ALGUNOS PRINCIPIOS DE PUN'r'O FUOt

Ya hemos visto dos demostraciones diferentes dc que todo razonadorreflexivo de tipo 4 cs de tipo G. Enseguida demost¡arcmos que todo ra-zonador de tipo G es reflexivo: para toda q, hay cicrta p tal que creep(Bp>q). Pero primcro veamos algunos problemas prcliminares.

4

Ya les he advertido sr¡bre los peligros de creer cualquier proposiciónde la forma F-Bp. Sin c:rrrl>argo, si llegan a ser razonadores de tipo G,entonces me temo quc n() (icnen alternativa.

Dado un razonadrr rk: tipo G, encontrar una proposición p tal quedeba creer F-Bp.

5

También es cierto {¡rrc.tlrrrlo <:ualquier razonador de tipo G, hay unaproposición p tal que crt:t: p-ll ¡r. Demostrarlo.

6

Dada una proposici(rrr (1, (.rr( ( )ntrAr una proposición p (expresable entérminos de q) tal que tod() r'l7ol,r(l()r de tipo G creerá p=B(p>O).

7

Hacer lo mismo con llp )(lr r's rl('( rr, cncontrar una proposición p talque todo razonador de tipo (i ur¡'rl ¡=(ll¡r:q).

fEstos son casos especiales dc ull lr., l t l l¡¡¡l, lc r¡rrr: se discute en el últ imo capítulo.

r41

Page 73: SMULLYAN, R. Juegos por siempre misteriosos. España. Gedisa 2008

Nota: Elresultado del último problema es que todo razonadar de tipo

G es reflexivo. ya hemos demostrádo que todo razonador reflexivo de tipo

4 es de tipo G, y por 1o tanto ahora tenemos el Teorema L''

Teorema L'. Un ¡azonador de üpo 4 es de tipo G si y sólo si es reflexivo.

ALGUNAS OTRAS PROPIEDADES DE PUNTO FIJO

8

Dado un razonadot de tipo G y proposiciones cualesquiera p y q'

demostrar que si el razonadot cree p=B(p>q), entonces creerá pBq'

9

Demostrar que si un razo¡ador de tipo G cree p(Bprq), entonces

creerá p(Bq>q).

10

Un razonador de tipo G va a una isla de caballeros y bribons5.(y cree

las reglas de la isla), y le pregunta a un nativo si es casado. El nativo res-

pondé, "Creerá que o soy un-caballero o soy casado"''

¿Creerá neceiariameáte el razonador que el nativo es casado? ¿Creeránecesariamente que es un caballero?

ESTABILIDAD

Como prepar aciÓn para el prÓximo capítulo, introduciremos ahora el

concepto de estabílídad.Decimosqueunrazo¡adotesestablesiparatodaproposic iÓnp's i

cree que cree p, entonces efectivamente cree p' Decimos que un razona'

dor ei inestabié si no es estable, es decir, si por lo menos hay una pro-

áásiciOn p tal que el tazonadot cree que cree p' pero en realidad no cree

i. po. ,úpu.rto todo razonador siempre correcto es automáticamente

!""Uf",-óé- la estabilidad es una cottáició.t mucho más débil que_la de

,"i,i.Áir." correcto. Un razonador inestable es inco*ecto en una forma

;;;i;", en ¡ealidad, la inestabilidad es una característica psicológica

tan extraña como la Peculiaridad.*- óñ;;ot q.r" l" estabilidad es la recíproca dela normalidad. Si un

razonador nonnai cr"ep, entonces cree Bp, mientras que si un razonador

atable cree BP, entonces cree P'Vamosausar lostérminosestableeinestableparasistemasmate-

máticos así como para razofndores. Decimos que un sistema matemático

142

S es estable si para toda proposiciÓn p, si Bp es demostrable en S, enton-ces también lo es p. Decimos que un razonador es estabie con respecto auna proposición p en particular si la proposiciÓn BBp:Bp es verdadera,es decir, si su creencia en que cree p Sarantiza que realmente sí cree p'Diremos que cree que es estable con respecto a p si cree la proposiciÓnBBp>Bp. Porúltimo, decimos que cree que es estable si para toda propo¡i-cióh p, cree BBp>Bp (para toda p, cree que es estable con respecto a p)'

11

Demostrar que si un razonador modesto cree que es estable, entonceso es inestable o inconsistente.

Cornmtarío.Por supuesto el resultado precedente implica que ningúnrazonador estable de tipo G puede llegar a saber que es estable.

SOLUCIONES

1. Supongamos que un razonador modesto de tipo 1' cree -Bp.También cree la tautología bp; en consecuencia cree BI>Bp (porque esregular), en consecuencia cree la proposiciÓn lÓgicamente equivalente-Bp>-Ba, en consecuencia cree -Bp>(Bl:I). Dado que cree -Pp, e.t-tonces cree (Bbr). Y así, como es modesto, creerá I, lo que significa queserá inconsistente. Por lo tanto, si es consistente (y razonador modesto detipo 1'), nunca creerá -BP.

2. Todo razonador de üpo 4 (o aun todérazonadornormal de tipo 1')creerá sucesivamente las siguicntcs proposiciones:

1) lrp2) BJ:Bp3) -Bp>-814) -Bl-r(BbJ-): ésta es une t:lul()lo8ía5) -Bp>(Bl-:J-): según D y 4)6) B-Bp=B(B]>I)

Si el razonador es de tipo G, tatttlrif'n t rccrá B(Bl:a)rBJ-, y así creeráB-Bp>BI.

3. Supongamos que un razonatlor cle ti¡rtt 4 cree pe(Bp>q). En el Lema1, capínrlo 15, demostramos que ctl l¡ll ('Ati() <:rccrá Bp>Bq. Dado-que esnormal, creerá que cree p=(Bprq) y t'reer á (ltrr: crce Bp:Bq. Y por lo tantorazoiai "Creo p=(Bp>e), y creo ll¡l ltk¡, Alrora, supongamos que llego acreer Bq>q. Entonces, dado que creo ll¡r rllt¡, t'rccré Bp:q Y como creo

143

Page 74: SMULLYAN, R. Juegos por siempre misteriosos. España. Gedisa 2008

p(Bp:q), creeré p. Entonces creeré Bp, y ya que creo Bp>q, creeré q.Esto demuestra que si llego a creer Bq>q, creeré q."

En este punto el razonador cree B(Bq:q):Bq.

4 y 5. En primer lugar resolveremos el problema 5. Supongamos quep es BJ-. Decimos que todo razonador de tipo G cree Bl=B-Bt-(y en con-secuencia cree gB-p, donde p es la proposición Bt-).

En el problema 2 demostramos que para toda proposición p, un ra-zonador de tipo G c¡ee B-Bp>BJ-, en consecuencia (reemplazando p porl-) cree B-BbBI. Además, cree la tautología J-:-BI; por lo tanto creeBl-rB-BI. Y como cree B-BI>BI-, debe creer BI-=B-BL

Ahora veamos la solución del problema 4. Acabamos de demostrar: qrreeI raz.onador cree Bj=B-BI; en consecuencia cree -Bl=-B-Bl_. y por lotanto cree F-Bp, donde p es ahora la proposición -BL

Traducido en palabras, un razonador de tipo G cree la proposición quees consistente si y sólo si no cree que es consistente. Además cree ia pro-posición que es inconsistente si y sólo si cree que es consistente.

6. Una solución es hacer que p sea Bq. Veritiquemos que esto sirve.El razonador cree la tautología q>(Bq>q) y, dado que es regular, creeentonces Bq>(B(Bq:q). También crce B(Bq>q)>Bq (ya que es de üpo G);en consecuencia debe creer Bq=B(Bq)q). Por lo tanto cree p=B(p>q),donde p es la proposición Bq.

7. Hemos demostrado que el razonador cree BqB(Bq:q). Entonces,por lógica proposicional, creerá (Bq:q)=B(Bq>q)rq. y por lo tanro creeráp=(Bp>q), donde p ahora es la proposición Bq)q. (Nota: Acabamos derepetir la demostración del Teorema R, capítulo 17. Según el problema 6,el razonador es fi.¡crtemente reflexivo, y por lo tanto, según a) del Teore-ma R, es reflexivo.)

8. Supongamos quc cree pl}(p:q). Entonces según el Irema 3,capítulo 15, creerá p>llq. También cree la tautología F(p>q), y al serregular, entonces cree I)<¡:B(p=q). Además, como cree p=B(p>q), debecreer B(p>q)>p. Dado que cree llq>B(p>q) y B(p>q)>p, creerá Bq>p. yporlo tanto cree Bq>p y p>Bq (como hcmos dcmostrado), y así debe creerp=Bq.

9. Supongamos que crce p=(Bp=q). Entonces creerá p:(Bprq), yademás, según el Lema 1, capítulo 15, creerá BprBq. Como cree la tau-tología qr(Bp>q), creerá q>p (dado que también cree p=(Bp>q)). pero esregular, y entonces creerá Bq>Bp. Al creer eso, y Bp>Bq, creerá Bp=Bq.Entonces usando lógica proposicional, creerá (Bp=q)=(Bq>q). Luego,dado que cree p=(Bp>q), creerá F(Bq=q¡.

10. El razonador no tendrá la menor idea de si el naüvo es casado o

1.44

no, pero creerá que el nativo es un caballero. Podemos comprobarlo delsiguiente modo.

Supongamos que m es la proposición que el naüvo es casado. El nativoha afirmado BQ<vm), donde k es la proposiciÓn que el nativo es un ca-ballero. En consecuencie el razonador creerá bB0<vm). Entonces creeráindudablemente k>B(kvm). También cree la tautología lo(kvm), enconsecuencia creerá Bk>B(kvm) dado que es regular. También creeB(kvm)=k, dado que cree k=B(k>m). En consecuencia creerá Bk>k. En-tonccs, por ser de üpo G, creerá k.

11. Supongamos que cree que es estable. Entonces paratoda propo-sición p, cree BBp>Bp; en consecuencia cree BBlsBI. Si es modesto, cree-rá entonces BI (porque para toda proposiciÓn q, un razonador modestoque cree Bq>q creerá q, y por lo tanto esto es verdad en especial si q esla proposición BJ-). Dado que cree BI, entonces si es estable, creerá I ypor lo tanto será inconsistente. Esto demuestra que si cree BBI:BI, nopuede ser modesto, estable y consistente. Por lo tanto, si es modesto,estable y consistente, nunca puede creer BBbBI-, y entonces no puedesaber que es estable con respecto a A.

745

Page 75: SMULLYAN, R. Juegos por siempre misteriosos. España. Gedisa 2008

Parte VIII

IMPOSIBLE DE DECIDIR

Page 76: SMULLYAN, R. Juegos por siempre misteriosos. España. Gedisa 2008

20

Por siempre indecidible

Hemos analizado el Segundo Teorema de la Incompleütud de Gódel(ningún sistema gódeliano consistente de tipo 4 puede demostrar su propiaconsistencia), pero todavla no hemos analizado su Primer Teorema de laIncompleütud. Ahora vamos a explayamos sobre este tema, primero en elcontexto de un razonador en una isla de caballeros y bribones.

Recordemos del capítulo anterior que un razonador óe considera es-table si para toda proposición p, si cree Bp, entonces cree p.

UN PROBLEMA DE INCOMPLETITUD

Diremos que el sistema de creencias de un razonador es lncotnpletosi por lo menos hay una proposición p tal que el razonador nunca creeráp y nunca creerá -p (le rcsuharáporslempre lndectdlble si p es'verdaderao falsa).

El siguiente problema se basa en el Primer Teorema de la Incomple-ütud de Gódel.

1

Un razonador normal de üpo 1 llega a la Isla de los Caballeros y losBribones y cree las reglas de la isla (que las reglas sean realmente válidas

. o no, es irrelevante). Se encuenEa con un naüvo que dice: 'Usted nuncacreerá que soy un caballero'.

Demostrar que si el razonador es tento consistente como estable,entonces su sisfema de creencias es incompleto. De modo más específico,encontrar una proposiciÓn p tal que las dos condiciones siguientes seanválidas:

a) Si el razonador es consistente, entonces nunca creerá p.b) Si el razonador es tanto consistente como estable, entonces nunca

creerá -p.

2, Un dual de 1

Supongamos que el nativo hubiera dicho en cambio: "Usted creerá que

149

Page 77: SMULLYAN, R. Juegos por siempre misteriosos. España. Gedisa 2008

soy un bribón". Encontrar una proposiciÓn p tal que las conclusiones a) yb) del problema 1 sean válidas.

El mismo razonamiento que se uülizÓ en la solución del problema r,aplicado a sistemas matemáücos en vez de a tazonadores, demuestra lasiguiente forma del Primer Teorema de la Incompletitr¡d de Gódel.

Teorerna l. Todo sistema gÓdeliano estable, normal, consistente detipo 1 debe ser incompleto. De modo más espec-rfico, si S es un sistemanórmal de tipo 1 y si p es una proposiciÓn tal que p=-Bp es demostrableen S, entonces si S es consistente, p no es demostrable en S, y si S tambiénes estable, entonces -p tampoco es demostrable en S'

Se dice que una proposición p es indecídible en un sistema S si ni ellani su negación -p es demostrable en S. Por lo tanto el Primer Teorema dela Incompleütud de Gódel nos dice que dado cualquier sistema gÓdelianoS estable, normal y consistente, siempre debe haber por lo menos una pro-posición p que, aunque es expresable en el lenguaje de S, no es decid'íbleen S: no puede ni demostrarse ni refutarse en S.

3.Variante de 1"

Supongamos que el nativo dice en cambio: 'Usted creerá que nuncacreerá que soy un caballero". Ahora encontrar una proposición p quecumpla con las condiciones a) y b) del problema 1'

(D - CQNSISTENCIA

Un número natural es por definición ó 0 Ó un número entero positivo1,2,3...De ahora en adelante usaremos la palabra "número" para significar"número natr¡ral". Consideremos una propiedad P de los números (natu-rales). Para todo número n, escribimos P(n) para significar que n tiene lapropiedad P. Por ejemplo, si P es la propiedad de ser un número par,éntó.rces P(n) significa que n es un número par, en cuyo caso P(0)' P(2)'P(4>, ... son todas proposiciones verdaderas; P(1), P(3), P(5), ... son todasproposiciones falsas. Por otro lado, si P es la propiedad de ser un númeroimpar, entonces P(0), P(2), P(4), ... son todas proposiciones falsas, mientrasque P(1), P(3), P(t, ... son todas ve¡daderas.

En lógica el símbolo estándar para "existe por lo menos un x tal que"es el símbolo *3", que se denomina técnicamente cuantifcador uísten-cíal. Para toda propiedad P de números, la proposiciÓn que existe por lomenos un número n que tiene la propiedad P se escribe: fnP(n). Ahorabien, supongamos qué tenemos un sistema matemático y una propiedadP tal que la proposición lnP(n) es demosrable en el sistema; sin embargopara éada n erlparticular, la proposiciÓn -P(n) es demostrable; es decir,iodas las infinitas proposiciones -P(0),-P(1),-P(2),...,-P(n)... son demos-

150

trables. Esto significa que por un lado, el sistema puede demostrar elenunciado general de que cierto número tiene la propiedad P,, y sinembargo se puede demostrar que cada número concreto na tJene la pro-piedad. Evid¿ntemente hay algún error en el sistema, porque li 3nP(n) esverdadera, es imposible que todas las proposiciones -P(0), -P(1),.",-P(n),... también sean verdadera5. Sin embargo, tal sistema no es necesa-riamente inconsistente: no se puede inferir necesariamente una contradic-ción formal a partir de todas esas proposiciones. Sin embargo, hayun nom-bre para esoisistemas. se denominai @-íncon$ístenres. (El símbolo uo¡o aveces se usa para representar el coniunto de los números naturales.)

Consideremos la siguiente situación análoga. Supongamos que

alguien nos da un cheque que dice: "Pagadero en cierto banco". Supo-nñndo que sólo hay una cantidad finita de bancos en el mundo, podemos

verificar si el cheque es bueno o malo en un período finitb de üempo;simplemente tenemos que tratar de cobrado en cada banco. Si por lo.nettos un banco lo acepta, entonces sabemos que el cheque es bueno; sitodos los bancos lo rechazan, entonces tenemos la prueba positiva de que

el cheque es malo. Pero ahora supongamos que vivimos en un universoen el qúe hay una cantidad infinita de bancos, y cada banco está numera-do con un número natural. Hay un Banco 0, un Banco 1, un Banco 2, ...,y así sucesivamente. También supongamos que somos inmortales, demodo que tenemos una cantidad inJinita de días por delante paratfatar decobrar él cheque" Ahora supongamos que en realidad ningún banco llegaráa cambiar el cheque. Entonces el cheque era malo en realidad; sin embargonopodernos dennstraio en ningún tienpofrnito. Podríamos tratar de co-brailo en los primeros cien mil millones de bancos y todos lo rechazan. Nopodemos ptése.ttar ese hecho como evidencia de que el que nos dio eliheque es deshonesto; él siempre puede decir: "Un momento, no me llamedeshonesto, ¡todavía no ha probado en todos los bancos!" Y por lo tanto,nunca podemos obtener una verdadera inconsistencia; lo único que te-nemos és una o-inconsistencia (e inclusive nunca sabremos esto en un pe-ríodo finito de tiempo).

lJna vez el concepto de o:-inconsistencia fue caractetizado humorís-úcamente por el matemático Paul Halmos, que definiÓ a una madre <o-incornisteÁte como alquien que le dice a su hijo: "Hay algo que puedes ha-cer, pero no puedes hacer esto y no puedes hacer eso y no puedes hacerlo otro..." nl hijo dice: "Pero, ma, no hay algo que pueda hacer?" La madreresponde: 'Sí, pero no es esto, ni aquello ni..."

- Un sistema se denomina <¡-consistente si no es <o-inconsistente. Por lo

tanto para un sistema co-consistcntc, si lnP(n) es demostrable, entoncesexiste por lo menos un número n tal quc la proposiciÓn -P(n) ¿o es demos-trable. un sistema inconsistentc dc tipo 1 también es <o-inconsistente,porque todas las proposicioncs son dcmostrables en un sistema incon-

t¡tente de tipo 1. Dicho de olra ftrrrtta, cn los sistemas de -üRo

1' !a o-consistencia irnplica automática¡nc:rtlt: t'onsistencia (ordinaria). Cuando seanalizala <>consistencia, a veccsi sc t ts¿ <:l tÓ rmino Cjltsistencií Simple pata

151

Page 78: SMULLYAN, R. Juegos por siempre misteriosos. España. Gedisa 2008

significar consistencia (para evitar cualquier posibilidad de confusiÓn)' Y

pó. lo anto, en esos términos, todo sistema tuconsistente de tipo 1 también

es simplemente consistente.AÉora volvamos al esn¡dio de los razonadores. En todos los problemas

que hemos considerado hasta ahora, el ordm en que el razonadortn

üeído varias proposiciones no ha desempeñado ningún papel' En los

p.áUfL-", ..si".ito de esre capít'lo, el orden desempeñará un papel

clave.El razonador l\ega a la Isla de los caballeros y los Bribones cierto día

que üamaremos el dia 0e. El siguiente se denomina día 1q; el día siguiente

s'e denomina dia 2e; y así sucesivamente. Para todo número nahrral n,

tenemos el día nq, y suponemos que el razonador es inmortal y,que tiene

infinita cantidad ¿é ¿iás por delante. Para todo número natural n y toda

proposición p, srrpongamos que B¿ e.9 la ^proposiciÓn

que el razonador

.r.é p "tt

algrin moménto durante'él día na. I¿ proposiciÓn Bp es, como

ri"*it , la iroposición que el razo¡adot cree p algún día, o, lo que es lo

;ido, lnB"p (Lxiste poilo -".tos

un ntal que el razonador cree p el día

"">. Oá.i-éi que el iazonador es <¡-inconsistente si existe por lo menos

ula proposicion p tal que el razo,nador (en algún momento) crea Bp'

"""q" para toaó n párticular, él (en algún momento) cree -Bp' El

razoiadór se denominará o¡-consistente si no es o-inconsistente.Ahora consideremos un razonador que cumple con las tres condicio-

nes siguientes.

Condtción Q. Es de tiPo 1.

CondiciónC,.Paratodo número natural ny p^r^ toda proposiciÓn p:

a) si el nzoiadot cree p el día na (tarde o temprano) cree-rá Bp;

bi si no cree p el día né (tarde o temprano) creerá -B"p' (La idea esque el razonador iiene presente las proposiciones que ha creído y que no

Éa creído en toda su vida.)

Condición C,. Para todo n y toda p, el razonador cree la proposición

B p>Bp (que, por supuesto, es una proposiciÓn verdadera)'" nt'sijuient! probiema se acerca mucho a la versión original de Gódel

de su Primer Teorema de la Incompletitud.

4. (Según Gódel)

Elrazonador,quecumpleconlastrescondic ionesci tadas, | |egaa] laIsla de los Caballeroi y los Bribones y cree Ias reglas de la isla' Se encu€nt¡a

con un nativo que lé dice, "Usted nunca creerá que soy un caballero".

Demostrar:a) Si el razonadores (simplemente) consistente, entonces nuncacteeÍa

que el nativo es un caballero.

1<)

b) Si el razonador es o¡-consistente, entonces nunca creerá que elnativo es un bribón.

Por lo tanto si el razonador es co-consistente (y en consecuencia tam-bién simplemente consistente), entonces, le resultará por siempre indecidible si el naüvo es un caballero o un bribón.

SOLUCIONES

1. La proposición p en cuestión es simplemente la proposición k, queel nativo es un caballero.

El nativo ha afirmado -Bk; en consecuencia el razonador creerák=-Bk.

a) Supongamos que el razo¡ado¡ cree k. Entonces, al ser normaL,creerá Bk. También creerá -Bk (dado que cree k y cree le-Bk y es de tipo1), en consecuencia será inconsistente. Por lo tanto, si es consistente, nun-ca creerá k.

b) Dado que el razo¡ador es de tipo 1. y cree k-Bk, también cree-k=Bk. Ahora bien, supongamos que llega a creer -k. Entonces creerá Bk.Si es estable, entonces creerá k y en corisecuencia se volverá incorsisten-te (dado que cree -k). Por lo tanto, si es estable y consistente nunca cree-rá -k.

En resumen, si es estable y consistente a la vez, nunca creerá que elnativo es un caballero y nunca creerá que el nativo es un bribón.

2. Observamos de la solución anterior que para cualquler proposiciónp (no tiene que ser la proposición particular k), si un razonador normal detipo 1 cree F-Bp, entonces si es consistente nunca creerá p, y si tambiénes estable, nunca cree¡á -p. Ahora bien, en este problema, el nzonadorcree k=B-k. Por lo tanto (al ser de tipo 1) cree -h-B-k, y por lo tanto creeF-Bp, donde p es la proposición -k. Por lo tanto, si es consistente, nuncacreerá que el nativo es un bribón, y si también es estable, entonces nuncacreerá que el natjvo es un caballero.

3. Una proposición p que sirve en es¡e caso es -Bk, como demos-traremos.

El razonador cree k=B-Bk.a) Supongamos que cree -Bk. Entonces, al ser normal, creerá B-Bk,

y entonces creerá k (dado que cree k=B-Bk y es de tipo 1). Al creer k yser normal, creerá Bk. Por lo tanto creerá tanto Bk como -Bk; en conse-cuencia se volverá incoruistente. Por lo tanto, si sigue siendo consistente,nunca creerá Bk.

b) Supongamos que cree --Bk. Entonces creerá Bk. Si es estable,entonces creerá k. Después, creerá B-Bk (dado que cree leB-Bk y es detipo 1). Entonces (de acuerdo al supuesto de que es estable), creerá -Bk.De este modo se volverá inconsistente (dado que cree --Bk). Esto

r53

Page 79: SMULLYAN, R. Juegos por siempre misteriosos. España. Gedisa 2008

demuestra que si el razonador es corEistente y estable a la vez, nunca

creerá --Bk.

4.Despuésdehaberresuel toelproblemal, laformamásfáci ldesolucionar'este problema es demostrar que todo razonadot que cumple

con las condiciones 1,2y 3 debe ser normal, y si es or-consistente, también

debe ser estable.a) Demostremos que es normal. Supongamos que crge P' E¡tonces

p^." .i"rto n, cree p ei di" no. Entonces por a) de la CondiciÓn 2, creerl'B_p. También cree B"p>Bp (por la condióión 3), en consecuencia al ser de

tifro 1 (Condición lj,'entónCes creerá Bp. Por lo tanto es normal'-

b) Ahora, supongamos que es co-consistente' Demostraremos que es

estable. Supongamos qrr" ct e Bp. Si nunca cree p' elior-Icgs p-an.cada

"ri-.ro ,r, .ro ."r.. p .f di" no, y én consecuencia por b) de la Condición

2, para to¿o tt creárá -8"p. Pero dado que cree Bp, entonces será cr>

iácbnsistente. por lo tanto, si es or-consistente y cree Bp, debe creer p al-

gún día. Esto demuestra que si es or-consistente, debe ser estable (91no-

iiendo que cumple con las Condiciones 1,2y 3, o aun simplemente b) de

la Condición 2).Por lo tanto, según el problema 1, le resultará imposible decidir'

27

iAun má,s indecisiones!

RAZONADORES DE TIPO ROSSER

Gódel demostró que toda una familia de sistemas matemáticos erar¡c9r.nnl9ta en el supuesto de que eran a-corslstentes.postetiormente J.Barkley Rosser descubrió un método ingenioso para demostrar que esossistemas eran incompletos según el supuesto más débil de que eransimplemente consistentes. El enunciado indecidible qn" .o.trtruy'ó Rosseres más complicado que el de Gódel, pero su indecidibilidad puedeestablecerse bajo el mero supuesto de simple consistencia.

volvamos a los razonadores en una isla de caballeros y bribones endonde el ordsn en el cual el razonador cree varias p.opoiiciorres tieneimportancia. Para proposiciones cualesquiera p y q, decimos que el razo-nador cree p antes de creer q si hay un dia en ei q.re ..ee p y todavía noha creído q Si 9l razonador nllnca cree q, pero ó.e p (uno u otro día),entonces consideraremos uerdad.eto que creé p antes d-e creer q. (En otraspalabras no tiene que llegar a creer q para creer p antes de creei q.) Su-pongamos que BpcBq es la proposición que el razonador cree p antes decreer q. Si Bp-<Bq es verdadera, entonces Bq<Bp es obviamenie falsa.

Ahora definimos un razonador de típo Rósser como un razonador detipo 1 tal que vale la siguiente condición.

condtción R. Para proposiciones cualesquiera p y q, si el razonadorcree p algún día en el que todavía no ha creíáo q, entorrces (tarde o tem-prano) creerá BpcBq y -(BqcBp).

La idea implícita en la condición R es que el razonad.or dene unamemoria perfecta paralo que ha creído y no ña creÍdo en toda su vida. sicree.p antes de creer q, entonces el primer día que cree p, todavía no hacreído q- (y quizá nunca lo haga, o tal vez lo creá en el futuro), y así cual_quiera de los-días siguientes recordará. que el primer día que creyó p,todavia no había creído q, y por lo tanto creerá-Bp<Bq y -(bq<Spj.

1

razonador de tipo Rosser rega a la Isla de los caballeros y losBribones y cree las reglas de la isla. se éncuentra con un naüvo que le'dice:"Nunca creerá que soy un cabalrero antes de creer que soy un bribón,,. qrra-

1,55

Page 80: SMULLYAN, R. Juegos por siempre misteriosos. España. Gedisa 2008

tducido simbólicamente, el naüvo afirma la proposición -(Bk<B-k).)

Demostrar que si el razonador es slmplenente consistente, entoncesdebe resultarle para siempre imposible decidir si el nativo es un caballeroo un bribón.

2

Supongamos que el nativo dijo en cambio: "Creerá que soy un bribónantes de creer que soy un caballero". ¿Se sigue la misma conclusiÓn?

Análisis. Las proposiciones demostrables de los sistemas matemáticosson demostrables en diferentes momentos. Podríamos considerar unsistema matemático como una computadora programada para demostrarvarias proposiciones consecutiuarnente. Decimos que p es demost¡a-ble anta que q (en un sistema matemático dado) si p se demuestra encierto momento en el cual q todavía no ha sido demostrada (q podría o nopodría demostrarse en algún momento posterior). Para proposicionescualesquiera p y q expresables en el sistema, la proposición BpcBq (p esdemostrable antes que q) también es expresable en los sistemas del tipoque consideró Gódel, y Rosser demostró que si p es demostrable antes queq, entonces la proposicón Bp<Bq y la proposición -(BqcBp) son ambasdemostrables en el sistema. Rosser también descubrió una proposición ptal que S-Gp<B-p) es demostrable en el sistema. (Tal proposición pcorresponde al nativo del problema 1 que dice: 'Usted nunca creerá quesoy un caballero antes de creer que soy un bribón".) Entonces, por elargumento de la solución del problema 1, si p es demostrable, entoncesel sistema es inconsistente, y si -p es demostrable, entonces el sistema esde nuevo inconsistente. Y por lo tanto, si el sistema es consistente, la pro-posición p es indecidible en el sistema.

El enunciado de Gódel puede parafrasearse como: "No soy demos-trable en ningún momento". La frase más elaborada de Rosser puede para-frasearse: 'No puedo ser demostrable en ningún momento, a menos quemi negación se haya demostrado previamente." La frase de Gódel, aunquees más simple, requiere el supuesto de la <o-consistencia para que elargumento funcione. La frase de Rosser, aunque es más complicada,funciona con el supuesto más débil de consistencia simple.

UN PROBLEMA DE INCOMPLETITUD MAS SIMPLE

Hasta ahora hemos a¡alizado dos demostraciones de incompletitud:la de Gódel y la de Rosser. Existe oÚa más simple que esas dos, quecombina el método de Gódel con el uso del concepto de uerdad, concep-to que más adelante introdujo el lógico Alfred Tarski. Siempre me ha resul-tado un enigma por qué esta simple demost¡ación -tan conocida por los

156

expertos- sea tan dejada de lado en los libros de textó elementales.En el problema siguientc, cl orden en el que el razonador cree va¡ias

proposiciones no tiene absolu [a nrc ntc nin guna importancia.

.J

Supongamos que tenemos un ¡azonacl rr. llamémoslo paul_ oue esslempre coffectoen sus creencias (nunca crc(: .r:r ¡rr.posición fabá). Notiene por qué ser de üpo 1, o normal, ni es ncr:cs;rr i , r¡rrr: visirc rcalmcnlcla Is la de los cabal lcros y los Br ibones. Lo único ( l r ( . r ( . r ( . r r .s,¡rr t .s.r l r , . rsobre él es que es sic:mpre correcto.

Un día un nativ() c()menta sobre él: *Paul nunca ct.(.r , ' r (¡r(. : .{ ,} , uncaballero". Rntonces sr.: sigue lógicamente que el sistema d.., , , . , . , , , , , , . . ,1,.Paul debe scr incomplcto. ¿l)or qué?

4

Supongamos que el nativo tlitr.<:n t'rrntrío: ,,Un día paul creerá que soyun bribón". ¿Aún se seguiría quc cl srstr 'rrr.r r lr : crcencias de paul es inco--pleto?

UNA SITUACION MAS (]OMI' I . I ( ,AI)A

Consideremos un razonador estable consist(.rtt(. r lt. tr¡,rr ( i I l ;ry urr;rcuestiÓn muy importante que le resuhará por sicnr¡rr r. r r rr lr ̂r r r lr l ' lr. l .r, r ¡¡ ':,üón de su propia consistencia. Nunca podrá decidir si <,s r or¡:,t:,tcrrtr. r, n,,¿Por qué?

Unapfegunta. Por supuesto, el resultado que acabanr<¡s rtr. r it,u r.:válido reemplazando"razr¡nador" por "sistema": Ünsistema cstalrlt. r.r,r¡:,r:,tente de tipo G nunca puede demostrar su propia consistencia, ni srr ¡rr oJrr,rinconsistencia.

Sin embargo, surge una inrp()rlante pregunta: ¿Cómo sabemos si ,a7sistemas estables consistentcs dc ti¡to G? ¿No es posible que el concept()mismo de sistema estable consi.stcnlc de tipo G oculte alguna sutil con-tradicción?

Esta cuestión será resuelta t()tírlr¡rcnte antes de terminar este libro.

sol . t r ( r f ( )Nl is

1. Dado que el nativo aflrrl(r '(llk<l]-k), el razonador creerák=-(Bk<B-k). Supongamos que <:l r¡rzru¡:rrlor cs (simplemente) consis-tente. Tenemos que demostrar qu(: nun( l < r<'t:rá k y nunca creerá -k.

a) Supongamos que llega a crr:t.r l< l):rtlo r¡rrt: cs consistente, nunca

157

Page 81: SMULLYAN, R. Juegos por siempre misteriosos. España. Gedisa 2008

creerá -k;en consecuen cia creerek antes de creer -k' Por lo tanto creerá

SkiB-t (po. ta CondiciÓn R). Pero también cree le-(Bk<B-k); entonces

.i".*:q'v al creer k, ierá inconsistente' Por lo tanto si es consistente'

nunca puede creer k.b)

-supongamos que llega a creer -k' Al ser consistente' nunca creera

k. fuégo Lree"ri -kaÁtes dJcreer k; en consecuencia según la Condición

n o".ia -(Bk<B-k). Pero cree k=-(Bk<B-k), por lo tanto creerá entonces

f. v se.a inionsistente. Y así, si es consistentg tampoco puede creer -k'

2. La respuesta es sí. Dejamos la demostracíÓn a cargo del lector'

3. Si Paul llega a creer que el nativo es un caballero' eso false.ará lo que

dijo -el-nativo, cónvirtiendó de este modo al nativo en un bribÓn' y en

cónsecuencia convertirá a Paul en incorrecto al creer que el nativo es un

i^¡"ft".o. Pero se nos dice que Paul es siempre correcto; en consecuencia

nunca llegará a creer queel nativo es un cabáilero' En consecuencia lo que

;ñ; ;;i;" es verdád; por lo tanto el nativo es realmente un caballero'gáio"a"t, como Paul es siempre correcto' nunca tendrá la creencia errÓnea

de que eí nativo es un bribén. Y así Paul nunca sabrá si el nativo es un

caballero o un bribÓn.

Aná.tisis. El contenido matemático del precedente acertijo es el

siguiente: en los sistemas investigados por GÓdel, tenemos no sÓlo ciertas

;;';ó;J;it;.s llamadas propottion" i demostrables' sino también .unail"é Áa,

"-plia de propási.io.tes de nominada proposicione s u erdaderas

á"itit,.-". i" ct"te ¿é-tai proposiciones verdaderas del sistema cumple

f¿il;il;;;i", i.gf", de tas mbtas de verdad para los conectores lógicos;

aáe-¿s, para toda lroposición p del sistema, la. proposiciÓill::,.:""

pr"p"tition uer¿aaeri del sistema si v sólo.si,p :t-:i",lj:o:::iu"demostrable del sistema. Ahora bien, GÓdel descubriÓ una notable

óroposiciO" g tal que la proposición g=-Bg cra una proposiciÓn uerdadera

á.i'riii.*, íen réalidaá, iñclusive demosrrable en el sistema, pero-este

hecho más fuerte no se .tecesita para el presente"argumenJg)' Si g fuera

falsa, Bg sería verdade."; "r,

co.,s.cuenciá gsería demostrable' en conse-

cuencia"verdadera, y tendríamos una contradicciÓn' Por lo tanto g es

verdadera; arr aonr"arr"rrcia -Bg es verdadera,en consecuencia g no es de-

mostrable en el sistemá. po, toianto g es verdadera pero no demostrable

en el sistema. Dado que g es verdadera, -g es falsa; en consecuenqa lam-

Doco es demost¡able tt' El 'i't"-"

(dado (ue todas las proposiciones de-

;;;ú1";;;; tto¿"¿.r"t). Y por lo t"ttto g es indecidible en el sistema'

4. La respuesta es sí. Deiamos la demostraciÓn a cargo del lector'

5. En el capítulo 18 demostramos que todo tazonador de tipo G es

modesto, y demostramos que ningún rázonador modesto consistente de

iip" Z iá'úausive de tipo i) puedi creer que es consistente' Por lo tanto

158

ningún razonador consistente de tipo G puede saber que es consistente.Por ot¡o lado, todo r¿zonador estable que cree que es inconsiStente es

realmente inconsistente, porque si cree que es inconsistente, entonces creeBJ-, y si además es estable, cree a, y en consecuencia es inconsistente.

Por lo tanto, ningún razonador consistente estable de tipo G puedeLlegar a creer que es consistente o llegar a creer que es inconsistente. Estácondenado a la incertidumbre eterna sobre esta cuestión.

r59

Page 82: SMULLYAN, R. Juegos por siempre misteriosos. España. Gedisa 2008

Parte IX

MUNDOS POSIBLES

Page 83: SMULLYAN, R. Juegos por siempre misteriosos. España. Gedisa 2008

7

22

¡No es necesariamente así!

Gran partc tk: lo que hemos estado haciendo en este libro se relacionaestrechamentc t on la disciplina conocida como lógica modal. Lo sorpren-dente sobre la ldrgir'a modal es que surgió de consideraciones puramentefilosóficas, penr L rs siistemas de axiomas que se derivan de ella han tenidorecientemente un:r intcrpretación completamente diferente, que ticneinterés matemátic'o y r lrrc figura prominentemente en lateoría de la demos-tración, la ciencia tlr' l,r t omputación y la inteligencia artificial. En capítulosposteriores, ten(lrr.nr,'', rnás para hablar sobre la interpretación ma-temáüca.

El concept<l l i rrrr l . l ru'rrtal de la lógica modal es el de proposiciónnecesarialnenle verrl,¡,l.r.r y no solamente verdade¡a por las ci¡cuns-tancias. Muchas u..,¡ . ' ¡ ,L,r rrrros: "Sí, es cierto que fue así, pero realmenteno tenía por quó s(t A'il l't,(lña haber sido de otro modo". Otras vecesdecimos: "Tmfa (¡r' 5.'r .r',r No podrÍa haber sido de otro modo". Y porlo tanto generalmcttlr lrJ' ¡'ilr,,:. rrna distinción entre algo que simplementeresultó ser verdade:ro, y ,rlg,r {lrr(: es necesariamenfe verdadero. Comoejemplo, es una rcal lr l r , I r ¡ l , Ir .ry ( 'xactamente nueve planetas en nuestrosistema solar, pero e¡¡ ltG:rl.:, l,lrrr('tlte concebible que las cosas pudieranhaber sido de ot¡o tttnrll 1, '¡u, ¡rotlrían haber exisüdo más o menos denueve planetas. Por rf rr lá '1", ur,¡ l )roposición tal como dos más dos escuatro no sólo es vcrrl¿rle¡¡, 4|ttt ,tt\ (sariamente verdadera. En ningunacircunstancia posiblc lr txl¡¡n i i . , t ' i . r tr) que dos más dos no fuera cuat¡o.

En vez de avann'¿r l t láq sl l i r I r ' rr l : l f i losofia de la verdad necesaria,vamos a considerar al¡¡tttlua !r! .'rlrl' "¡ lfr¡iic:os que ilustfan lo que se conocecomo la semántica dc Ktl l r l ' . - l i r¡r .r¡ .r l iz lrcmos en el próximo capítulo.Preparándonos para cg(t, eál l l , l ¡ : . :ur, r ' , ¡¡cstra notación.

Según el lÓgico morlal | | li"' r',, rrrilremos la lelra N para nece-sc¿ríamente verdadera, ( l tu l* ¿' i i ¡¿i¡ ' l r , l ' l sírnbolo más usado esfl .) Deeste modo, pAratOdA pfQlrr,álr ¡ t : ! i r l ! l i l l ' r l [ ( ' lamos Np Como "p es neCe-sAriarnmte verdadera". Y ¡rrr l i , t l i t t l ' r tr tr t , l l : l notaciÓnserá como la delos capítulos anteriores, exrFlr lr¡ i l r tF. t ! ' :u, rros la letra N en vez de la B.La definiciÓn de un coniuni l t r le ; ' i rr¡","¡, r{ ' r i ¡" ' ( lc t ipo 1, 2, 3, 4 Ó G es lamismaqueantes; laúnicadl fcre¡ le ieF=,1i1¡ t r r r ' \ i .u¡)cnteusamosNenlugarde B.

163

Page 84: SMULLYAN, R. Juegos por siempre misteriosos. España. Gedisa 2008

I

1. Un uniuerso de razonadores

Ahora considcremos un universo entero dc razonadores: dcnomina-remos este unive¡so U,. Dada toda proposiciÓn p, todo razonador cree p

o descree p, pero no ambas cosas. (Descreer una proposición significacreer que es falsa.) Todo razonador descree I y sus creencias siguen lasreglas de las tablas de verdad para los conectores lÓgicos. Por ejemplo, crccp:q si y sólo si o descree p o crec q (o ambas cosas). A partir de esto sesigue que todo razonador cree todas las tautologías. Además, si unrazooador cree p y cree p)q, debe creer q (porque si descree q, crccría py descreería q; en consecuencia descreeria p->qenvez de crcer p:q). Por

10 tanto todo razonador es de tipo 1. Es tambiÓn un dato que todo razo-

nador sabe lo que creen todos los otros razonadores.Ahora viene lo curioso. Por un motivo u otro, todo razonador tiene

absoluta confianza en el criterio de sus padres, y por lo tanto para todaproposición p, un razonador cree que p es necesaríümente verdadera siy sólo si sus padres ambos creen p. Esto se conoce como "regla funda-mental" del universo. Es tan importante que vamos a registrarla formal-mente.

Reglafundamental d.e (J,.IJn razonador c¡ee Np si y sÓlo si sus padres

ambos creen p.

Comentario.Hay un rumor de que un compositor de nuest¡o universo-un compositor norteamericano, en realidad- al visitar el universo U, y

enterarse del motivo por el cual los habitantes creían que una proposiciÓnera necesariamente verdadera, negó conla cabezz en un gesto de escep-ticismo, y dijo: "¡No es necesariamente así!" Pcro yo no puedo 8 ranLizarlo,lo escuché sólo como un rllmor.

Una proposición p se denomina establccida (para el universo U,) sitodos los habitantes la crcen.

Obviamente todas las tautologías son establccid"as, pero el conjunto delas proposiciones establecidas trasciende las tautologías. En realidad, elconjunto de proposiciones establccidas debc ser de tipo 3, es dccir:

1a) Todas las tautologías son establecidas.lb) Si p y prq son establecidas, también lo es q.2) (Np&N(p>q))>Nq es establccida.3) Si p es establecida, también lo es Np.

Demostrar que el conjunto de proposiciones establecidas es de tipo 3.

2. Segund.o uniuerso

Ahora visitamos otro universo, que denominaremos Ur. Las condi-

ciones que definen este universo son como las de U' con una importantediferencia. En este universo, un razonador cree que p es necesariamenteverdadera si y sólo si todos svs antqosatd.os cteen p. (Para simplificar lascosas, suponemos que todos los habitantes son inmortales, y por lo tantotodos los antepasados de una persona todavit viven.)

Registremos este hecho fundamental.

Hecho 2. En el universo U2, un razonador x cree Np si y sólo si todoslos antepasados de x creen p.

Ahora las cosas se ponen más interesantes.Demostrar que el coniunto de proposiciones establecidas del universo

U, debe ser de tipo 4.

3. Tercer uniuerso

Hasta ahora, no hemos dicho si el universo tuvo o no un comienzo cnel tiempo. Bueno, ahora consideramos un tercer universo U, qrre cumplccon todas las condiciones que hemos dado para U., más la condición deque sí tuvo un comienzo en el tiempo. Esto significa que para todo in-dividuo x, si consideramos un antepasado x' de x, luego otro antepasadox" de x', y seguimos retrocediendo de este modo, tarde o tempranollegaremos a un antepasado que no tiene antepasados, y en consecuenciano tiene padres. (Responder la pregunta sobre la forma en que nacieronesos individuos sin padres está fuera del alcance de este libro. El lectorinteresado debería consultar un libro sobre la evolución o un libro sob¡ecreacionismo, según sus intereses científicos o teológicos.)

Como ya debe ímaginarse el lector, nuestra meta es demostrar que lasproposiciones establecidas de este universo forman una clase de tipo G.Pero antes de seguir con esto, debemos aclarar un punto para el lector queno esté famtliartzado con la lógica de todosy algunos.

Supongamos que alguien dice sobre cierto club: "Todos los francesesen este club usan boinas", y resulta que no hay ningún lrancés en el club.Entonces el enunciado ¿debería considerarse falso, verdadero o inapli-cable? Los que no estén familiarizados con la lógica formal bien puedentener diferentes opiniones sobre el asunto, pero la convención adoptadaen lógica, matemática y las ciencias naturales es que todo enunciado de laforma: "Todas las A son B" debe considerarse falso sólo si existe por 1omenos una A que no es B. Y por lo tanto la única forma en que el enunciado"Todos los franceses del club usan boinas", puede ser falso es si hay porlo menos un francés en el club que no usa boina. Si sucediera que no hayf¡anceses en el club, entonces indudablemente no hay ningún francés enel club que no use boina, y por lo tanto se considera entonces uerdaderoque todos los franceses del club usan boinas. Esta es la convención queadoptaremos.

Aplicando esto a nuestro univcr.so lJ' si x es un individuo que no tieneantepasados, entonces todo lo quc sc ¡rr-rcde decir sobre todos sus ante-

r& ro)

Page 85: SMULLYAN, R. Juegos por siempre misteriosos. España. Gedisa 2008

Ypasados es verdadero (¡porque no üene ninguno!). En particular, dadatodaproposición p, consideramos aerdade¡o que todos los antepasados de xcreen pt y porlo tanto si x no tiene antepasados, entonces x cree Np. CIaúnica forma en que un individuo x puede no creer Np es si por lo menostiene un antepasado que no cree p, lo cual no es posible para un individuoque no tiene antepasados en absoluto.) Registremos esto como Hecho 1.

Hecho./. Si x no tiene antepasados, entonces para toda proposición p,x cree Np.

Nuestra meta es demostrar que el conjunto de proposicionesestablecidas de U, es de üpo G. Indudablemente es de tipo 4 (por elproblema 2, dado que todas las condiciones de U, también válen para U).Entonces nos resta demost¡ar que para toda p?oposición p, odos lóshabitantes de U, creen la proposición NQ.{p>p)=Np. La demostración esmuy bonita; la ídea clave está contenida eñ el siguiente lema.

Iema 1. Si x descree Np, entonces ¡ debe tener un antepasado y quealavez descree p y cree Np.

Primero, demostrar este lema. Luego demostrar que el conjunto deproposiciones establecidas de U3 es de tipo G.

,La forma en que todo esto se relaciona con la semántica de Kripke seráexplicada en el próximo capínrlo.

SOLUCIONES

. 1. 1) Sabemos que las condiciones 1a) y 1b) son verdaderas, dado quecada nzonador es de tioo 1.

2) Por lo tanto el siguiente paso es demosúar que todo razonador xcree (Np&N(p>g))>Nq, o, lo que es lo mismo, si cree Np&N(p>q), enton-ces también debe creer Nq. por lo tanto, supongamos que x creeNp&N(p>q). Enronces cree tanro Np como N(p>q): Dado que cree Np,entonces sus padres ambos creen p. Dado que cree N(p>q), entonces suspadres creen prq. Por lo tanto sus padres tanto creen pcomo p:q, y al serde tipo 1, ambos creen q. Dado que sus padres creen q, entonces x creeNq.

De este modo hemos-demostrado que todo razo¡ador x de U, cree(Np&N(p>q))>Nq, y por lo ranro esra proposición es establecida. "

3) Por último, debemos demostrar que si p es establecida, también loes Np. rJs¡e no significa que todo razonador que cree p también creerá Np,sino sólo que si todos los razonadores creen p, entonces todos creen Np.)Esto realmente es bastante obvio. supongamos que todos los razonadoiescreen p. Consideremos un razonador x. Entonces sus padres creen p(porque todos los razonadores lo hacen); en consecuencia x cree Np.

-

166

2. La importancia del cambio de "padres" a 'antepasados" es ésta: Si

/ es un progenitor de x y z es un progenitor de y, dificilmente podemosllegar a la conclusión de que z cs un progenitor de ¡. Pero si l/ es una,ntQasado de r, y z es un antcpasado de y, entonces z es un antepasa-do de r. (En terminología matemática, la relación de ser un antepasado estrarcitiua)

La demostración de que el conjunto de proposiciones establecidas deluniverso U. es de tipo 3 es la misma que para el último universo U,(simplemente reemplazando la palabra "padres".por'antepasados"). Peroahora podemos demostrar el hecho adicional de que todo razonador deeste universo cree NprNNp, y en consecuencia que el conjunto de pro-posiciones establecidas es de tipo 4.

Supongamos que .lc cree Np. Tenemos que demostrar que tambiéndebe creer NNp. Bueno, supongamos que rc'es un antepasado de x y

supongamos que ,c" es un antepasado de ¡c'. Entonces ¡" también es unantepasado de ¡. Dado que rc cree Np y x" es un antepasado de Jc, en-tonces x" debe creer p. De este modo todo antepasado x" de x' cree p;

en consecuencia .lc'cree Np. Esto demucstra que todo antepasado x' dex cree Np, y por lo tanto ¡ debe creer NNp.

3.Pzra demostrar el Ie ma 1, supongamos primeramente que rc descreeNp. Entonces por lo menos debe tener un antepasado rc' que descree p(porque si todos sus antepasados creyeran p, é1 creería Np, lo cual no pasa).Ahora bien, si x' cree Np, terminamos: tomemos como y a ¡'. Pero si x'no cree Np, entonces descree Np; en consecuencia x'debe tener por lomenos un antepasado r" que descree p. Si x" cree Np, terminamos(consideramos que.,y es ,r'). Pero si no es así, entonces consideramos algúnantepasado r"' de x" que descree p, y seguimos así hasta que finalmentedebemos llegar a algún antepasado y de ,ú que descree p y que o no tieneningún antepasado (en cuyo caso j!., cree Np por el Hecho 1), o que no úeneantepasados que crean p, y en consecuencia y debe creer Np. (La razónpor la que finalmente debcmos llegar a ese antepasado y es porque eluniverso U, tuvo un definido comienzo en ei tiempo, hecho que no se diopara el universo Ur)

Ahora podemos demostrar que todos los razonadores de este univer-so deben creer N(Np:p):Np (y en consecuencia que el coniunto de pro-posiciones establecidas es de tipo G). Para demostrarlo, basta demostrarque todo razonador que cree N(Npp) creerá Np; o, lo que es lo mismo,que todo razonador que descree Np también descreerá N(Np=p).

Supongamos que x descree Np. Entonces según el Lema, rc üene unantepasado y que descree p y cree Np. Entonces cree que Np es verdaderay p es falsa; por lo tanto debe descreeer Np>p. Por lo tanto n tiene unantepasadoy que dcscree Nprp; en consecuencia no todos los antepasa-dos de .r creen Np>p, por lo tanto x debe descreer N(Nprp).

r67

Page 86: SMULLYAN, R. Juegos por siempre misteriosos. España. Gedisa 2008

7Esto demuestra que si r descree Np, descree N(Np>p); en conse-

cuencia si ¡ cree N(Np>p), debe creer Np, y por lo tanto x debe creerN(Np>plNp.

23

Mundos posibles

El tema de la lógica modal es antiguo: se remonta por lo menos aAristóteles. Los conceptos principales son los de proposición necesaria-nmte verd^dera y proposición posíblernmte verdadera. Cualquiera delos dos conceptos puede definirse en términos del otro. Si empezamos conel concepto de verdad necesaria, definiremos una proposición comoposi-blemente uerdadera si no es necesariamente falsa. Alternativamente, po-dríamos emryzar con el concepto de una proposiciónposiblemente ver-dadera y luego definir una proposición como necesarianTmte verdadera,si no es posible que sea falsa.

La lógica modal provocó considerable interés ent¡e los filósofos yteólogos medievales y posteriormente fue fundamental en la filosofia deIeibniz. El pensamiento de lcibniz fue el que inspiró al filósofo conrempo-ráneo Saul Kripke para inventar el campo conocido hoy en día comosemántlca de los mundos poslbles, también llamada semóntica de kipke(que es lo que esh¡vimos haciendo en el último capíh.rlo, usando unaterminología diferente).

I¡ibniz pensaba que habitamos en un lugar llamado el mundo real,que sólo es uno entre una cantidad de mundosposíbles. Según la teologíade leibniz, primero Dios examinó todoslos mundos posibles y luego creóel que consideró mejor: este mundo. De allí su aforismo: ',Este es el meiorde todos los mundos posibles". (En Cándido Volraire se burlaba conri-nuamente de esa idea. Después de describir casi todas las catástrofes posi-bles, Vcltaire agregaba:'...en éste el mejor de todos los mundos posibies.',)

Ahora sigamos con Leibniz. Una proposición p debía considerarseverdader^ para o enun mundo dado ¡(real o posible) si describía correc-tamente ese mundo, y falsa para ese mundo si no lo hacía. Si p se de-nominaba uerdadera sin calificación, debía entenderse que significabaverdadera para este mundo (el real). Decía que p er^ necesariamenteverdadera si era verdadera p^r^ todas los mundos posibles, y poslblemen-te verdadera si era verdadera por lo menos para un mundo posible. Esta---€n resumen- era la filosofia de los .mundos posibles', de Leibniz. (Sise hubiera hecho realidad algún orro mundo posible, me pregunto siIcibniz hubiera tenido la misma filosofia.)

Antes de 1910, el tratamiento dc la lógica modal carecía de la preci-sión de las otras ramas de la lógica. Ni siquiera Aristóteles, que fue nota-

168 169

Page 87: SMULLYAN, R. Juegos por siempre misteriosos. España. Gedisa 2008

blemente preciso en su teoría del silogismo, dio una explicación igual-mente clara de la lógica modal. El filósofo norteamericano C. I. Lewis fuequien, en una serie de ensayos publicados entre 1910 y 1920, describió unaserie de sistemas axiomáücos de diferente poder e investigó qué propo'siciones eran demostrables en cada uno de ellos. En cada uno de esossistemas, todas las tautologías están entre los axiomas (o por lo menos sondemostrables a partir de ellos), y para toda proposición p y q, Iewis adoptócomo regla que si p y p>q son ambas demost¡ables en el sistema, tambiénlo es q. Por lo tanto todos los sistemas de Iewis son por lo menos de tipo1. Luego, Iewis razonó que si p y prq son ambas necsariatnmte ver-daderas, también lo es q. En consecuencia todas las proposiciones de laforma (IrIp&N(p>q))>Nq (o alternaüvamente, todas las proposiciones dela forma N(p>q)>(Np:Nq)) fueron tomadas como axiomas. Luego, parcciarazonable que algo que podría demostrane puramente sobre la base deaxiomas necesariamente verdaderos debe ser necesariamente verdadero,y hoy en día muchos sistemas modales 0os llamados normales) considerancomo regla de inferencia que si una proposición X ha sido demostrad4entonces se justifica que lleguemos a la conclusión NX. (Esto no signifi-ca que )(>NX sea necesariamente verdadera, pero si X ha sido demostrada-puramente sobre la base de axiomas que son ellos mismos nece-sariamente verdaderos- entonces se justifica que afirmemos NX.)

El sistema que hemos descrito hasta aquí es de tipo 3 y tiene un nombreestándar en la actualidad. Se denomina sistema tnodal K y consütuye labase de una amplia clase de sistemas modales.

Ahora bien, ¿qué pasa con NXrNN)C2 (Si X es necesariamente verda-dera, ¿es necesario que sea necesariamente verdadera?) Bueno, las propo-siciones de esta forma se consideran axiomas en algunos sistemas modalesy en otros no. El sistema modal crryos axiomas son los de K junto con lasproposiciones de la forma NX'rNNX es muy importante y hoy en día se lollama sístetna modal Kt Obviamente es de tipo 4.

El sistema modal G -que consiste en Kd con el agregado de todas lasproposiciones de la forma N(Np:p)>Np como axiomas- llegó sólomuchas décadas después (a mediados de la décadade 1970) y no surgióde ninguna consideración filosófica del concepto de necesid,adlígica, sinodel Segundo Teorema de Gódel y delTeorema de Lób. Nos explayaremosmás sobre este tema en el próximo capínrlo.

Modelos de Krípke. A fines de la década de 1950, Saul Kripke publicósu famoso ensayo, "A Completeness Theorem in Modal Logic", que originóuna nueva era paralalógica modal. Por primera vez se construyó una teoríaprecisa de los modelos para los sistemas modales, que no sólo era deinterés matemáüco sino que ha conducido a toda una rama de la filosofiaconocida hoy en día como semántica de los mundos posibles.

Primero Kripke planteó una cuestión básica sobre el sistema de Ie ibnizque evidentemente leibniz no consideró. Según Leibniz, habitamos en elmundo real. Los llamados mundos posibles ¿son todos los mundos que hay,

170

o sólo son los que son posibles relatiuos a este rnundo? En otras palabras,desde el punto de vista de otro mundo, la clase de los mundos posibles ¿esdiferente de la clase de mundos que son posibles relativos a este mundo?O, pan plantear la cuestión aun de ot¡a forma más, supongamos quehacemos una descripción de algún mundo r, y consideramos la propo-sición "¡ es un mundo posible". ¿Es la verdad o falsedad de esa propo-sición algo absoluto, o podría ser que esa misma proposición fueraverdadera en algún mundo y perc falsa en algún otro mundo z? En estecaso la cuestión dc la transitividad es de particular importancia: Si el mun-do yes posible rclativ<¡ al mundo r y si el mundo z es posible relativoal mundoy ¿se sigrrt: <¡rrc cl mundo ztiene que ser posible relativo almundo x? La rcs¡rrrr:st:r :r (rsta pregunta determina qué sistema de la lógi-ca modal es apro¡r iat lr .

Según Kripkc, rL't irrrrs r¡uc el mundo y es accesible al mundo x siyes posible relativo u.r:. Y ahora consideremos un "superuniverso" demundos posibles. I )rrt L ¡s r¡ ¡ur ulos cualesquiera )c e y, o y es accesible a ro no lo es. Una vcz ( lu( ' sr ' rk ' tcrmina qué mundos son accesibles a quéotros, tenemos lo r¡rrr' :,r' rk:nomina técnicamente un rnarco. Dadacualquier proposición ¡r y r r r.rlr ¡uicr mundo ,c, p es verdadera en ¡ o falsaen .r. Yuna vez quc s(' rL'lrr r r u r.r <¡uó proposiciones son verdaderas en quémundos del marc<1, lcilr.ilr¡,:, l( ¡ (lUc se denomina modelo de Kripke.Debeentenderse que -L (:s l .r l : , ,¡ r ' r¡ r ,rr la uno de los mundos y que p)q esverdadera en el mu¡rt |rr, \ ' : ,r 1, : , ír lo si no es cierto que p sea verdadera en.r y q sea falsa en ,c. l \ ' r ' , , tc rrr ' r lo, g)ara cada mundo.r, el con.iunto detodaslasproposic iorrr .svr ' r , l . r , l , r . rsr ,nr€sdet ipo1.Paracompletar lades-cripción, una proposi l i r ' rrr F.J¡, . .r ' ,1,.r l ;rra verdadera melmundox si y sólosi p es verdadera crr /orlrrr htt ttut¡ttlt$ accaibles a¡. Decimos que p estáestablecida en un n¡orlr.l, t, t t e ',, t',ill(lil en un modelo, si p es verdaderaen tOdOS los mundr¡s t l r , l r r ¡ , , , | r .1, ,

La estructura (¡t(. l r .ru rr ' , ' . .¡ l r , ,r , t tcalmente es la misma que la delúl t imocapítulo,ex(:(r l l l { r ¡ r , r l r t , ' r r r r r rn l , rgía.Enlugardeloselementosdeluniverso denominatkrs t t t : t ' t t , t , l t , , , ' \ , .r l r()ra se denominan mundos. Enlugar de la relación " .y t , r i t r r r l , r , ,H,,r ¡ r , r t l { : x" , o 'y es un antepasado dex", ahoradecimos "T <:r ' ,rr r r=:¡ l ,1, .r r " l 'ol úlümo, en lugar de "p es creídapor el raz.onador x", Alror¡ l¡ ' rr , ¡rr, , : " lr , , , vcrdadera en el mundo ¡". Conestas transformacioncs, tr¡¡ lu,¡ l , , ' , rr . ,ulr.rr los del úlümo capítulo cambian.El conjunto de todas l : ts ¡rrol r , :¡ ' ¡¡¡n . , (¡u( ' son vál idas en el modelo deKripke deben ser dc t i¡ ,o I (¡rr, , l ,1, n.r l , r ,r fr í tulo 22),y en consecuenciael sistema modal K es a¡r l l , ¡ l , l ¡ ' r 1,, , 1,, ' . l ¡r : r¡gdelos de Kripke.

Supongamos quc ¡ l tol , t l l , l t r l t ,r , " , l , r , orrr l ic ión de t¡ansit ividad: paratodo mundo X, y,y z, si ¡r t'r ir , i,-¡l,l' t \ y z cs accesible a /, entgncesZ eS aCCeSible a.f. Ent<¡lt< ¡.r lr'nF'r I r r'J lil r lr r,' '.,' tlclnOmina mOdelO trAnSíliUOde Kripke. Bueno, para l{xLr ¡¡rr , ' l ¡ 1,, tr .rr ' .r{rv() dc Kripke, el con.iunto detodas las proposicioncs (¡rr,ql¡r r . .r ' l . r , l , r ,r ' , l ) :rra todos los mundos delmodelodebenserdet i ¡ ro. l { ¡ r l i l r l ' r ¡ ¡ ,1 . ' , r . r ¡ r r t r r lo22),ypor lotantoels istema modal apropiar lo r ' : r=nt ' ' ¡ r ' , , , . r ¡ ,1r , , r l , l r '

171

Page 88: SMULLYAN, R. Juegos por siempre misteriosos. España. Gedisa 2008

f

Así el sistema modal K es aplicable para todos los modelos de Kripke,y el sistema modal Ko es aplicable a todos los modelos ffansitíuos deKripke. Estos dos resultados se conocen como teoremas de la solidezsemántíca paraK y Kr. Kripke también demostró sus inversas: 1) Si p esválida en todos los modelos de Kripke, entonces p es realmente demos-trable en el sistema modal K. 2) Si p es válida en todos los modelos tran-sitiuos de Kripke, entonces p es demostrable en \ . (Más adelante expli-caremos exactamente lo que significa demostrabilidad. en un sistemamodal.) Estos dos resultados se conocen como teoremas de la comDletituélpara K y K,.

Decimos que un modelo de Kripke es tertnínal si la siguiente condi-ción es válida Dado cualquie¡ mundo x del modelo, si pasamos a un mun-do x' accesible a x y luego a un mundo x" accesible 4 x,, y seguimosavanzando, finalmente debemos llegar a un mundo y al otal ningúnmundo es accesible (lós llamados mundos tennínales, que se comportancomo los razonadores sín padres del último capítulo). Decimos que unmodelo es de tipo G si es transitivo y terminal. Según el mismo razo-namiento que en el último capÍtulo, observamos que la clase de proposi-ciones que son válidas en un modelo de tipo G deben ser de tipo G, y enconsecuencia el sistema modal apropiado es el sistema modal G. De estemodo obtenemos el llamado teo¡ema de la solidez para el sistema modalG: Toda proposición demostrable en G es válida para todos los modelosterminales transitivos. La recíproca de esto --el teorema de la completitudpara G-también ha sido establecida por el lógico Krister Segerberg. Diceque todas las proposiciones que son válidas en todos los modelos de tipoG son demostrables en el sistema modal G.r

Como análisis filosófico del concepto de necesidad, el'sistema modalG parece muy inapropiado. Su verdadera importancia radica en la in-terpretación de la demostrabílídad, que analizaremos en el próximo ca-pítulo.

Sístema de Iewís Sr. l,ewis tenÍa varios otros sistemas de lógica modal,uno de los cuales menóionaremos brevemente. Lewis nazonó que toda pro-posición que es necesariamente verdadera también debe ser ve¡dadera.(En términos de Leibniz, si una proposición es verdadera en todos losmundos posibles, indudablemente debería ser ve¡dadera en éste.) y porlo tanto Lewig agrsgfi como axiomas para K. todas las proposiciones dé hfo¡ma ND(>X. Esto se conoce como sistema modal 5..

La teoría de los modelos apropiada para S. es un inodelo transitivo deKripke (pero no uno terminal) con la condición adicional de que todo

i Ias demosrraciones de los teorernas de compleritud para las lógies modales K, I( yG pueden encontrarse en 7he tlnprcuabílitjt of coisisreztgr de Georgé noolm (cambiidieunivers.ity press, 1979), y una demosúación muy simplifi cada para G puede encontrarselnconPubaury and Logic de Georye Boolos y Richard c. Jeffrel(cambiidge university press,1980, segunda edición).

112

rnundo es accesible a sí mismo. Resulta entonces fácil observar (lue NXrXes válida en tal modelo.

Los sistemas modales S. y G forman una genuina divisoria tlc aguas.Resulta imposible combinar los dos sistemas para farmat un solr ¡ sistemasin obtener un sistema inconsistente Qpuede el lector descubrir ¡x>r qué?).Y por lo tanto dcbemos hacer una elección, según nuestro pro¡rírsito.Como análisis filo.sófico del concepto de verdad necesaria, el sistt:rna S.parece el apropiaclo. Parala teoría de la demost¡ación, el sistema (l c:.s elimportante. Pero vcrcnros más sobre esto en el prÓximo capítulo.

Ejercicio 1. ¿Por qtré c.s imposible combinar los sistemas G y Sn sinobtener una inconsistcn( ir?

Ejercicio 2. En un moclckr <lc tipo G, ningún mundo puede ser accc-sible a sí mismo. ¿Por qué?

Ejercicio 3. Demostrar quc crr rrn modelo de rípo G, hay por lo menosuna proposición p y por lo menos un mundo x tal que la proposición Np:pesfaka en el mundo x.

Ejercicio4. ¿Es lo siguiente verdaclcro o falso? En un modelo de Kripkede tipo G, hay por lo menos un mundo cn el cual la descabellada pro-posición NI 0- es ncccsariamente vcrdadcra) es realmente verdadera.

173

Page 89: SMULLYAN, R. Juegos por siempre misteriosos. España. Gedisa 2008

24

De la necesidad a la demostrabilidad

El siguiente desarrollo importante en lógica modal después de lasemántica modal de Kripke se produjo al principio de la década del 70,cuando se dio a la palabra "necesario" la intetpretación de la dernos-trabilidad. Rezulta sorprendente que esto no se pusiera de moda antes,dado que Gódel lo sugirió en un ensayo muy breve publicado en 7933.Gódel utilizó el símbolo'B" en lugar de la'N" de Lewis, y sugirió que Bpfuera interpretada como p es demostrable (e¡ el sistema de la Aritmética,o en alguno de los sistemas estrechamente relacionados investigados porGódel). Ahora bien, todos esos sistemas son de upo 4, y por lo tanto losaxiomas de It son todos correctos según esa interpretación. Sin embargo,los sistemas matemáticos en investigación resultaron ser hasta de tipo G(como descubrió Lób), y así resultó simplemente nan¡ral inventar unsistema axiomático modal que los abarcara. De este modo nació el sistemamodal G. Lo han estudiado varios lógicos, incluyendo a Claudio Bernardi,George Boolos, D. H.J. de Jongh, Roberto Magari, Franco Montagna,Giovanni Sambin, Krister Segerberg, C. Smorynski y Robert Solovay. Lainvestigación sobre este sistema aún continúa.

A esta altura, nos será :ú¡il analizar los sistemas axiomáticos modalescon más rigor. El simbolismo de la lógica modal es el de la lógica propo-sicional, con un nuevo símbolo adicional: consideremos que este símboloes '8". (Recordemos que Lewis utilizó el símbolo 'N", y el símbolo másestándar en la actualidad es " E ". Pero prefiero usar el simbolo "B" deGódel.)

IJnafórmula modal -dicho en forma más breve, una fórmula- es unaexpresión que se construye según las siguientes reglas:

1) l- es una fórmula y cada una de las variables proposicionales p, q,r, ... es una fórmula.

2) Si X e Y son fórmulas, también lo es Qb\O.3) Si X es una fórmula, también lo es BX.Lo que en el capÍnrlo 6 se denominí fónnula ahora puede llamarse

fórmulaprcposicional. Una fórmula proposicional es un caso especial defórmula modal; es una fórmula en la cual el símbolo B no aparece. Perode ahora en adelante nos ocuparemos de las fórmulas modales y las lla-maremos simplemente fónnul as.

t74

Los conectores lógicos -, &, v, f, = s€ definen.a partir de r y I comose explicó en el capín-rlo 8.

Cada sistema modal tiene sus propios axiomas. En cada uno de lossistemas modales que vamos a considerar, se empieza con los axiomas yse demuestran sucesivamente nuevas fórmulas por medio de las dos reglassiguientes:

Regla 1 (conoclda corno tnodus ponens). Después de demostrar X yCb)), podemos inferir X.

Regta 2 (conocldacomonecestdad). Después de demostrar X, podc-mos inferir BX.

Una demostración formal en el sistema significa una secuencia flnitade fórmulas (que gc:rx:ralmente se escriben verticalmente y se leen <i<:arribahtciaabaio),llarnadas líneas de la demostraciÓn, tal que cada líncaes un axioma del sistcrna, o proviene de dos líneas anteriores de la demcls'tración según la Re¡¡ll l, o proviene de alguna línea anterior de lademostración según la ltc¡¡la 2. Una fÓrmula X se denomina d.emostrabk:en el sistema si hay una <l<:mostración formal cuya última línea es X.

Los tres sistemas (lr.r('s()n de particular interés para nosotros son lo.ssistemas K, Ly G, cuyos :¡xiomas repasamos más abajo.

A)ciomas de K: 1)'lixl¡s l:rs tautologías.2) Todas las fórmulas rlc l:r forma BOb]))(BXrB1).

Axiornas d'e K.: Los (lc l( lrrtlt() con3) Todas las fórmulas rlr' l¡ firrma B)bBBX.

Axlomas de G: Los <le K. ltrrtlo con4) Todas las fórmulas tlc l¡ l,r¡r¡¡a B(B)b}OrBX.

Comsntarío. Vamos a trlrllrnos a los axiomas del grupo 4) comoaxiomas apectales de G. ltt'r tttrlct¡tt¡s cl Teorema de Kripke-de Jongh-Sambin del capítulo 18, <¡re rlkr'r¡trt: si un sistema de tipo 3 puededemostrar todos los enunclsth¡e rlc l;t forma B(B>bD>BX, entoncestambién puede demostrar t(xl('F Lrc rlr lr fbrma BX=BBK De este modo,alternativamente podríamos lrelx'r r .¡¡5i111'¡26[9 como nuestros axiomaspara G aquellos de los grupor l), J ) y 4 ), l;rs fórmulas del grupo 3) hubie-ransidoentoncesdeducibles. 11¡t ¡¡ltae lrrl.tlrras, si agregamos los axiomasde 4) a los de K, envez de K,, lrrrlavil {)l, l(:ndríamos el sistema modalG completo. El sistema G se ¡ttcinettll gctrrt;tlmente en esta forma alter-nativa.

Análtsis. El conocimiento d6 Fálná álá1.!rrr,rs modales proporciona in-formación $obre los sistemas máB r nfl¡tllreg rle lr matemática. El sistema

L75

Page 90: SMULLYAN, R. Juegos por siempre misteriosos. España. Gedisa 2008

v

modal K es válido para todo sistema matemático S de tipo 3 (si inter-pretamos BX como 'X es demostrable en S". De modo simila¡, el sistemaáxiomático Kn es válido para todo sistema S de tipo 4, y el sistema axio'mático G es válido para todo sistema S de tipo G. Por lo tanto esos sistemasaxiomáticos modales proporcionan informaciÓn útil sobre la demos-trabilidad en los üpos más comunes de sistemas matemáticos.(que son nomodales). Hoy en día los científicos de la computación también se inte-resan en los sistemas axiomáticos modales por el siguiente motivo. Ima-ginemos una computadora programada pata imprimir varios enunciados,álgunos de los cuales son afirmaciones sobre 1o que la computadora puedey ño puede imprimir. Entonces la interpretaciÓn de BX se convierte en: "Lacomputadora puede imprimir X". Esas computadoras son, por decirlo así,"autóreferentés" y, por eso, son de interés para los que trabaian en inte-ligencia arüficial. Vamos a considerar esos sistemas en un capítulo pos-

terior.En gran parte de este libro hemos tratado la "creencia" como una

moda[dád. Empezamos usando'8" para "creído" (por un razonadordetipo apropiado). ta lÓglca modal nos permite un trato unificado de losrázonádoies que cteenproposiciones, computadoras que pueden lmpri'mlr proposióiones (o mejor dicho, enunciados que los expresan), ysistemas matemáticos que pueden demostrar proposiciones.

Slstemas modales oractonala.rJna oracíÓn modal es una fÓrmula mo-dal en la que no

^parece ninguna de las variables proposicionales p, q, r,...

expresiones como BJ:I, B(lrBI). Por lo tanto todas las oracionesmódales se construyen a parür de los cinco símbolos B, I, l, (,).Un sistemamodal oraclonal significa un sistema modal cuyos axiomas son todosoraciones (a partir de lo cual se sigue fácilmente que sÓlo las oraciones sondemostrables). Para todo sistema modal M, suponemos que M es esesistema cuyos axiomas consisten en todas las oraciones que son axiomasde M, y cuyas reglas de inferencia son las mismas que las de M (gene-

ralmente serl¡ modus poners ynecesldad).Vamos a interesarnos parti-cularmente en los sistemas modales oracionalesX, Kr, yG.No resulta difi-cil demostrar que si M es alguno de esos tres sistema's, entonces toda o,ra-ción demostrább en M también es demostrable en M. (l'l lector podríaprobar esto ahora como ejercicio: la soluciÓn se dará más adelante, en elcapítulo 27, anando necesitemos usar este hecho.)

Volveremos al estudio de sistemas modales después de tratar elfascinante tema de la autoreferencia, el cual veremos en el siguientecapítulo.

Parte X

EL I\UCLEO DE LACIJESTIOI{

176

Page 91: SMULLYAN, R. Juegos por siempre misteriosos. España. Gedisa 2008

25

Un uniuerso gódelizado

Ahora consideremos lo que puede llamarse el núcleo de la cuestión;es decir, la autorreferencia. Todavía no le hemos dado al lcctor ningunaidea de cómo Gódel se las ingenió para construir una oración autorrefc-rente: una oraciÓn que afirmaba su propia no demostrabilidad cn cl sistcntaen consideración. Lo hizo inventando un método sumamente ingcniosoconocido como diagonalización. En este capítulo y en el próximo vattrosa considera¡ el argumcnto diagonal de Gódel en varias formas.

UN I.JNIVERSO GÓDELIZADO

Consideremos un u¡rivcrso con una cantidad ínJinitade razonadores.También hay infinita carttirlr¡tl de proposiciones sobre ese universo. Demodo más específico:

1) I es una de esas ¡rnr¡rosit'iones (y, por supuesto, es falsa).2) Para cada una dc ersits ¡rlo¡rosiciones p y cada razonador R, la

proposición que R cree p eg ttttá rlr: s5¿s proposiciones.)) Para proposiciones cltálesr¡rrit'ra p y q sobre el universo, pDq es

nuevamente una proposiciótl ¡¡ulrrc r'l r¡niverso y es verdadera si y sólo sip es falsa o q es verdadera,

De ahora en adelante usar¡ton la p,rl,rlrrl "proposición" para significaruna proposición sobre el universo, lk:l i tt irrros los conectores lógicos -, &,v, = a partir de = y I como en cl eflFltt l lo H

Para todo razonador R y todt ¡tru¡lralt lirrr ¡r, .sca Rp la proposición queR cree p. Para razonadores cual€lqUlere lt y 5 y <:ualquier proposición p,RSp es la proposición que R cr('é qu€ E ¡rc'e f r Si incluimos otro r^zona-dor K, KRSp es la proposición K eree que ¡t { r('r (luc S cree p, y de mo-do similar si agregamos más razon¡dcfea

Los razonadores se divertlan nlu€hq f€Érrt¡lrrk¡ sobre esas propo-siciones, pero las cosas eran algo erétl€l: ltarta r¡ric rrn lógico de otrouniverso visitó su universo v puúo h¡ g*A¡ €ll trtrlerr. l.<l primero queadvirtió fue que los razonadorc$ no tÉBfÉñ ñgñLrrC, y ¡rr ir k I tanto le asignóun número a cada razonador (es akélfi gñ ñÉ:*i€r{r errtr,r'o Jrositivo) co-

179

Page 92: SMULLYAN, R. Juegos por siempre misteriosos. España. Gedisa 2008

tnocido como nutnerc fu GÓdet del razonador. Razonadores distintos

i.r.r-,16*ror disüntos y cada número era el número de algún razonador.

¡ft"o f* razonadores tenían nombres: \ es el razonador cuyo número-es

ilR";;á;;nador cuyo número esz,i para cada n' \es el razonador

cuyo número es n.--' Después el lÓgico ordenÓ todas las proposiciones sobre el universo en

cierta iücesión inñnita P,, Pz, '.', P^," Piraiada n, el número.n se conocía

como el número de C6üei'de lá proposición p"' Después de hacer esas

aot"t, "t

lógico se fue y regresÓ a su r¡niverso natal'--l;;;

drpués de ú p;uda de1 lógico, el más despierro de los habi-

tantes se dio cuenta del siguiente hecho curioso'

Hecho l.Para cada razonador R, había un razonador R' tal que para

cualquier proposiciÓn p,, el razonador R' creía p, si y sólo si el razonador

iT;H;5'ñ';i;;,. 6"-atit' par:a cada nzoiadot R' había un razona-

;r;t ,r-i;;p"i" táh" númeró i, la proposición R.p¡RR,p,es verdadera.)

Este hichó tiene algunas consecuencias interesantes' como revelaran

los siguientes Problemas.

L EI princiPio diagonal

Demostrar que para todo razonador R, hay por lo menos una

prop*i.iO" p tal que'pnp es verdadera (en otras palabras, p es verdadera

si y sólo si R cree P).

2. Razonadores ine|tos

Un razonadof de este universo se denomina tOtalmente ln@to si cree

todas las proposiciones falsas y no crée ning:na verdadera'

Demostrar que ningún rázo¡ador de este universo es totalmente

inepto.

Se descubriÓ otro hecho importante sobre este tmiverso poco después

de la partida del lógico.

Hecho 2. Pata czda rezonador R y cada proposición q'-hay. algún

razonadorStalqueparacualquierproposiciÓnp,ScreepsiysolosrKcreep=q. (Por lo tanto SPR(p>S) es verdadera')

J. Principio de Tarski

Unrazonadordeesteuniversosedenominaperfectosicreetodaslaspropásiciones verdaderas y no cree ninguna falsa. (Es diamet¡almente

!opuesto a un razonador totalmente inepto') r - i: -- -^-

Duranteañoslosrazonadoresdeesteuniversosededicaronala.

180

búsqueda de un ¡azonador perfecto (de su propio universo), pero nopudieron encontra¡lo. ¿Por qué no pudieron?

La próxima cosa importante para descubrir sobre este universo es queciertas proposiciones se denominan establecidas y que hay un razonadorE que cree las proposiciones establecidas y sólo ésas.

4

Suponiendo quc todas las proposiciones establecidas son vcrdadcras,demostrar que el conjunto dc proposiciones establecidas es incomplcto: csdccir, debe haber por lo menos una proposición p tal que ni p ni -¡r crsestablecida. (Esto significa tambión que el razonador E nunca puedc crccrp y nunca puede crecr -p. Dcbe ¡esultarle por siempre indecidible si pr csverdadera o no.)

Luego se revela¡on los dos hechos siguientes.

I{echo I. Para todo razonador R, hay un razonador R' tal que para todonúmero i, la proposición R'p¡RR,p es establecida. (Esto se diferencia dclHecho 1 en que ahora decimos eslablecidaenvez de uerdadera.)

Ilecho II. Para todo razonador R y toda proposición q, hay un razoÍra-dor S tal que para todo número i, Sp¡R(p,rq) es establecida. (Esto sedilerencia del Hecho 2 en que decimos establecida eívez de verdadera.)

5

Demost¡ar que para todo razonador R, hay una proposición p tal quela proposición p=Rp es establecida.

6

Supongamos que el con.junto dc las proposiciones establecidas es detipo 1.

a) Demostrar que para todo razonador R y toda proposición q, hay unaproposición p tal que Ia proposición FR(prq) es establecida.

b) Demost¡ar que para todo razonador R y toda proposición q, hay unaproposición p tal que la proposición ¡>(ltp:lq) es establecida.

Por úlümo, se dcscubrió el siguicntc hc<:ho

Hecho II. El razonador E era de tipo 4 (y (:n (:onsccuencia el conjuntode proposiciones establecidas e¡a de tip<> t¡1.

1it 1

Page 93: SMULLYAN, R. Juegos por siempre misteriosos. España. Gedisa 2008

7

Demostrar que el conjunto de proposiciones establecidas es de üpo G.

Ahora observamos que el conjunto de proposiciones establecidas de

este universo es de üpo G, y en consecuencia, si este coniunto es con-

i¡i".rt , el hecho de que sea consistente, aunque verdadero, no es una de

las proposiciones estáblecidas del universo. De modo equivalente, si el

razonaCior E es consistente, nunca puede saberlo.

RELACION CON tOS SISTEMAS MATEMATICOS

El lector podría preguntarse qué tiene todo esto que ver con la teoría

de los sistemis matémáticos. Bueno, supongamos que tenemos un siste-

ma matemático S con todas sus proposiciones ordenadas en cierta suce-

sión infinita p,, p,, ..., p-,... Ahora, en lugar de razonadores vamos a con-

siderar aerái iñpieü¿es de las proposiciones; esas propiedades tam-

bién se ordenan en cierta sucesiÓn infinita \' R, "'\,"' Para todapropiedad \ y toda proposición q, sea ahora \p, la proposición

-que la

propieAaa R, a uállda para la pro¡iosición P,' Supongamos tambre.n que

ia f,ropiedaá -[amémosla E- d¿ ser una froposición demosnable del

sisie-a es una de las propiedades en la lista y supongamos que los H_echos

I, II y III son válidoi reemplazando las palabras "razonador" por;propi"O"a' y "establecida" po. ¿de-ost able" (es decir, demostrable en S)'

e^nonces poi un simple cambio de terminología, los argumentos anteriores

de este cápítulo demuestran que el sistema S debe ser de üpo G'

En realidad, los sistemas investigados por GÓdel no comenzaron con

orooiedades de proposiciones sino con propiedades de números' Sin

!-É"rgo, poret -etodo

de Gódel para numeración de proposiciones,.todapropleáad^ de los números coffesponde_ entonces a cierta propiedad de las

proposlciones: para toda propiedad A de númetos, supongamos que. A es

ia propiedad que es válidá páralas proposlciones. p, tal que A es válida pa-

,." el .tú-..o i. Daremos un ejemplo concreto de esto en el prÓximo ca-

pítulo.' La autorreferencia también puede lograrse sin la numeración de

Gódel, como se indica en el siguiente ejercicio'

Ejercicio 7. Existe ot¡o universo de tazonadotes en el que no importa

si h cáruidad de razonadores es finita o infinita. Algunos de los razonadores

son inmortales, pero ningún razonadot sabe cuáles razonadores son in-

mortales, y *ítér son mórtales. En realidad, ningún razonador sabe si é1

mismo es mortal o no. Para todo razonador R, supongamos que R es la pro-

posición que R es inmortal. Para todo razonador R y S, supongamos que

ñS es ta proposición qveg cree que S es inmortal; para cualquiera de los

r82

tres razonadores R, S y K, supongamos que RSK es la proposiciÓn que Rcree que S cree que K es inmortal, y así sucesivamente (si participan másrazonadores).

En lugar del Hecho 1 del último universo, tenemos el siguiente hechopara este universo: Para cada razonador R, hay un razonador Rr tal que paratodo razonador S, el razonador R' cree que S es inmortal si y sólo si R creeque S se cree inmortal (de este modo R"Ses verdadera si y sólo si RSS esverdadera).

Dado un nzonador R, entontrar una proposición p tal que p seaverdadera si y sólo si R cree p.

Nota; Este método de lograr autorreferencia sin la numeración deGódel pertenece al área de la lógica combinatoria. En mi \bro To Mock aMochlngbird puede encontrarse un sinnúmero de problemas autorrefe-renciales relacionados con este campo.

SOLUCIONES

1. Coruideremos un razonador R. Según el Hecho 1, hay un razonadorR' tal que para todo número i, el razonador R' cree q si y sólo si R crccla proposición R,p,. Ahora bien, R' tiene cierto número de Gódel h, y porlo tanto R* es el rzzonador Rn. Por lo tanto para todo número i, lo siguicntces verdadero:

1) Rn cree p, si y sólo si l l crt:c: <¡ttc It, crcc p. l)ado quc cs() cs ( i(:r l()para. tod.o n:úmero i , tambión ( : .s < i t ' r lo t t ¡ : t t t< l t l i t :s t : l r t í ¡ t ¡ t<:r t ¡ l t . l ' t ¡ t l t ¡ l : t t t lo

tenemos:2) Rn cree pn si y sól<t s i I ( t ¡ r ' r ' t ¡ t t t ' l ( , , r t r ' t ' ¡ r , , . l i r t lot t ( ( 's l ( ) t r l : l t l t ( )s (r()

mo p la proposic ión quc l l r , ( r ( i ( ' l ) r , , y t t l r r , r ' tv . l t t tos ( l l ¡ ( ' l ) ( 's v( ' r ( l i l ( l ( : tA s l

y sólo si R cree p.

2. Acabamos de vcr <¡t tc: l ) i t ¡ l l ( ) ( l ( ) !J/r , t t , r r lor l ( , I r : ry rrrra ¡ l roposic iónptalquepesverdadcrasi ysolr>si I t r t l r ' ¡ r l " lo: , r ¡ i r r r l r t ' : t<¡ucunodelosdos casos siguientes dcbc vrt l t ' r : l ) ¡r t" ; r ,r ' r , l ¡ ,1¡ '¡ ,¡ y l{ trcc p',2> p es falsayRnocreep.Si l )esvál id:r , ( ' t l l ( ) t l (c.s l ( ' t , ' ! '1, , ' r l ( ) rn( : t l ( ) .sunaproposic iónverdade¡a--esdecir ,p-y( : t t ( ( ) l ts( : ( t ¡ r ' i l , t . t l (no(:¡ ; totalmenteinepto.Si2) es vál ida, entonces hay ¡r rr l t ¡ n tr ' ¡ t , ,1 r n.r l ,r o¡losición falsa ----es decir,p-- tal que R no c¡ee p; cr) ( ( )nrr('( r.r('n( r,r l( | rr ' I t(c lodas las proposicioncsfalsas, y por lo tanto nu(:v l t t t l ( ' l l l t ' l ( t ¡ ' , ¡ ' , l r ) l . l l l r ' ¡cnte inepto.

3.Usandoel I{echo2,r( ' ( ' ¡ ¡ r l ) l , r7. lnr , ¡ ' , ¡ l lx)¡ laproposic iónI . Entonc:crspara todo razonador R, h:ty trrr r ;rz( )rr.r , l , ' r l ( ' ( l lamado "S" en el Hecho 2)quecreeesasysóloesasf)r() l x) i ¡ r ( r { ) t ) (" 'p l . r l ( lueRcreep=I.(Puededccirse que esa clase de razonat lor l ( 's t ' ry ' ' ¡ ¡ r r r ' a R.)

Ahora, supongamos t¡rrt : l ( l rrcrt ¡rr ' r lcr ' to. Entonces para toda pro¡-r<rsic iónp,Rcreep:J-siy.s()Lrsi l ) r l t ' : r ' , ' r r l : rdera, loqueasuvezescicr t ( )

l f i1

i

Page 94: SMULLYAN, R. Juegos por siempre misteriosos. España. Gedisa 2008

tsi y sólo si p es falsa. Por lo tanto R cree p>J- si y sólo si p es falsa. Además,R' cree p si y sólo si R cree p>J-. Reuniendo estos dos últimos hechos, R'cree p si y sólo si p es falsa. Esto significa que R' es totalmente inepto.

De este modo observamos que si el universo contiene un razonadorperfecto R, entonces también debe contener un razonador R' totalmenteinepto. Pero hemos demostrado en el problema 2 que el universo no con-tiene ningún razonador totalmente inepto. Por lo tanto el. universo noconüene ningún razo¡ador perfecto.

4. Decimos que un ruzonador es slempre cotrecto si no cree ningunaproposición falsa. Ahora consideremos cualquier razonador siempre co-rrecto R. En el problema 3 observamos que ninguno de los razonadores esperfecto, en consecuencia R también es imperfecto. Esto significa que Rcree alguna proposición falsa, o que no cree alguna proposiciÓn verdadera.Dado que R es siempre correcto, no cree ninguna proposiciÓn falsa, enconsecuencia debe ocurrir que R no cree alguna proposiciÓn verdadera.Esto demuestra que para todo razonador siempre correcto R, hay por lomenos una proposición verdadera p que R no puede creer. Dado que p esverdadera, -p es falsa, en consecuencia R, por ser siempre correcto, no cree-p tampoco. Por lo tanto, el sistema de creencias de un razonador siemprecorrecto R es incompleto. Existe por lo menos una proposiciÓn p ul queR ni cree p ni cree -p; debe resultade por siempre indecidible si p esverdadera o faisa.

Si suponemos que todas las proposiciones establecidas son verda-deras, el razooador E es siempre correcto (porque cree todas las propo-siciones establecidas y ninguna otra). Por lo tanto hay una proposiciÓn ptal que E ni c¡ee p ni cree -p; en consecuencia ni p ni -p es establecida.

5. La demostración es esencialmente la del problema 1 usando el He-cho I en lugar del Hecho 1.

Dado un razonador R, hay un nzonador \ (llamado R') tal que paratodo número i, la proposición \p,=RRp, es establecida. En consecuenciaRnpn=RRnpnes establecida. Por lo tanto pRp es establecida, donde p es laproposición Rnpn.

6. Supongamos que el conjunto de proposiciones establecidas es detipo 1.

a) Conside¡emos cualquier razonador R y cualquier proposiciÓn q. Porel Flecho 2, hay un razonador S tal que para toda p, ia proposiciónSp=R(p:q) es establecida. Además según el problema 5 (poniendo R iguala S), hay una proposición p tal que p=Sp es establecida. Entonces se sigueque p=R(p>q) es establecida (dado que es una consecuencia lógica de lasdos últimas proposiciones).

b) Nuevamente consideremos cualquier razooador R y cualquierproposición q. Según a), hay una proposición -llamémosla p,- tal quepr=R(pr>q) es establecida. En consecuencia (p,:q)=(R(p,>q):q) es

184

establecida (recurso que ya hemos usado antes), y asl p=(Rp>q) es esta-blecida, donde p es la proposición pr>q.

7. Ahora el dato es que E es de tipo 4, en consecuencia indudable-mente es de tipo 1. Entonces según b) del úlümo problema, parz- fodaproposición q, hay una proposición p tal que Ia proposición p{Ep>q) esestablecida: por lo tanto el conjunto de proposiéiones establécidás ei re-flexfao.Y sabemos del capftulo 19 que todo sistema reflexivo de tipo 4 esde üpo G.

185

Page 95: SMULLYAN, R. Juegos por siempre misteriosos. España. Gedisa 2008

7

26

Algunas notables má'quinas lógicas

LA ULTIMA MAQUINA DE FERGUSSON

Al lógico Malcolm Fergusson de mi libro .The Lady orthe Tlger? le gus-

taba construir máquinas tOg."t para ilustrar los principios importantes de

la lógica y la teotia de la demostraciÓn'É.,

"r" libro se describió una de esas máquinas. En años poster¡ores,

cuando Fergusson se enterÓ de los teoremas de Gódel y LÓb' empezÓ

inmediatamente a construir una segunda máquina, que le encantaba mos-

rar a sus amigos. Para su saüsfacciÓn demostró que la máquina era un¿

máquina consistente y estable de üpo G, y disfrutaba particularmcnte d€-

moslando que la háquina, auñque e.ra .consistente' nunca podía

demostrar su propia consistencia. La máquina ilust'ra en forma muy simple

e instructiva lás ideas esenciales detrás del primero y del segundo Teorema

á" iá1"i"*ptetitud de Gódel así como del Teorema de LÓb' Por esta razÓn'

Á" ."rrrtt" particularmente grato comunicarle los detalles al lector.

La máqlina imprime varias oraciones construidas a partir de diecisiete

símbolos. ios primeros siete símbolos son los siguientes:

nos referiremos a la expresión como la expresión n¡. (por ejemplo, pd esla expresión 16¡; J- es la expresión 21.)

La máquina es autoffeferida en el sentido de que las oraciones queimprime expresan proposiciones sobre lo que la máquina puede y nopuede imprimir. Una expresión se denomina imprirnibte si la máquinapuede imprimirla. El símbolo "p" significa ,,imprimible,,, y para todaexpresión E constn¡ida a parrir de los diecisiete símbolos, si quéremos es-cribir una oración que afirme que E es imprimible, no escribimos pE, sinoPE (es decir, P scguida por el número de Gódel E). por ejemplo, unaoración que afirrna que (Pt_:t_) es imprimible es PGIS; es decir,P412325.

Para expresiones cualesquiera X e y, Fergusson definió la diagona-lizaclón de X con respecto a Y como la expresión CX6,D:>]). el símbolo"d" es la abreviatura de "diagonalización" -y para expresiones cualcs-quiera X e Y, la expresión Pd(XD es una oración que expresa laproposición que la diagonalización de X con respecto a y es imprimible.

Ahora vamos a definir lo que significa que una expresión sea unaoración y lo que significa que una oración sea uerdadera.

1) -l- es una oración y J- es falsa.2) Para expresiones cualesquiera X, la expresión pXes una oración y

es verdadera si y sólo si la expresión X es imprimible.3) Para expresiones cualesquiera X e y, la expresión pd(Ey) es una

oración,y es verdadera si y sólo si la expresión CKaXY) r\) --{ue es ladiagonalización de X con respecto a y- es imprimible.

4) Para toda oración X e Y, la expresión (X¡y) es una oración y esverdadera si y sólo si o X no es verdádera o y es verdadera.

- _Debe entenderse que ninguna expresión es una oración salvo que elhecho de serlo sea una consecuencia de las citadas reglas. Los coneótoreslógicos -, &, v, = se definen a parttr de : y I en la forma que se explicóen el capítulo 8.

Ahora damos las reglas para lo que la máquina puede imprimir. Lamáquina está programada para imprimir una lista infinita de oraci,cnes con-secutivamente. ciertas oraciohes llamadas axiomas pueden imprimirse encualquicr etapa dcl proceso. Entre los axiomas están todas las iautologías.(De este modo para toda tautologia X, la máquina puede imprinir Xsiempre que lo desee, independientcmente de lo que haya o no hayaimpreso en alguna etapa previa.) Luego, la máquina está programada paraoraciones cualesquicra X e Y, si la máquina en cie¡ta etap ,ya ha impresoX _y X1Y, entonces puede imprimir y. po¡ lo tanto la máquina es dé tipo1 (en el sentido de que la clase de oraciones imprimibles es áe tipo 1). Da-doque es cierto que si X y )L:Y son 4¡nbas imprimibles, también lo es y.entonces la oración (pX&p@):py cs verdidera; o, lo que es lo mismo,la oración PObD:(PX:PD es verdadera (las dos braciones sonlógicamente equivalentes). La máqrrina ,'conoce', la verdad de todas lasoraciones de la forma PObD¡(pX¡pS y las considera axiomas, por lotanto Ia máquina es de tipo 2. Luego, si la máquina llega a imprimir una

Pl-r()d,1,2 3 4 5 6 7

Debajo de cada uno de estos siete símbolos, he-escrito su númeto de

Gód,el.Lós diez símbolos festantes son los conocidos dígitos 1,,.2, 3, .4, 5,

6,7 ,8,9,0. A estos dígitos se les asignan los números de GÓdel del siguiente

"i"io U número ¿JCo¿et de 1 ei 89 (8 seguido por un 9); el número de

óáJ"f ¿. 2 es899 (8 seguido por dos 9;y asi sucesivamente' hasta 0' cuyo

.rútn"ro es 8999999g99t (8 s¿8uido por diez 9)' De este modo cada uno

J" foi Ai".lri"t" símbolos tienóun.,úmeto de Gódel. Dada una expresiÓn

compleja se encuentra el número de GÓdel reemplaz¿ndo.tld1:TP.olo

oo, ir, Áú-"to de Gódel: por ejemplo, cl número de Gódel de (PIrr) es

ZtZlZS. Orro eiemplo, el ñúmero de GÓdel de P35 es 7899989D99' Para

rodl expres¡Oín, E significa el número de Gódel de E (escrito como una

Ñfo"¿'" los dígitos I,-2, ...,0). No todos los números son los números de

éáá"r ¿".r.r" é*pr"tióÁ (por e¡emplo, 88 no es el número de GÓdel de

.ti.rg,rtt" expresiÓn). Si n ei el número de GÓdel de una expresión' a veces

186

- ror€id

187

Page 96: SMULLYAN, R. Juegos por siempre misteriosos. España. Gedisa 2008

oración X, osabe" que ha impreso !Y tarde o tfmprTg imprimi-rá la ora-

ción verdadera pX. 0¿ oraciÓn PX es verdadera^dado qu9 I ha siclo

impresa.) Y así la máquina es normal, por lo tanto es de tipo 3' Dajlo qt€

la máquina es normal, entonces paraioda oraciÓn X, la. oraciÓn PTIIPX

;;;ri;;;. p.ilr-i'""tó-h má?pina es iniciatmenr€ "conscienre" de la

verdad de todas esas oraciones y lás considera axiomas a todas' En conse-

cr¡encia la máquina es de tiPo 4.:Hay atgo mas qrre puede hacer la máquina, y esto es absolutam€nte

crucial. Para expresiori"r-á"t.tq.riera x 9 -Y,.1" oraciÓn PdG'D es

"Lrá"J"." si y sólo s (>G,9.=r).és imprimible, lo que a su vez es cjerto

si y sólo si la oración l6&$vl e¿gir¿-a,aera' Por lo tanto la siguiente

oráción es verdadera: pd6,D=p(XG'D¡9'

Lamáquinaconocelaverdaddetodasesasoracionesylasconsideraaxiomas. Esos axiomas se denominan Qciornas díagonales'

Ahora repasemos sistemáticamente todos los axiomas y operaciones

de la máquina:Axiomas.GruPo l.Todas las tautologías.á"ipo z Todas las oracion"es de la forma l@GX>lD'

Crupo 3.Todas las oraciones de la forma PX:Pffi'GruDo4aosax'!!¡mascliagonales).Todaslasoracionesdelaforma

p¿C,YtpCx(kBln, donde i e Y son expresiones cualesquiera (no ne-

cesariamente oraciones).

Reglas de o|eración'1) Los axiomas pueden imprimirse en cualquier er?p^'.2) Dadas las oráciones X y O<:>n ya impresas, la máquina puede

entonces imPrimir Y. . '--3) Dada una oraciÓn X ya impresa, la máquina puede imprimir PX'

Estas son todas las reglas que gobiernan la imprimibilidad de la máqui-

na. Debe entenderse que la única forma en que la máquina puede imprimir

una oración X en una etapa dada es siguiendo unade las reglas citadas.

Es decir, X es imprimible in una etapa dada sólo si una de las t¡es condi-

ciones siguientes es válida: 1) X es un axioma; 2) hay una oración Y tal que

vv (v=ñ hansido ambas impresas enuna.etapaprevia 3) hayuna or1-cióny tal que X es la oraciÓn PY e Y ha sido impresa en una et^pa pfevla'

cornentario.para cadaoraciÓn X, sea BX la oración Px. El símbolo "B"

noeSpartedel lenguajedelamáquina; lousamosparahablardelamáquina. Usamos "5" p:r^ representar la operaciÓn que le asigna a cade

or"éiór, X la oración PX. Cuahdo decimos que la máquina es de- tipo 4'

queremos decir que es de tipo 4 con-referencia a esa operaciÓn B- Esen-

cialmente, sin loi axiomas diagonales, el sisrcma axiomático de esta

máquina és el sistema modal Kn Veremos.que el hecho de agregar los

a*iomas diagonales nos da todo el poder del sistema modal G'

188

Demostrabilidal. Hemos definido para cada oración Io que significaque la oració¡ sea uerdadera, y por lo tanto cada oración é*pr*i" ,,r.r.proposición definida, que podría ser verdadera o podría ser falsa. Decimosque la máquina demuestra una proposición si imprime alguna oración queexpresa la proposición. Por e.jemplo, la oración -p2 expresa la proposiciónque la máquina es consisrente (dado que 2 es el núméro de COd-el de r),y por lo tanto si la máquina imprimiera -p2, demostraría su propiaconsistencia. si la máquina imprimiera p2, entonces demostraría su propialnconsistmcía.

. Decimos que la máquina es siempre correcta sitodas las proposicionesdemostrables por la máquina son verdaderas. Decimos quela máquina esconsistente si no pucdc demostrar I, y que la máquina es estable si paratoda oración X, si PX cs imprimible, también lo erX.

Reflexiuidad.Ahora ¡tascmos a la demostración de que la máquina esgódeliana, en realidad, rcflcxiva.

t. La oracíón G de Gódel

Encontrar una oraci(rn O tal que la oración G=-pG -es decir, lao¡ación G=(PG>l)- c.s inr rrlinriltlc,.

2. lleflcxiuidad

Demostrar que para lotl;r or:¡t iírn y, hay una oración X tal que lao¡ación Xr(PX))) es im¡rrinril rlt:

Soluclones. El problenra I cs r¡n r:aso especial del problema 2; por lotanto primero resolvercmos cl l)r ( )r)r( 'r r;r 2. su pongamos que y eJ cualquieroración. Para toda expresjón Z, l.r 'r;r,. ior.r l,¿CZD=ptffiS es impri_mible (porque es uno dc tc¡s ;r x i,,r¡r,rs r l i :rt ir rnalcs). Rcemplazamos Z pór laexpresión Pd, y por lo ranro I,t lr I ' 'J,\ ') I,(¡,d(p-d,D)y) es imprimiblé. Co_mo la máquina es dg tipo 1 , rt .sr r rr .r t'1.I r ).ccs que la siguieñe oración esimprimible : (p d(p?,D:yF(I, ( I \ | ('T[1Í), r) ->v).

De este modo la o¡ación X 0,X,\ ') r:s imprimible, donde X es laoración (Pd(trd,D:D.

El problema 1 es un caso c.\lx.( i,r l rk' .1, r<:c:ntplazando yporJ_. por lotanto la oración de Gódel g para (..\t,r rrr, ir¡rrir 'r cspd(FE,f)>i, es decir, laoración (Pd(16,2):r).

Observemos más de cerca l:r rrrl:¡ lr lr. rr;rr. ión clc Gódel G. En primerlugar, ¿qué dice la oración I>d(16,¿)) t)rrr.t¡rr<. hr <),iagonalización de laexpresión 16s con respecto a la {:x¡rrt'srírrr ,r' i's ir.¡rrimible. Aho.a bien, laexpresión 1# es Pdy la expresi(rrr Zr r.s l, y ¡ror Lrianro pd(1.6,2)dice quela diagonallzación de Pd con r.(,slx.(tr);¡ I <.s ir 'primible, p"ro.rt"diagonalización es la oración (pcl( l(r,l) , I ), ¡.:; tlt.<,ir., ia propiá óración G.Y por lo tanto Pd(16,2) dice <¡rr t ' ( i t , . r i r r r ¡ r r i r r r i l r l t : , cn-consecuencia

189

Page 97: SMULLYAN, R. Juegos por siempre misteriosos. España. Gedisa 2008

V

(Pd(16,2>r) -que es la oraciÓn G- dice que p no es imprimible (o' lo

;;;;i;ñ-o,'quela imprimibilidadde G implica falsedad). porIo ranro

ó- ¿i." que G no Ls imprimlUte; G es uerdadera si y sÓlo si G -no

es im-

priÁiUf"l Por lo tanto G áfir-a t.t propia no.imprimibilidad' Aquí' en pocas

i"t^Ur^t, está el ingenioso método ¿le Co¿et de lograr la autorreferenci¿'' L^ óración G=--PG ----es decir, la oración G=(PG:I)- no sÓlo es

.r.rd"á"r., sino realmente imprimible (es uno de los axiomas diagonales)'

Oado que'la máquina es normal y de tipo 1,-sigue del Primer Teorema de

t" i.,.o-pt"tinrd de Gódel (Teoráma 1, capítulo 2Q, página 150) que si la

máquina es consistente, entonces G no es imprimible' y si la- máquina

adcirras es estable, entonces -G tampoco es imprimible' Y Po: Io tanto sí

la máquina es consistente y estable al.avez,la oración G es indecrdrDle en

el sistéma de oraciones que la máquina puede imprimir'

Ahora bien, la máquina es cÁ rcalidad de tipo 4' y dado que .eseódeliana -la oración G=-PG es imprimiblc- sigue entonces del

3;ñ;á; T".rema de la Incompletinr<l de Gódel (cuarta parte del Resumen

I*,"capítulo 1.J, págioa 100) que si la máquina.es consistente, entonces no

puede demost.", ,., propi" ionsistencia, es decir, no puede demostrar la

5r*,0" _itz.,rJ"-ar, si ü máquina es consistcnte, entonccs la oración-P2

es uerdadera, y así es otro éfemplo de una oraciÓn verdadcra que la

máquina no Puede imPrimir.Además,'la máquiña es reflexiva (problema 2) y al ser de.üpo 4'. debe

ser de Lób (éegOn ei Teor"*" de LÓb), y por lo tanto para toda oració¡ X'

,ipxrx es im"primible, también lo es X. Entonces se sigue según el reo-

rema Mr, capitulo 18, que la máquina es de tipo G'

dadgi-reOlo si (x(x,D>Y) es imprimible, lo@esierto si y sólo siP(X(X,Y)>Y es verdarl<.ra. Por lo tanto Pdg¡,g=f CXG,Y)>y) es verdadera.

Ahora sabemo.s (¡r(: todos los axiomas de la máquina son verdaderos,pero tenemos que (l(:r¡r().strar que todas las oraciones impdmtbles sonverdaderas.

Recordemos <¡rrr. l;r rrráquina imprime oraciones en varias etapas.Ahora queremos estrlrlr.rr.r el siguiente lema, teorema y corolario.

Lema. Si X es u¡la or:rción impresa en cieria (:ta[)a y todas las oracio-neS impreSas antes (l(: (..s:l (:tapa sOn verdaderas, (:nt()n('cs X también esverdadera.

Teorema 1.Toda orirt i i¡rr irnpresa por la máquina (:s v(.rrl,t( l(,!.1

Corclario. La máquirr:r <'s < on.sistente y estable.

3

¿Cómo se demuestran k¡s t:ita<l<>s lema, teorema y corolario?

Solttciones. En primer lu¡¡ar rl<:mostramos el lema: Considerenros l,rhipótesis de que todas las ora<:i<¡n<:s prcviamente impresas sonverdadcr¡s,tenemos que demostrar quc X cs vcrdadera.

Caso 1. X es un axioma. lintonces X es verdadera (como ya hemosdemostrado).

Caso 2. Existe una oración Y tal quc Y y Cf>X) yahan sido impresas.Entonces según la hipótesis formulada, Y y CY:D son ambas verdaderas;en consecuencia X debe ser verdadcra.

Caso3. X es de la forma PY, donde Y es una oración ge ha sido pre-viamente impresa. Dado que Y ha sido impresa, entonces PY es verdadera;es decir, X es verdadera.

Con esto concluye la demostración del lema.

Demostracíón del Teorerna 1. La máquina está programada paraimprimir todas las oraciones imprimibles en cie¡ta sucesión infinitadefinida Xr,\, ...,\, ...; \significa la oración impresa enlaetapa n. Ahorabien, la primera oración impresa por la máquina (la oración X,) debe serun axioma (dado que la máquina no ha impreso otras oracionés todavía),en consecuencia X, debe ser verdadera. Si la citada lista infinita llegara acontener alguna oración falsa, entonces debe haber un número n menortal que \ sea falsa; es decir, debe haber ufia prirnera oración falsa queimprime la máquina. Sabemos que n no es igual a 1 (dado que X, esverdadera); en consecuencia n es mayor que 1. Esto significa que la máqui-na imprime una oración falsa en una etapa n pero sólo ha impreso ora-ciones verdaderas en etapas previas. Pero esto contradice al lema. Por lotanto la máquina nunca puede imprimir oraciones falsas.

tÁ. CORIiECCION DE LA MAQUINA

Hemos demostrado que si la máquina de Fcrgusson es consistente'

.ntorr.o .to puede demostrar su propiá consistcncia; pero ¿cÓmo sabemos

si ta maquiná es consistcnte o nó? Aho'a dcmostraremos que la máquina

no sólo es consistente, sino siemprc correcta; es éecir, que toda oración

impresa por la máquina es vcrdadcra'Ya hemos demostrado que todos los axiomas de la máquina son

vcrdaderos, pero repascmos cuidadosa-cntc los argumcntos: Los axiomas

aáf C-po i'son toüos kutologías, cn consecuencia son indudablemcnte

vcrdaderos. Con respccto a los axiomas dcl C-rupo 2' dccir quc

P@¡(PÍ>PD es v-ercladera es dccir q¡e si PQ(>Y) y PX son ambas ver-

¿a¿eras, también lo es PY, lo cual signiiica que si ObY) y X son ambas

t,úrt"tttÉr, también to eÁ y. Esto ei obviamente cierto en virtud de la

;é;;.¿;i Por lo tanto los axiomas del Grupo 2 son todos verdadcros'

Con respecto a tos a*ioÁai a.r cryq9 3, deci-be ob,nlT.t:^:t:,99:1"es decir que si pFes vefdadera, también 1o es PPX, lo que a su vez slgnrllca

á;;;ii¿t imprimible, también lo es PX;y esto realmente es cierto' en vir-

tud de la operación 3. Con respccto a los axiomas diagonales Pdff,Y) es ver-

190191

i

Page 98: SMULLYAN, R. Juegos por siempre misteriosos. España. Gedisa 2008

t

DemostraclÓndelco¡olario. Dado que la máquina es siempre correc-

ta (segÍrn el Teorema 1), entonces r nunca puede imprimirse, porque I es

falsa. Por lo tanto la máquina es consistente.Luego, supongamos qrre PX es imprimible' Entonces PXes verdadera

(según á i"oL-á 1), lo cual significa que X es imprimible. Por lo tanto

la máquina es estable.AÉota v.*ot que la máquina de Fergusson es consistente' pero,nunca

puede demostrar sü propia óonsistencia. Por lo tanto ustedes y yo (así co-'rrro n rg.rrron) sabehoi que la máquina es consistente, ¡pero la pobre

máquina no lo sabe!

LA VARIANTE DE CRAIG

cuando el inspector craig (un buen amigo de Fergusson) se enteró de

la máquina de Feigusson, se le ocurrió una interesante variante, que no

,r"."rü de h numéración de Góde1. La máquina de Craig utilizaba simple-

mente los siguientes seis símbolos:PJ-r()Rirrs deiin¡ciones de oraclón y oraciÓn verdadera están dadas por las

siguientes reglas:- 1) I es una oraciÓn y I no es verdadera'i) p^r oraciones cualesquiera X e Y, QblD es una oraciÓn y es

verdadera si y sólo si X no es verdadera o Y es verdadera'

3) Para tóda oraciÓn X, PX es verdadera si y sÓlo si la máquina puede

imprimir X.4) Para oraciones cualesquiera X e Y, la expresiÓn Q(R\D es una

oración y se declara verdadera si y sólo si la máquina puede imprimir

(Q{RxrY)."- -e p.iiri"ta vista podría parecer que 4) es circular, dado que la verdad

de Crcin se define én términos de una expresión que implica la let¡a R;

p.rJ "i"

.it*laridad sólo es aparente. Si-Craig hubiera dichg Oy9 CxnY)

!s verdadera si y sólo si ((xRprta) es aerdadera, la definició-nhubiera sido

circular, dado que no podríamos saber lo que significa que oGY) sea_ver-

dadera sin sabér priméro lo que significa que CXR)g sea verdadera. Pero

Craig no hizo esó. il hecho es que para oraciones cualesquiera X e Y, o

la eipresión (CXruO¡fD es imprimible " lo._E] símbolo "R" representa la

relación que hay entre X e Y cuando (Cxrug,D es imprimible. Por.lo tanto

Cáá "o

áefineia relación R en términos de sÍ misma, sino en términos del

slnúolo"R", y eso no constituye ninguna circularidad'Los primeros tres grupos áe axiolnas de la máquina de Craig s.on los

mismos que los del sistema modal Kn (donde oraclónsignifica oraciÓn en

"t t"rrg,r"i" de la máquina del sistemá de Craig). Por lo tanto son como los

pri-eiás't es grupos'de axiomas de la máquina de Fergusson, eliminando

i"r r"y"t sobñ x,y,z. El cuarto grupo de axiomas de Craig puede inter-

f.Lt"it" .o-o , 4') (axiomas diagonáles de Craig). Todas las oraciones de

la forma QütYFP(CKru0>]0.

792

Por supuesto todos los axiomas diagonales de Craig son oracionesverdaderas.

Las reglas de operación de la máquina de Craig son las mismas que lasde la máquina de Fergusson.

4

a) Demostrar que la máquina de Craig es reflexiva (y en consecuenciatambién de tipo G, dado que es de tipo 4).

b) Encontrar una oración de Gódel parala máquina de Craig (es decit,una oración G tal qrrc C=-PG es imprimible por la máquina de Craig).

Solucíón. a) daclo quc para todas las oraciones X e Y, la o¡aciÓn

CKRY=P((XRIO:Y) cs irnprimible (es un axioma diagonal), entoncestambién es cierto si ocrrrrc que X es la propia oraciÓn Y. Por lo tanto

OaR))=P(0fR)):Y) cs irnprirnible, y entonces también lo es la oración(CíRY)D=(PCTRY)rY) )Y), luego ZÉ(PZ>Y) es imprimible, donde Z esla oración ¡lYgrfrY).

b) Reemplazando I ¡ror Y, obtenemos la o¡ación de Gódel ((-LRl-):l-).

Nota:Los axiomas tlt' l,r rrr:'rr¡uina de Craig también son todos verda-deros, y por el misnr<) r.r /{ 'n.uni( 'nto que usamos para la máquina deFergusson, puede obscrv:us(' ( lu(' t()da oración imprimible por la máquinade Craig es verdadera. l 'or lr) l .rr)to la máquina de Craig también esconsistente y estable (y tl<' ti¡ro t i).

5. La ol¡l¡'t tutt ititt rle McCulloch

Cuando Valter McCullo< lr ( un .rrrri¡',o de Craigy Fergusson) se enteróde la máquina de Craig, hizo l,r :,r¡irrrlrrtc observación interesante: Dadastodas las oraciones X e Y crr l.rs ,¡u,' nr) i{[)arece el símbolo "R", hay unaoración Z enla cual "R" no : l l ) ,rr( ( r ' , Lr l r¡rrr: la oración (XRY=Z) es impri-mible. (Esto implica, en part i( r¡1,u, , l r¡( l , ,uir cualquier oración X hay unaoración X'en la cual "R" no af ):r ( ' ( r ' t .r I r ¡rrc X=X'es imprimible por la má-quina de Craig.)

¿Pueden demost¡ar euc l : t ol r :r ' rv.r, r f ,r) ( lc McCulloch es cor¡ecta?

Solución. En la solución clt:l ¡,r,,1 rk ¡ rr.r l{ i k'l <'apítulo 19 demostramosquetodosistemadet ipoGqut ' ¡ r r t r ' , l t ' r l ¡ t r r r , . l r . r ¡ fER(p)q)puededemos-trar pBq. (Demostramos eso l):u.r l r , : r ,¡ . ' , ,n.r( l() l( 's, pero el mismo argu-mento sirve para los sistemas.) Alror¡ l¡r¡ ' ¡r , r ' l : .r , t<'rna de Craiges de üpoG, ypara toda oraciÓnX, la oraciórr ( X l( I tsl'{ ( i l( \ ) }X) cs imprimible, y porlo tanto si reemplazamos p p()r (Xl(X ) y r¡ ¡r ' r \ , r ,r ' .sigue que QCTXFPXes imprimible. A part ir de esto tc;t t l l . t ,¡r t ' ' {( \ : l ( \) rY):(PX:Y) es impri-mible,yenconsecuencia(por la r t ' ¡ . ¡ r r l , r r t r l , ¡ , l r l ' t { x l ( \ ) ¡Y)=P(PXI))esim-primible. Pero además QOtYtsp((Xltr )

"t ' ) r ' ' ; l ' l ' runrl) l( ' cn consecuen-

193

Page 99: SMULLYAN, R. Juegos por siempre misteriosos. España. Gedisa 2008

cia Q(RYFP(PX>Y) es imprimible. Si ahora el s-rmbolo nRn no aparece ya

sea en X o Y, entonces no aparece en P@X:Y), y por lo tanto tomamosZ igaú a P(P)bY).

27

Sistemas modales autoaplicados

Inspirado por las ntác¡trinas lógicas de Craig y Fergusson, ahora voy aconsiderar los sistemas axiottt/tticos modales según el punto de vista de laslntetpret aciones aubm rtt n I n I c s.

Recordemos quc una t¡rutclón ¡nodal -más brevemente, úrra ora-clón- significa una fórn¡rrl:¡ nrodal sin variables proposicionales. Ahoradecimos que un¿ oraclólr t¡rrxl¡tl cs verdadera para un sistema modal Msi es verdadera cuandointetl)tctitntos que B es detnostrable en M. Por lotanto:

1) I es falsa para M.2) Para oraciones ntorl¡lrs crr;tlcsquiera X e Y, la oración X>Y es

verdaderaparaMsiysólo.cl o X t,r, (:s vcrdaderaparaM oYes verdaderaparaM.

3)Para toda oración X, lt on,'lrrrt llX es verdaderapar^ M si y sólo siX es detnostrable en M.

Nota: Í)na oración X puede Eet tta*ladera para un sistema modal Msin ser demostrable en M, y vlfevetBt. l'or ejemplo, la oración -BI esuerdadera para M si y sólo sl M e¡ r'ott¡¡lslcnte. Decir que -BI es demos-trable en M es decir que M puctle rk,rltohlrar su propia consistencia. Enseguida veremos que el sistcmt f¡lr(l¿l (; Fs consistente, y por lo tanto laoración -Bl- es uerdadera pam G, Ferr¡ le r¡¡ar'ión -Bl- no es demostrableen G. Por otro lado, todo slgtcm¡ mtxlal lttr'onsistente de üpo 1 puededemostrar todas las oracione!, én Éttlrer ur'ttcia la oración -Ba en par-ticular. En este caso la oración -Bl e¡ da*uxltttÍtlc en el sistema, pero nouerdadera para el sistema.

Ahora consideremos un glst6mt Ftotlal M, y un sistema modal M,(posiblemente el mismo que M, o poálblernenlé rlifcrente). Decimos quéM,es correctoparaM, si toda oreclón ÉFmr t¡li Élrlc en M, es uerdad.eraparaui. v aeomoi que r'rñ sistema modrl M e. atftnw|vcitcmente coteitosiM-es coffecto paruM; en otras pelfbf¿É, !l tflclÉ r¡¡er'icrn demostrable en Mes verdadera para M.

Todo sistema autorreferentemen€ €-gfEEtg tlel*, ¡r'.r consistente (por-que si r fuera demostrable en el rlrtrmlrgl tElerrr¿ rro podría ser auto-rreferentemente correcto, dado qu€ I ff felEÉ FÉfÉ el Fliltcma) y tambiéndebe ser estable (porque si BX es demc¡llahh Fn Fl stql¡nra y el sistema

194195

Page 100: SMULLYAN, R. Juegos por siempre misteriosos. España. Gedisa 2008

res autorreferentemente correcto, enton@s BX debe ser verdadera pan elsistema, lo qr.re significa que X es demostrable en el sistema). Y por Io tantotodo sistema autorreferentemente correcto es automáticamente consisten-te y estable alavez.

La corrección autorreferente üene una curiosa característica. Resultaposible que un sistema M sea autorreferentemente correcto; sin embargosi alguno de los axiomas se eliminara, el sistema resultante ya no sería auto-rreferentemente correcto. Por ejemplo, podríamos tener un axioma queafirme que un segundo axioma es demostrable en el sistema; si este segun-do axioma se elimina, el primer axioma podría convertirse en falso.

Algunos sistemas autoffqferentemente correcfos. Recorden:.os lossistemás modales oracionales t, 4 v G descritos en el capÍtulo 24 (soncomo los sistemas K, K., y G, excepto que todos los axiomas se reducena oraciones). Ahora nuestra rne¡a es demostrar que esos tres sistemas sonautorreferentemente correctos (a partir de lo cual, entre paréntesis, podre-mos demostrar que los sistemas K, K. y G son autorreferentementecorrectos).

Si M es alguno de esos tres sistemas axiomáticos K, Kn, G, parademostrar que M es autorreferentemente correcto, basta demostrar quetodos los axiomas de Ii son verdaderos para M; la razón de esto es unaconsecuencia del siguiente lema, que también tiene otras aplicaciones.

Lema A. Supongamos que q es cualquier sistema modal oracionalcuyas únicas reglas de inferencia son modu.s poners (a partir de X y X>Ypodemos inferir lD y la rcgla de necesidad (a partir de X podemos inferirB)). Supongamos que M, es cualquicr sistema modal tal que todas las ora-ciones demost¡ables en M],son dcmostrables en M, y tal que todos los ario-mas de I son uerdadercs paraMr. Entonces toáas las oraciones demos-trables en NII son verdaderas para Mr; es decir M, escorrecto para Mr.

CorolarioA,. Para todo sistema modal oracional M cuyas únicas reglasde inferencia son moúr ponersy necesidad, si todos los axiomas de Mson verdaderos para ñi, entonces )vi es autorrelerentemente correcto.

1

Demost¡ar el Lema A. (Pista: La demostración es esencialmente igualal argurnento que dimos en el úlümo capínrlo para demostrar que todas lasoraciones demostrables de la máquina de Fergusson eran verdaderas, unavez que establecimos que todos los axiomas de Ia máquina eran ver-daderos.)

Solución.Consideremos cualquiersucesión \, ..., X" de oraciones queconstituye una demostración en el sistema {.- Obse'i.raremos que Iaprimera línea X, debe ser verdadera paraM,luego la segunda línea \ debe

196

ser verdadera paralvlr,luego la brcera línea \, y así sucesivamente h¿rstala última.

La primera línea \ debe ser un axioma, en consecuencia es verdaderopara M2 por hipótesis. Ahora consideremos la segunda línea \. O es unaxioma de M, (en cuyo caso es verdadero para M), o debe ser la oraciónP\, 9t cuyo czlso es indudablemente verdadera para It! dado que X, yaha sido demostrada en M' y es en consecuencia demostrable en tr!. Ahorasabemos que las primeras dos fneas son verdaderas para I!. Ahoraconsideremos la tercera línea X^. Si es un axioma de M, o es de la formaBY, donde Yes una línea anteriór 0t oX), entonces \'es verdader^par.aM, por las mismas razones que antes. Si \ no es ninguría de hd'dos cósas,entonces debe inferirse a partir d" 4 y Xrpor mohsponerrs, y dado queya sabemos que X,. y X, son verd¿deras para M, se sigue que \ esverdadera paraM". (Indudablemente, para toda oración X e Y, si X y X>Yson ambas verdaderas para \, entonces Y es verdaderu p ra lvl.) Ahorasabemos que las tres primeras líneas de la demostración son todasverdaderas para M, y al saberlo, la verdad de Xn puede establecerse porel mismo argumento. Entonces, conociendo la verdad de las cuatro pri-meras líneas, obtenemos de modo similar la verdad de la quinta y asísucesivamente hasta llegar a la última lÍnea.i Así concluye la demosüacióndel Lema A.

La corrección autorreferent€ de los sistemas K, En y G se sigue a partirdel Corolario A, y del siguiente lema.

Icrna B. Para todo sistema modal M:a) Si ltt es de tipo 1, entonces todos los axiomas de Kson verdaderos

panM.b) Si M es normal y dc tipo 1, entonces todos los axiomas de Kn son

verdaderos para M.c) Si M es de tipo G, entonces todos los axiomas de G son verdaderos

para M.

2

¿Por qué es correcto cl lcnra I]?

Sohtción. a) Suponf¡anr()s (lr¡c el conjunto de oraciones demostra-bles de M sea cerrado por m<xhts ¡xtneru.Todas las tautologías son obvia-mente verdaderas para M (sr¡n vcrcladcras para todo sistema modal seacual fuere). Los otros axionr¿s <lc K son oraciones de la forrna(BX&BG:>Y))>BY, o altcr¡ra t ivrr I¡v.ntc IIQK:\):(B)bB]D (realmente noimporta). Decir que (BX&I)(X rY)) rllY c.s verdadera para M es decir quesi X es demostrable en M y X rY r.s rlt:r¡rostrable en M. también lo es Y.

rEste üpo de argumento se (r)n(r c t '¡tv¡ tnthu:ción mtemática

r97

Page 101: SMULLYAN, R. Juegos por siempre misteriosos. España. Gedisa 2008

r

Bueno, esto es cierto, dado que las oraciones demostrables de M soncerradas por modus Pone/6.

b) Supongamos que M es un sistema normal de üpo 1. Entonces todoslos axiomas de K son verdaderos para M, por a). Los otros axiomas de K,nson las oraciones de la forma B)bBBX. Pues bien, decir que tal oración esverdadera para. M es decir que si BX es verdadera para M, también lo esBBX. En otras palabras, si X es demostrable en M, también 1o es BX. Estoocurre porque M es normal.

c) Supongamos que M es de tipo G. Entonces es indudablemente detipo 4, por lo tanto por b), todos los axiomas de \ son verdaderos paraM. Los áxiomas restantes de Gson oraciones de la forma B(BX>)Q>BX, ydecir que es uerdaderaparaM es decir que si BX>X es demostrable en M,también 1o es X. En otras palabras, que M es de LÓb. En el capítulo 19demostramos que todo sistema de tipo G es de LÓb. Así concluye la demos-tración del lema B.

Seguimos suponiendo que M es alguno de los sistemas K, Kn, o G.Entonces M es respectivamente K, (, o G.

Codario 8,. Todos los axiomas de Ñi son verdaderos para ili.

Codaio B, Todos los axiomas de M son verdaderos para M.

Denostractones. Dado que K K, yGson respectivamente de tipo 1,no¡males y de üpo 1, y de üpo G, el Corolario B, es inmediato a partir delLema B. Además los sistemas K, K,, y G son respecüvamente de tipo 1, norma-les y de tipo 1, y de üpo G, y por lo tanto también tenemos el Corolario B'

A parür del Corolario B, y del Corolario A,, ahora tenemos el Teo-rema 1.

Teorcma 1. Los sistemas ( x", y dson todos autorreferentementecorrectos.

Corclario. Los sistemas K 4, v C son consistentes y estables.

Ahora tenemos un tercer ejcmplo de sistema consistente y estable detipo G; iustamente, el sistema modal G (los otros dos sistemas son lasmáquinas del úlümo capítulo). Pronto veremos que el sistema modal Gtambién es autorreferentemente correcto (en consecuencia también esconsistente y estable a, la vez).

Espero que ahora el lector advierta plenamente lo absurdo que esdudar de la consistencia de un sistema sobre la simple base de que nopuede demosrar su propia consistencia.

I.;rs slstemas K, Kry G. Para establecer la corrección autorreferente delos sistemas K, Kr, y G procedemos de la siguiente manera. En primer lugar,el lema A tiene otro corolario.

198

Codarío A, Supongamos que M es cualquier sistema modal cuyasúnicas reglas de inferencia son rnodus ponens y necesidad. Entonces, sitodos los axiomas de M son verdaderos para M, todas las oracionesdemostrables de M son verdadcras para M.

Se sigue a p rtir del Corolario { y el Corolario B, que si M es algunode los sistemas modales K, K., o G, entonces todas las oraciones demos-t¡ables en M son verdaderas pára M. Pero eso no es todo. Resta demostrarque si M es cualquiera de esos tres sistemas, entonces toda oraciÓn de-mostrable en M también cs demostrable en M (hecho que fue afirmado sindemostración al final dcl capítulo 19). Una vez hecho esto, estará comple-ta la demostración de la corrccción auto¡refe¡ente de K, K* y G.

3

¿Por quó es cierto que si una oración es demostrable en M (M es K, Kn,o G) entonccs cs demostrable cn Ñ-f

Solución. I)ucdc verse por inspccción de cualquiera de estos tressistemas que si X cs un axioma dc M, entonces rcemplazando las variablesproposicionalcs cn X por oraciones cualcsquicra (poniendo la mismaoración cada vct, (lr¡c aparcce la misma variable, por supuesto), entopcesla oración resultxnl(' r's también un axioma de M y por Io tanto de M.Tomemos ahora urr:r oración particular, digamos T, y para cualquierfórmula X sea X'<: l rr 'srr l l : tdo de reemplazar todas las variables proposi-cionales por T. I)or su l ,r ¡( slo si X es ella misma una oración, tomamos X'igual a X. Ahora ol)sr 'rv,¡nros los sigr.r icntes hechos: 1) Si X es un axiomade M, entonces X'<'s rrrr .rxiorna de M. 2) Para fórmulas cualesquiera X eY, la oración (X¡Y)'r ' : . l . t

' r . t<: ión X')Y', y por lo tanto para fórmulascualesquiera X,Y , y ' / , , : ,r i r", rrrf i :r ible a part ir de X e Y por mochts ponens,entonces Z'es infcri l r l , . .r l r , i l t rr rL: X'e Y'por mOdtSpAnens.3)Paratodafórmula X, la ora<:i(rrr ( llI l' ¡"' l.r oración BX' (es decir, B seguida por X'),y por lo tanto si Y t 's l¡¡ | , ' t t l , l ¡ ' ,r l r l r t i r de X por laregla de necesidad,entonces Y'es dcriv;r lr l r r l ¡ r . ' ¡ ' , rr l : t rcgh de necesidad. Por lo tanto sesigue que dada cual<¡rr icr r ju,, ' : r . ,r X,, , \ de fórmulas, si esta sucesiónconsti tuye una dem(tslt . l larr r rr r ' l : , t : , l r 'nla M, entonces la sucesiÓn \ ' ,X, ' , . . . ,X- 'const i t l tye unJ r l r r r r"r l r r ' r ' ¡n ( 'n M. Y por lo tanto, s i X es unafó'rmulddemostrablc r: l r M, I ¡ | r .r , r ' ' r | \ ' ( " , cntonccs demostrable en Ñi. Si,además, ocurre que X t:s t t t t , t tr t : l r i i ! r , r 'nl()rtccs X'=X, y en consecuenciaX misma es demost¡abft , , , r r ñ l l . ¡ , , ,1, ¡ ¡ ¡ ¡ ¡1.5¡ra que toda oración demos-trable en M es demostralrh' , l [ l

199

Page 102: SMULLYAN, R. Juegos por siempre misteriosos. España. Gedisa 2008

Parte XI

FII\TAL

Page 103: SMULLYAN, R. Juegos por siempre misteriosos. España. Gedisa 2008

28

Sistemas modales, má.quinas Yrazonadores

En el próximo capínrlo tropezaremos con algunos razonadores real-mente muy extraños. Para poder apreciados plenamente' primero consi-deremos el tema de los razonadores mínimos.

RAZONADORES MINIMOS DE VARIOS TIPOS

Una oración modal X no es en sí misma ni verdadera ni falsa; expresauna proposición definida sÓlo una vez que se le da una interpreaciÓn alslmbolo'8": Hemos definido una oraciÓn como aerdadera paraun sistemamodal M si es verdadera cuando "8" se interpreta como de¡nostrabilidaden M. Siempre hemos entendido que una oraciÓn modal es verdadera paraun razonador si es verdadera cuando "8" se interpreta como creída por

el razonad,or.Decir que una oración e s uerdadera para un razonador significa algo

totalmente diferente a decir que escrelda por el razonador. Pór ejemplo,decir que -Bl- es verdadera para un tazonador es decir que el razonadores consistente, mientras que decir que -BJ- es crelda por un razonador esdecir que el razonador cree que es coruistente.

Una máquina puede programarse fácilmente para imprimir,todas¡sólo aquellas-oraciónes demostrables en el sistema modal G dándole a lamáquiÁa estas instrucciones: 1) En cualquier etapa, puede imprimirsecualquier axioma de G. 2) Si en alguna etapa se han impreso las oracionesx y (x>'ó, entonces puede imprimirse Y.,3) Si en alguna etapa se haimpreso X, entonces puede imprimirse BX. (La máquina puede asl recibirmái instrucciones que garanticen que todo lo qúepuede hacer tarde o tem-prano lo hard,,y porloianto toda oración demostrable en Gfinalmente.serái-pres" por la'máquina.) Digamos que tal máquina es una máquina G

- Ahora bien, imaginemos un razonadot que mira constantemente la

salida de información de la máquina. Sin embargo, no interpreta BX como"X es imprimible por la máquina", o como "X es demostrable en B, sinocomo uYo creo X". (Piensa que la máquina está imprimiendo oraciones

203

Page 104: SMULLYAN, R. Juegos por siempre misteriosos. España. Gedisa 2008

qt tsobre é1.)ljace,lo que podríamos llamar una interpretaci ón egocéntrlca rJelas oraciones modales.

. Luego supongamos que el razonador tiene absoluta conFtanzaen quela máquina sabe lo que él cree y por lo tanto siempre que la máquinaimprime una oración X, la cree inmediatamente (según lá interpretáci(rrregocéntrica, por supuesto). Su sistema de creencias incluirá enton cestadn\las oraciones demostrables enG Esto no guantizaque sea de tipo G (puede no ser normal aun cuando. cree que lo es, y hastá puede noier de'ti¡r.1, aun cuando crea que lo es). Por supuesto si cree conectantmte tod,¿llas oraciones demostrables en Q entonces es fácil ver que es de üpo G,

Pero ahora, supongamos que cree todas ,¡ sólo aquellas oracioncsimprimibles por la máquina; su sistema de creencias coincide entoncc¡¡exactamente con el conjunto de oraciones demostrable en G, y dado qucGes de tipo G, entonces él debe ser de üpo G. Decimos'que un'talrazonador es un razonador mínimo de tipo G. Dado que el sistema G esautorreferentemente correcto (como demostramos en ei capín-rlo anterior),se sigue que un razo¡ador mínimo de üpo G debe ser siempre correcto ensus creencias. Además se sigue que todo razonador mínimo de tipo G estanto consistente como estable.

Ahora observemos que el concepto de nzonador estable, consistentcde tipo G noimplica una contradicción lógica. Un razonador de tipo G noes necesariamente consistente (en realidad, todo razonador inconsisten einclusive de üpo 1 también es de tipo G ya que cree todo) pero un razo-nador rnínimo de üpo G es tanto consistenté como establé.

Ahora consideremos un razonador que es de üpo G y que no pierdede vista la salida de información de la máquina (y qüe interpreta toáas lasoraciones modales egocéntricamente). ¿Creeró necesariamente todas esasoraciones (según la interpretación egocéntrica)? Bueno, resulta fácilverificar que cree todos los axiomas dec (En realidad, en el capínrlo 11demostramos que todo razonador de tipo 4 sabe que es de tipo 4; én conse-cuencia creerá todos los axiomas de {. un razonador de üpo G tambiéncree que es modesto, lo que significa que creerá todas las oiaciones de laforma B(BX>}O>BX, y por lo tanto cret todos los axiosras de G.) y dadoque la máquina imprime oraciones que no son axiomas sólo usando lasreglas de modus ponens y neccsidad, y las creencias del razonador soncerradas por moduts ponerls, y es normal, entonces creerá consecutiva-mente toda oración que se imprima. (Si alguien llega a interrumpir elproceso y le pide al razonador su opinión sobre la máquina, el razonadorresponderá: "La máquina es verdaderamente sorprendeñre. Todo lo que haimpreso sobre mí hasta ahora es verdadero.',)

Así vemos que un razonador de tipo G realmente c¡ee todas lasoraciones demostrables en G.

Supongamos ahora, que una oración modal X es creída por todos losrazonadores de tipo 6; ¿se sigue que X es realmente demostrible en G? Larespuesta es sí, dado que si X es creída por todos los razonadores de tipoG, entonces debe ser creída por un razonador tnínimo de tipo G, y án

2Q4

consecuencia debe ser demostrable en G. Y así ahora vemos que t¡ní¡oración es demostrable en G si y sóto st es creída por todos los razona<.lo-

res de tipo G. Dicho de otra forma, cualquier razonador mínimo de üpo

G cree aquellas y sÓlo aquellas oraciones que son creídas por todos los

razonadores de tipo G.porsupuesto, cuando dos razonadores diferentes considQran la misma

oración módal, la interpretan en forma diferente: cada uno interpreta que.8" se refiere a sus propias creencias (del mismo modo en que la.palabra"vo, tiene distintas-referencias cuando la usan disüntas personas). Y por

ló anto cuando hablamos de una oración modal creída por todos los

razonadores de tipo G, queremos decir c¡eída por cada uno según su

propia interpretaciÓn egocéntrica.Por *prr"tto todo l,c que hemos dicho sobre el sistema 1n9$! G y los

razonadorés de tipo G tambien es válido para el siste:na modal Kny los ra-

zonadores de üpó 4: Una oración es demostrable en Rn si y sólo s.i_es creída

Dor todos los raionadores de tipo 4. Del mismo modo, una oración es de-

mostrable en K si y sólo si es creída por todos los razonadores de üpo 3.

MAS SOBRE SISTEMAS MODALES Y RAZONADORES

Los resultados de los problemas en el resto de este capítulo no son

necesarios para compfender los prÓximos dos capín-rlos, pero son intere-

santes por sí mismos.

1

Supongamos que las creencias de un razonador son cerradas por

moaus poñens y qúe para todo axioma x de R], el. razonador. cree X y

también cree que cree X. ¿creerá necesariamente todas las oraciones que

son creídas por todos los iazonadores de tipo 4? (Recordar que puede no

ser normal.j La respuesta se da después del Problema 2'

2

Ahora, reemplacemos Kn por G. ¿Creerá necesariamente el citado

ruzonador todas ias oracionei que son creídas por todos los razonadores

de tipo G?ia respuesta a estos problcmas la proporciona un conocido teorema

sobre los sistemas modalls K.y G (y que támbien se aplica a {y G), que

vamos a enuncia¡ y cuya demostración esbozaremos.

sea M cualquier sistcma modal cuyas únicas reglas de inferencia son

modttsponensynecesidad. Sea M'el sistema modal cuyos axior¡as son los

axiomai de M iunto con todas las fórmulas BX, donde X es un axioma cl<:

z0'i iI

I

Page 105: SMULLYAN, R. Juegos por siempre misteriosos. España. Gedisa 2008

sobreél .)Hacelo que podríamos llamaruna interpretaciÓn eSocéntríca delas oraciones modales.

Luego supongamos que el razonador tiene absoluta conftanza en quela máquina sabe lo que é1 cree y por lo tanto siempre que la máquinaimprime una oración X, la cree inmediatamente (según la interpretaciÓnegocéntrica, por supuesto). Su sistema de creencias incluirá entoncestodaslas o¡aciones demostrables en G Esto no Sarantiza que sea de tipo G (pue-de no ser normal aun cuando cree que lo es, y hasta puede no ser de tipo1, aun cuando crea que lo es). Por supuesto si cree cor-rectarnente tod^slas oraciones demostrables en Q entonces es fácil ver que es de tipo G.

Pero ahora, supongamos que cree todas y sÓlo aquellas oracionesimprimibles por la máquina; su sistema de creencias coincide entoncese*ácta-ente-con el conJunto de oraciones demostrable en G, y dado que6 es de tipo G, entonces él debe ser de tipo G. Decimos que un talrazonador es un razonador rnínimo de tipo G. Dado que el sistema G esautorrefe¡entemente correcto (como demostramos en el capínrlo anterior),se sigue que un razo¡ador mínimo de tipo G debe ser siempre correcto ensus creencias. Además se sigue que todo razonador mínimo de tipo G estanto corlsistente como estable.

Ahora observemos que el concepto de razonador estable, consistentede tipo G no implica una contradicción lÓgica. lJn razonador de tipo G noes necesariamente consistente (en realidad, todo razonador ínconsistenteinclusive de üpo 1 t¿mbién es de tipo G ya que cree todo) pero un razo-oador mínimo de üpo G es tanto consistente como esLable.

Ahora consideremos un razonador que es de tipo G y que no pierdede vista la salida de información de la máquina (y que interp¡eta todas lasoraciones modales egocéntricamente). ¿Creeni necesariamente todas esasoraciones (según la interpretación egocéntrica)? Bueno, resulta fácilverificar que cree todos los axi.omas de G (¡n realidad, en ei capínrlo 11demost¡amos que todo razonador dc tipo 4 sabe que es de tipo 4;en conse-cuencia creerá todos los axiomas de {. Un r¿zonador de tipo G tambiéncree que es modesto, 1o que significa que c¡eerá todas las oraciones de laformi B(Bx:D)Bx, y pór lo tánto cree todos los axiomas de G-.) y dadoque la máquina imprime oraciones que no son axiomas sólo usando lasreglas de modus ponerts y necesidad, y las creencias del ¡azonador soncerradas por modtts poner$, y es normal, entonces creerá consecutiva-mente toda oración que se imprima. (Si alguien llega a interrumpir elproceso y le pide aI razonador su opiniÓn sobre la máquina, el razonadorresponderá: "La máquina es verdadcramente sorprendente. Todo lo que haimpreso sobre mí hasta ahora es verdadero.")

Así vemos que un razonador de tipo G realmente cree todas lasoraciones demostrables en G.

Supongamos ahora, que una o¡ación modal X es creída por todos los¡azonadores de tipo G; ¿se sigue que X es realmente demostrable en G? Larespuesta es sí, dado que si X es crcída por todos los razonadores de üpoG, entonces debe ser creída por un razonador mínimo de tipo G, y en

204

consecr¡encia debe ser demostrable enG. Y así ahora vefix)s (¡rrc unaoración es demostrable en G si y sólo si es creída por todos l<¡s r .r , r ¡nado-res de tipo G. Dicho de otra forma, cualquier razonador mínirrr, ' ttc tipoG cree aquellas y sólo aquellas oraciones que son creídas por /riilo.s losrazonadores de üpo G.

Por supuesto, cuando dos razonadores diferentes bonsidqrirrr l.r nrismaoración nrodal, la interpretan en forma diferente: cada uno intc¡ lrr(.ta quenB" se rc:fir:re a strs propias creencias (del mismo modo en quc l.r ¡'.rlabrauyo" ticnt: tlistintas referencias cuando la usan distintas personr.\) Y poilo tanto (rf:rr)(lo hablamos de una oración modal creída por l{)(l():i losrazonad<rrt's (l(: tipo G, queremos decir creída por cada uno s('¡lrr) supropia intc r ¡,r llación egocéntrica.

Por srr ¡rr rr '',ro todo lo que hemos dicho sobre el sistema morl:rI t i y losrazonadotr ' t , , l , tr¡ro G también es vál ido para el siStema modal (y l 's r ' :rzonadorc.s r i' trl u r .i, Ilna oración es demostrable en I(" si y sólo si <.s < r t.í<lapor todos I , , ' . , r . , , r r r ( l ( ) res de t ipo 4. Del mismo modo, una oraciórr r .s t lcmostrablc crr ii '., 1 ,,rlo si es creída por todos los razonadores dc tiln) 3.

MAS S( ) l t l i l . ' l ' l l lMAS MODALES Y RAZONADOI{I jS

Los rest¡ l l . ¡ ,1" ' . , l ' 1,". I ' roblemas en el resto de este capítulo no sonnecesarios l):u ,r ' , ,r r t

, " , ,, 1, r lo.s próximos dos capín:los, pe¡o son intcrc-santes Por s i t r t ¡ ' . t t ' , ,

I

Suponganros r¡rr l r ' , ¡ . . , ¡ , , . r ' ¡ dc un razonador son cerradas porTnOdUSponens y t l r t t : l i r ¡ . i I ' l ' ' , t \ t r ) ,nax de { , e l razonador cree X ytambién cfee qu(' ( t i r ' ! ' ! , r , ,¡ , , , .rr iamente todas las oraciones quesonc¡eídasport(x l ( ¡ ' ¡ l ' , : . : , . . , , , ,1, , , , . rk ' t ipo4?(Recordarquepuedenoser normal.) La rc ' r ¡ r t t r , ' , t+ " , ' ! r , ! . I ' r i ' ( l ( ' l Problema 2.

Ahora, teemPlatt ' t t r , , , , ! , ¡ ' :

razonador todas las ( )t J ' ti ,r,' ¡

, , , | | r¡ccesariamente el ci tado,, l r ' . t , ( ) r todos los razOnadOreS

de üpo G?La reSpueSta a est()5 prr l , l , , " := i= ¡ , ¡ , ¡ , , , , r . r ur l cOnOCidO teofema

sobre lossistemas mocl ; t l , ' : ! ' , . , " ' ' , . , , '1, ' , ! ! . , ( : r l ) l ica a {yG), que

vamos a enuncia¡ y cuyl t , l r t t t " . ' , ; i ' ' , ! r , , ,

Sea M cualquier s istc l l t ¡ ¡ ¡ i , ¡ ¡ !=!modl,tsponensynecesidatl l ' ¡-¡ l ' ! =i =:: : , 'axiomas de M lunto con t()(1,15 l¿= ! ' , ' , , ' ' : ! '

, , ¡ , l r . ,k. in lerencia son¡ i l , , , , , , ' , . rx i9mas SOn IOS! ' , .1, , . r ¡n axioma de

205

-

Page 106: SMULLYAN, R. Juegos por siempre misteriosos. España. Gedisa 2008

VM, y cuya únlcaregla de inferencia es tnodus poneres. Es obvio que todolo demostrable en M' es demostrable en M (porque para todo axioma X deM, BX t¿mbién es demostrable en M, y por lo tanto todos los axiomas deM' son demosuables en M), pero en general no es cierto que todo lo demos-trable en M sea demostrable en M'. Sin embargo, tenemos el siguienteresultado interesante:

Teotema 1. Si todos los axiomas de Ko son demostrables en M, en-tonces es cierto que todo lo demostrable en M es demostrable en M' (y enconsecuencia los sistemas M y M' demuestran exactamente las mismasfórmulas).

¿Puede el lector descubrir cÓmo demostrar el Teorema 1?

(Písta: Demostrar primero que si.X es demostrable en M, entonces BXes demostrable en M'. Hacedo demostrando que para toda demostraciÓnX,, ..., X. en M, todas las fÓrmulas BX1, ..., B\ son consecuüvamentedemostrables en M'.)

Unademostraciónmás dctallada.Dado que el modtsponens es unaregla de M'y todas las tautologías están entre los axiomas de M', entoncesM' es claramente de tipo 1. Además las t¡es condiciones siguientes sonválidas para M':

1) Si BX y BQ(r't) son demost¡ables en M', también lo es BY, porqueBOb)Dr(BXTBY) es un axioma de M'y M'es de tipo 1.

2) Si BX es demostrable en M', también lo es BBX, porque B)bBBXes un axioma de M'y M'es de tiPo 1.

3) Si X es un axioma de M, entonces BX es demostrable en M', porquees un axioma de M'.

Ahora, supongamos que una sucesiÓn \, ..., \de fórmulas constinryeuna demostración en M. Cada línea de la demostraciÓn proviene o de doslíneas anteriores por modts poners, o de una línea anterior por la reglade necesidad, o es ella misma un axioma de M. Utilizlndo los hechos 1),2),y 3) precedentes, se sigue fácilmente que B\ debe ser demostrable enM'; luego que B\ es demostrable en M', luego que B\ es demostrable enM', y así sucesivamente hasta B\. Dejamos la verificación de esto a cargodel lector.

Ahora sabemos que si X es demost¡able en M, entonces BX es demos-trable en M'. Por lo tanto, si X es demostrable en M', entonces BX es de-mostrable en M' (porque si X es demostrable en M', también es demostrableen M). Y por lo tanto observamos que M' es normal (aun cuando \a reglade necesidad no es un dato para M'). Entonces, dada cualquier demos-tración X1, ..., \ en M, resulta fácil ver que cada una de las líneas \, .-.,X mistnas ouéde demostrarse consecutivamente en M'. (Dejamos lavérificación i , rgo del lector.)

'¿06

Codario. Las fórmulas demostrables de Kn Y K,' son las mismas. Lasfórmulas demost¡ables de G y G'son las mismas.

El siguiente teorema y su corolario pueden demostrarse por un afgu-mento similar.

TeoremaTParatodo sistema modal oraclonal Ñ enel cual todos losaxiomas de {son demostrables, y en el que mod.tts ponent

-y necesidad

son las únicas reglas de inferencia, las oraciones demostrables de M sonlas mismas que las oraciones demostrables de M'.

- Codarío. Las oraciones demostrables de {son las mismas.que las de

il. r"r oraciones demostrables de G son las mismas que las de G.' Por supuesto el corolario anterior da una respuesta afirmativa a los

problemas 1 y 2.Observamos en virtud del corolario del Teorema 1 que los sistemas

modales Kn y G pueden axiomatizarse alternativamente usando sistemasatya única regla de inferencia estrndus poners.

207

Page 107: SMULLYAN, R. Juegos por siempre misteriosos. España. Gedisa 2008

29

Razonadores extraños

RAZONADORES QUE SON CASI DE TIPO G

- Decimos que un razonad or es casi de tipo G si cree todas las oraciones

cqída_s por todos los razonadores de tipo G (o, lo que es lo -ir-",

,i .r."todas las oraciones demostrables en eliistema'modal G) y si sus .rl"rr.i",son cerradas por modus poners. Lo que puede impedidb ser un razona-dor de tipo G es que quizá no sea nórmá1.

como demostraremos, un razonador que es casi de tipo G, a diferenciade un razonador que es de tipo G, puede creer que es consistente sin per-der su consistencia. pero entonc,eJ debe padecár oüo defecto: ;" ¡;;"ser normal.

Ahora vamos a examinar este tema con más detalle.

1

Dado un razonador consistente que es casi de tipo G y que cree quees consistente, encontrar una proposición p tal qrre Ll r.zonrdor cree'p;pero nunca puede saber que cree p.

2

Un razonador anormal debe no creer por lo menol una proposiciónverdadera, porque nay una proposición p ul que cree p pero no puede

:f:l Bp,,^nqu"

lp.gr verdadeia (dado que crle p). poi li, t "ü ";;;;de creer la proposición verdadcra Bp.

Entonces se sigue a partir del último problema que dado un razonadorconsistente que es casi de tipo G y.cree que es consistente, debe haber porlo menos una proposición verdáder" qu" .to puede creér. rraas sorprán-dente es el hecho de que debe haber pór lo ménos una proposición'farsaque sí cree.

¿Qué proposición falsa debe creer?

EJerclcio 1. Determinar si lo siguiente es verdadero o falso: Todorazonador no normal de tipo 1 es consistente.

208

EJerciclo 2. Determinar si lo siguiente es verdadero o falso: Todo ra-zonador no normal de üpo 1 que cree todos los axiomas de I! debe creerpor lo menos una proposición falsa

RAZONADORES QUE SON DE TIPO G*

Como veremos, un razonador que es casi de tipo G no sólo puede creeren su propia consistencia sin ser necesariamente inconsistente; hasta pue-de creer que es slempre cortecto sin ser necesariamente inconsistente.

Decimos que un razonador es de tipo Gr si es casl de tipo G y creetodas las o¡aciones de la forma B)bX (cree que es siempre correcto). Enotras palabras, un razonador de tipo G'es un razonador de tipo 1 que creetodas las oraciones demostrables en G y cree todas las o¡aciones de l¿forma BX>X.

Esa clase de razonador también, por supuesto, debe creer la o¡aciónBI>J- y dado que es casi de üpo G, entonces lo que hemos demost¡adoen la solución de los problemas 1 y 2 también es válido para é1. Y por lotanto hemos establecido el Teorema 1.

Teorema 1. Un razonador consistente de tipo G':a) Cree que es consistente, pero nunca puede saber que c¡ee que es

consistente.b) También cree la proposición falsa B-BI:BB-BI (es decir, -cree

errÓneamente: "Si llego a crccr que soy consistente, entonces creeré quecreo que soy consistcntc").

l¿ recordamos al lcctor <¡rrc la oración B-BI)BB-BI es falsa para unrazonador consistente dc ti¡ro (l', dado que B-BJ- es verdadera (cree quees consistente), pero llll-lll cs falsa (no cree que cree que es consistente).

Por supuesto se dccluc:c a ¡rarlir dcl Teorema 1, que todo razonador detipo G'debe tener p{rr lo nrt.rros rrr¡:r crcencia falsa, porque si es consis-tente entonces la tienc (¡ror t:l 'li'ort.nra 1), y si es inconsistente, sin dudala tiene.

Razonadora mínhrun dt tl/xt ril Llanrlrros sistema modal G. alsistema cuyos axiomas son t(xlls l;rs lTrr ¡r¡r¡lrrs rlt:rnost¡ables de G iunto contodas las fórmulas de la ftrlnr:r llX rX, y t rry;r rirri<a rcgla de inferencia esrnodusponers. Sea G" el sistc:rrr:r ( i 'r 'rryo: :rxiorrr:ts sc rcducen a oraciones:es decir, los axiomas de C' st.rtt lr x l,rs l;rs or,l l ior rt s rlr.nlostrables en G. mástodas las oraclones de la f<¡rrtr;r l lX ,X l,¡ rrr¡t,,r rr.Hllr dc inferencia de Fes modus ponens. (Podentos tlclrror,tr, l l l írr rlr¡u.ltr ' l tr lr medio de unargumento similar al usado cr¡ c:l t .r ¡ rrlu lo , ' / { ¡ tr. l .¡s or ¡rr ' ioncs demostra-bles de Gt son las mismas <¡rrc l ls ttt l tt l ttutt\ r l lr¡¡ostrables de G*.)Llamamos razonador mlnlmo dt: li¡ro ( i' .¡ r¡rr r,¡ ¡' ¡!r,rr lor (lr.rc cree aquellasy sólo aquellas oraciones demc¡stnrlrlt.¡¡ r 'n t l¿ l{r ',,rrlt,r f ¡( i l ( l(:r i lostrar quetodoslosrazonadoresdet ipo(] ' t l t ' lx ' l r { r r ' ¡ ' r 1, , , l , ¡ , , 1,¡ ' , { ¡ r ; r ( i ( )ncsdcmos-

209

Page 108: SMULLYAN, R. Juegos por siempre misteriosos. España. Gedisa 2008

trables en G (.creer" en este caso cae en la inÉrpretación egocéntrica, por

supuesto). Por 1o tanto un razonador es un razonador mlnimo de tipo G'

si y sólo ii cree aquellas y sólo aquellas oraciones que son creídas por to-

dos los razonadores de tiPo G'.Dado que todo razonádor de üpo G. üene por lo menos una creencia

falsa, también la tiene un razonador mínimo de tipo G*. En consecuencia

se sigue qg existe por lo menos una oraciÓn demostrable en G' qu-s

i"tr^"p"r"'F(falsa, iuando "B" se interpreta como demostrabilidad en Gt).

Y por lo tanto tenemos el Teorema 2.

Teorema2. El sistema modal F no es autorreferentemente correcto.

A la luz del Teorema 2, el lector bien puede preguntarse cÓmo el

sistema modal F podría servir pata algo. Bueno, elfrecho de-que existe

una ofación demoit¡able de Gt que es fals^ para G* no significa que no

haya alguna oÍa interpretación de "8" en_ la cual todas las oraciones

demostábles de Ct seah verdaderas. ¿Hay alguna interpretaciÓn así? Sí, la

hay, y es muy importante.

Teorema J. Toda oración demostrable ".t

F es verdaderapara el

sístema modat d.Esto significa que toda oración demos¡able en F egerdadeta si "8"

se tnterprela conxo demostrabilidad m G, en uez de en G*'

3

¿Por qué es correcto el Teorema 3?

El teorema 4 es un fácil corolario del Teorema 3.

Teorema 4. El sistema F es consistente.

4

¿Por qué el Teorema 4 es un co¡olario del Teo¡ema 3?

Por supuesto, el Teorema 4 implica que todo razonador mínímo de

tipo Gr es ionsistente. Y por 1o tanto un razonador mínimo de tipo G* es

cónsistcnte, cree que es consistenle, pero nunca puede creer que cfee que

es consistente (poi el Teorema 2)' Dicho alternativamente en términos del

sistema modal F: es consistente, puede demostrar su propia consistencia,pero nunca puede demostrar quc puede demostrar su propia consistencia.Además, el sistema modal G' no es normal.

La conxpletítud ded paradAhora vamos a enunciar un resultado más

amplio .rry" d.-ortraci6n desafortunadamenté trasciende el alcance de

este libro.

210

Dados dos sistemas modales M, y M, dijimos que Mr es colTectop raM, si toda oración demostrable en M, es uerdadera para M2. Decimos queM, es completo para M, si toda oración que es verdadera para M2 esrealmente demostrable en M,.

El Teo¡ema 3 dice que el sistema modal G' es colrecta para el sistemamodal G. Bueno, también resulta ser cornpleto para G; toda oraciÓnuerdadera para des demostrable en Gr. Y por lo tanto las oraciones de-mostrables de Gt son precisamente las oraciones que son verdaderas paraG. Por lo tanto una oración es demostrable en G' si y sÓlo si es uerdaderapara todos los razonadores de tipo G.

RAZONADORES DE TIPO Q (RAZONADORES RAROS)

Definimos un razonador raro -o razonador de üpo Q- como unrazonador de üpo G que cree que es incoTtsístente. ¿Puede un razonadorraro ser consistente? Pronto veremos que sí. Por supuesto, todo razona-dor raro es normal.

Definimos el sistema modal Q como el sistema modal G con la oraciónBaagregada como axioma. Definimos un razonador míninxo de tipo Q aun razonador que srbe aquellas y sÓlo aquellas o¡aciones demostrables enel sistema modal Q o, lo que es lo mismo, un razonador que cree todas ysólo aquellas oraciones crcídas por todos los razonadores de tipo Q.

Teorerna5. El sistema modal Q no es autorreferentemente correcto,pero es consistente.

5

¿Por qué es verdadeio el Teorema 5?

Por supuesto se deduce del Teorema 5 que todo razonador minimo detipo Q es consistente, aunque cree que no lo es.

Comparación Resulta divertido e instructivo comparar a los razo-nadores mínimos de los tipos G, G'y Q.

1) Un razonador mínimo de tipo G es consistente, pero nunca puedesaberlo.

2) lJ¡ razonador mínimo de tipo G' es consistente, cree que es con-sistente, pero nunca pucdc saber que c¡ee que es consistente.

3) Un razonador mínimo de tipo Q cree que es inconsistente, pero estáequivocado: realmentc cs con.sistcnte.

21,1

Page 109: SMULLYAN, R. Juegos por siempre misteriosos. España. Gedisa 2008

tSOLUCIONES

1. Una proposición p de esa clase es la proposición que el razonadores consistente.

.Se nos dice que cree -Bl y tenemos que demostrar que si esconsistente, no puede creer B-BJ- Bueno, dado que cree todas lasoraciones demostrables en G, ciertamente cree todas las tautologías, y dadoque sus creencias son cerradas por modrc paners, obviamente es de tipo1. Cree -BJ-, por lo tanto cree Bbl-. Si qeyera B-Bt-, creería B(Bl>l_). Sinembargo cree B(BI:l-)>Bt (porque cree todas las oraciones demostrablesen G). Y por lo tanto qeeria entonces B(Bl-rl-) y creeria B(B-bt-):Bl-, enconsecuencia creería Bl-. Pero dado que cree -Bl-, sería inconsistente.

Esto demuestra que si cree B-B-1, sería inconsistente. Pero se nos diceque es consistente; en consecuencia nunca puede creer B-BI (aun cuandoB-BI sea verdadera).

2. B-Bl- es verdadera, pero dado que no la cree, entonces BB-BI_ esfalsa; en consecuencia B-B-bBB-B-L es falsa. Pero debe creer esta oración(porque es de la forma BX¡BBX donde )G-Bl-), en consecr¡encia es unaxioma de G. Y por lo tanto cree la oración falsa B-BJ¡BB-BI-. (C¡eeerróneamente: 'Si creyera que soy consistente, entonces creeria que creoque soy consistente". Esta creencia es errónea, dado que en realidad sí creeque es consistente, Wrc no cree -y nunca c¡eerá* que cree que esconsistente.)

A propósito, por el mismo argumento, cualquíer razooador no nor-mal que cree todas las oraciones demostrables en Kn debe tener por lomenos una creencia falsa. Existe cierta p tal que cree p pero no cree Bp,por lo tanto Bp>BBp es falsa (para esa clase de razonador), sin embargoes un axioma de Kn, y el razonador por lo tanto lo cree. Esto responde elEjercicio 2.

Ejercício 1. Es verdad. Si fuera inconsistenre y de tipo 1, crceúa todaslas proposiciones, en consecuencia no existirÍa ninguna proposición p talque crea p y no crea Bp (porque c¡ee tanto p como Bp, dado que cree todo).Por lo tanto todo razonador inconsistente de tipo 1 debe ser normal. Di-cho de otra forma, todo razonador no normal áe tipo 1 es consistente.

3. En el último capítulo demost¡amos que G es autorreferentementecorrecto, en consecuencia:

1) Todas las oraciones demostrables en dson verdaderas para G.Además:2) Tqdas las oraciones de la forma BX>X son verdaderas p r^d.La razón pa:a 2) es que si BX es verdadera para G, entonces X es

demostrable en G (eso es lo que significa qge BX sea verdadera para G),y X por lo tanto debe ser verdadera para G (dado que G es autorefe-

212

rentemente correcto). Y por lo tanto, si BX es verdadera para G, tambiénlo es X, lo cual significa que BX:X es verdade¡a para G.

En virh¡d de 1) y 2), todo axloma de G' es ve¡dade no par^ G. Dadoque la única regla de inferencia de G' es modus ponenq y dado que elconjunto de oraciones que es uerdadero pa.a dis cerraáo por modusponms(si X¡1X:Y son verdadcras par^ G, entonces obviamente Y es ver-dadera paraQ, se sigue que toda oración demostrable en F debe ser ver-dadera para G. Por lo tanto el sistema G' es corecto para 6.

4. Dado que G es corrccto paraG, entonces si l- fuera demostrableen G', sería verdadera para G,lo cual es absurdo. Por lo tanto _L no esdemostrable en Gi y por lo tanto Gcs consistenLe (aun cuando no es auto-rrcferentemente corrcct o).

5. El sistema Q cs dc tipo G, y por lo tanto por c) del Lema B, capítu-lo 27, todos los axiomas dc (i son vcrdaderos para Q.

Aho13, supongamos morncntáncamcnte que la oración Bl- es verclaclc-ra p ra Q. Fntonces obtcncnl<.r.s la .siguicnte contradicción: Si Bl- cs vcrdadera para Q, entonces dado <¡rrc to(l().s los otros axiomas de O(cs dccir,los axiomas de G) son verdadcros_¡ara Q, tendríamos que lodo.r losaxiomas de Q son verdaderos_¡lara Q. Iint,rncc.s, por el coiolario A, <lt,LemaA, capítulo27,elsistemaQscría autorrc:fcrcntcfircntc corrc(:t(). Y l)()rlo tanto, si Bl- es verdadera para Q, cntorlccs Qcs autorrcfcrcnl(.r¡x.nt(.correcto. Por ot¡o lado, decir cluc IJJ. es vcrclarl<:ra ¡r:rla Q c.s dcc'ir-.r¡ut' Ies demostrabl" gt Q, y dado que J- es obvianlcnlr: fels:r pela (.), r.str,significaría que Q no es autorrcferentementc c()r'rc([(). l)or lr¡ [:ulto t 'scontradicto¡io suponer que BI es verdadera para Q. Iin torrst,t 'ut,nr' i :t l t Ies falsa para Q. Lo cual significa que 1r?oes dcmostral¡lc: cn (1, y ¡ol lotanto Q debe ser consistente. Pero además Bl- es un axion¡a rl<r (]; t:rrconsecuenci4por supuesto es demostrable en Q y, dado quc cs Íalsa ¡'raraQ, entonces Q no es autorreferentemente co¡recto. Y por lo tanto vcmosque Q es consistente pero no auto¡referentemente cor¡ecto.

)11

I

Page 110: SMULLYAN, R. Juegos por siempre misteriosos. España. Gedisa 2008

30

Retrospectiua

Iniciamos este estudio con los razonado¡es introspectivos y hemosterminado en los laberintos de la lógica modal. Resumamos algunos de lospuntos principales que hemos aprendido en el viaje.

1. Un sistema gódeliano siempre correcto de üpo 1 no puede demos-trar su propia corrección: es decir, no puede demostrar todas las propo-siciones de la forma B)bX.

2. Todo sistema gódeliano de üpo 1 que puede demostrar que élmismo es siempre correcto no sólo es incorrecto, sino peculiar; es decir,debe haber una proposición p tal que p y -Bp son ambas demostrables.

3. Todo sistema gódeliano de tipo 1' que puede demostrar su propiano peculiaridad es peculiar.

4. (Según el Primer Teorema de la Incompletitud de Gódel.) Todosistema gódeliano consistente, estable y normal de tipo 1 debe ser incom-pleto. De modo más específico, si S es un sistema normal de tipo 1 y p esuna proposición tal que F-Bp es demost¡able en S, entonces:

a) Si S es consistente, p no es demostrable en S.b) Si S es consistente y estable, entonces -p tampoco es demostra-

ble en S.5. (Segun el Segundo Teorema de Gódel.) Ningún sistema gódeliano

de tipo 4 puede demostrar su propia consistencia.6. Un sistema gódeliano de tipo 4 puede hasa demostrar que si es

consistente, entonces no puede demostrar su propia consistencia; es decrr,puede demostrar la proposición -Bl-¡-B(-Bl-).

7. (Según Lób.) Si S es un sistema reflexivo de tipo 4, entonces paratoda proposición p del sistema, si Bp>p es demostrable en el sistema,también lo es p.

8. Un sistema de üpo 4 es reflexivo si y sólo si es de tipo G.9. Un sistema de tipo 4 es de Lób si y sólo si es de tipo G.10. (Según Kripke, deJongh, Sambin.) Todo sistema de üpo 3 en el que

todas las proposiciones de la forma B(B>bD>BX son demost¡ables debeser de tipo 4 (y en consecuencia de tipo G).

1,1,. Un sistema consistente de üpo G no puede demost¡ar ningunaproposición de la forma -BX; en particular, no puede demostrar su propiaconsistencia.

214

12. Un sistema estable y consistente de tipo G no puede demostrar supropia consistencia ni su propia inconsistencia.' '13.

(Teoremas de la solidez semántica.) Para toda fórmula modal X, six es demostrable en K, entonces es váüda en todos los modelos de Kripke;

si es demostrable en Kn, es válida en todos los modelos transitivos; y si esdemostrable en G, enionces es válida en todos los modelos terminalestransiüvos.

14. Es cierto que existen sistemas estables consistentes de tipo G: por

ejemplo, la máquina de Fergusson y Cralg,. y el sistema modal G. Estos

sistemas, aunque son consistentes, no pueden demostrar su propia con-

sistencia.15. Los sistemas modales R, fr y G no sólo son consistentes y esla

bles, sino autorreferentemente correctos. Lo mismo vale para los sistemas

K' K. ' Y G'i6. Nlngrr.to de los sistemas G* o Q es autorreferentemente corrcct(),

pero amboJson consistentes. El sistema Q es normal, pero el sistcma <;;

no lo es.17. a) rJ¡ razonador mínimo de tipo G es consistente, pero nt¡r)(:r

puede saberlo.b) Un razonador mínimo de tipo G' es consistente y cree (lrr(' ('s

consistente, pero nunca puede saber que cree quq es consistcntc.

c) Un razonador mínimo de üpo Q cree que es inconsistentc, l)( ' t ( , r",

en realidad consistente.

Esto parece un buen punto final. Hay muchas más cosas fítst ttr,ttrl, ,

sob¡e el sistema modal G. Por lejos, la mejor referencia general ( lrr( ' ( ' rr '1, '

actualmente es The Unprouability of Corsistency de Boolos, (lrrr' r'

comiendo sinceramente para leer a continuaciÓn de este l i l>ro l ' , ¡r ' ¡ , ' ,

timular la curiosidad del lector, voy a darles un ejemplo dc ttrt trt:tp,tttlt, , ,

¡esultado -un teorema de punto fijo- cuya demostraciÓn f)tr(r(l(' ('tt, r,rl

trarse allí.consideremos una fórmula modal conuna sola variallk: l)t ( )l ){ ):,rr ir ril I l.

drgamos la letra p. Denotemos esa fó¡mula como A(p). I) i t t l t lor l , t I ¡r ,r ' | r 'Jr l

modal S, A(S) siSnif ica el resultado de poner S cadavc't ( lrr( ' :r l r ,rr |r r ' I ' , rr

A(p). Por ejemplo, si A(p) es la fÓrmula p)Bp, enton(:cs A(:i) t':. l,r ¡ ¡t t' r, ,tt

BS=S. Decimos que una oraciÓn S se denomina punkt. l i l t r l r ' At l ' t ' , r I ¡

oración S=A(S) es demostrable en G. En el problcnr:r 4 r l ' l { .r l ,r l i l l , ' l ' , , I 'pedimos al lector que encontrara una proposición p tnl t¡ t t t ' l r x I r t ¡ rn' .¡ , l , ,¡

de t ipo G creyera p=-Bp, y encontramos que -1l l ( ' r ; l l l t r r " r r i t r '

t " r r l " ' ,

lo tanto todo razonador de t ipo G creerá -Bl-=-l ]"- l l l , y r\ i r ' \ l , l rr i ¡r l . ,rr

esdemosyableenG.Estosigni f icaque-B-Lesur lJ) l I t l l r | | l ¡ r r r l r ' l , r f i , r I I i t I | ¡-Bp. La fÓrmula B-p también üene un punt() l r lo, r l r r r ' r

" l l l , ' " . r"

descubr imOS en la soluciÓn del problema 5, ( :e l r r l l t l ( ! l ' ) l ' .1 . t ¡ ,1 l , r l r r ¡

podría resultar un ejercicio provechoso t¡atar tltt tlcl¡ tr tqi! =r I r ¡tti: l,t lirr i lr r I r

Bp>BI tiene un punto fijo, a sabe¡ BBI-rD l.

No toda fÓrmula A(p) úene un punto f i io, ¡r ' rr t ' i r : l l i ¡r l t ' , i ' ¡ [¡=rt¡¡r¡ l ¡ i

Page 111: SMULLYAN, R. Juegos por siempre misteriosos. España. Gedisa 2008

217

I

no lo tiene (de lo contrario el sistema G sería inconsistente. lo cual sabemosque no es asO. Ahora bien, una fórmula A(p) se denomina rnodallzada enp si p sólo aparece en la parte de A(p) de la forma BX, donde X es unafó_rmula. (Ejemplos: Bp>BBp es modalizada en p; Bp>p no lo es, peroB(Bp=p) lo es.) Los lógicos Claudio Bernardi y C. Smórynski han demos-trado en forma independiente que toda fórmula A(p) que es modalizadae.n p tiene un pr¡nto fijo S. Además, la fórmula B(trA(p))¡B(FS) esdemostrable en G. Este resultado se conoce como el .Teoñma dél puntoFijo de Bernardi-Smorynski".

Los puntos fijos son algo notable. En virtud de la corrección auto_rreferente del sistema G, todo punto fijo S de una fórmula A(p) no sólo esdemostrable en G si y sólo si A(S) es dernostrable, sino también uerdadera(para G) si y sólo si A(S) es verdadera, porque la demostrabilidad de S=A(S)implica su verdad. Digamos que una fórmula A(p) uate para una oraciónS si A(S) es verdadera (para G). Un punto fijo de una fórmula puedeentonces considerarse como una oración que afirma que la fórmuia valepara esa misma oración.

- De modo más general, consideremos una fórmula.t(p,q) que no üene

más.variables proposicionales que p y q. para toda fórmulá X, ACX,q)significa el resultado de reemplazar p por X en A(p,q). Un punto fijo deA(p,q) significa una fórmula H que no tiene otra variábie más que q, dl quelafórmula H=A(H,q) es demosrable en G. Los lógicos D. H. J. de ¡onghy Giovanni Sambin han demostrado que si A(p,q) es modalizada en p (peiono.neces_ariamente en q), entonces A(p,q) üene un punto fijo H; además,la fórmula B(fA(p,q))¡B(p=H) es demosrrable en G.. Ya hay ejemplos conocidos desde el capítulo 19. Bq es un punto fijo

de p!n:C). por el problema 6, y Bq:q es un punro fijo-de Bpiq, po, álproblema 7. En realidad, la reflexividad es eqüivalente a la eiistén'cia deun.punto ñjo pan Bp>q. Lo notable es que para un sistema modal de tipo4, Ia existencia de- un punro fijo sólo para la fórmula Bp>q basta párag^r^ntizar punros fijos para todaslas fórmulas A(p,q) qué son modiliza-das en p. Una demostración de este caso tambié; pugáe encontrarse enBoolos.

COMENTARIOS FINALES

Espero haberle dado al lector alguna idea de por qué estos sister¡asmodales son tan interesantes. Hemos visto que pueden interpretarse tantointernamente (autorreferentemente) como extérnamente (aplicados a lademostrabilidad en otros sistemas matemáticos), así como a los procesosde razonamiento, tanto para los seres naturalmen¡e inteligentes (algunoshumanos y otros animales) como para mecanismos artifiáahentelrcü_gentes (tales como l¿s computadoras).

. Las aplicaciones que esto pueda tener en el campo de la psicología esalgo que bien podría merecer una investigación mái amplii.

216

Es una feliz vuelta del destino que la lógica modal, que históricamentesurgió de intereses puramente filosóficos, haya resultado ser tan impor-tante hoy en día en la teoría de la demost¡ación y la ciencia de la compu-tación. Esto fue una consecuencia de los Teoremas de Gódel y Lób y deltrabajo de aquellos que posteriormente han considerado la teoría de la de-mostración a partir del enfoque teórico modal. Y ahora hasta aquellos filó-sofos que en el pasado miraron con desaprobación la importancia de lalógica modal están obligados a reconocer su importancia matemática.

La antigua oposición filosófica alalígica modal se ha basado aproxi-madamente en tres creencias enteramente diferentes (e incompatibles):Primero, están aquellos que creen que todo lo verdadero es necesa-riamente verdadero, y en consecuencia no hay ninguna diferencia entreverdad y verdad necesaria. Después, están aquellos que creen que nadaes necesariamente verdadero, y en consecuencia que para toda pro-posición p, la proposición Np (p es necesariamente verdadera) direc-tamente es falsa. Y en tercer lugar, están aquellos que afirman que laspalabras 'necesariamente verdadera" no significan absolutamente nada. Ypor 1o tanto cada uno de esos tipos filosóficos ha rechazado la lógica mo-dal según sus propios puntos de vista. I{asta un conocido filósofo tienefama de haber sugerido que la lógica modal moderna fue concebida enpecado. A lo cual Boolos ha respondido apropiadamente: "Si la \ógicamodal moderna se concibió en pecado, entonces ha sido redimida porGódel".r

|Gódlín¿ss usa el autor en un iuego de palabras intraducible, ya que, además de la¡efe¡encia implícita a Gódel, s.ignifica religiosidad, devoción. [T.]

-